You are on page 1of 107

This Copy is for Dr.

Mohamed ElHodiby
Five pregnant women (A, B, C, D, E) have been screened for hepatitis B during pregnancy. The
Question 1
results are shown in the table (click icon to view). The hepatitis B status of patient C is
Five pregnant women (A, B, C, D, E) have been screened for hepatitis B during pregnancy. The
Question 2
results are shown in the table (click icon to view). The hepatitis B status of patient D is
Five pregnant women (A, B, C, D, E) have been screened for hepatitis B during pregnancy. The
Question 3
results are shown in the table (click icon to view). The hepatitis B status of patient E is
Options for Questions 1-3

A Previously vaccinated B Previous infection with immunity


C Chronic carrier D Acute infection
E Susceptible

A(Correct answ er: A)

A(Correct answ er: D)

A(Correct answ er: C)

Explanation
Hepatitis B Serology
• HBsAg detected before HBeAg - both detectable before the onset of jaundice. HBeAg levels fall rapidly
• Anti-HBs antibody detectable at the onset of jaundice and before anti-HBe antibodies
• Anti HBs antibody detected about 4 weeks after the onset of jaundice
• Detection of HBeAg indicates high infectivity
• Detection of anti-HBs IgG indicates previous infection or vaccination
• Chronic carriage associated with persistence of HBsAg
Tests Results Interpretation
Negative
HBsAg
Negative
anti-HBc Susceptible
Negative
IgG anti-HBs

HBsAg Negative
anti-HBc Positive Immune due to natural infection
IgG anti-HBs Positive
HBsAg Negative
anti-HBc Negative Immune due to vaccination
IgG anti-HBs Positive
HBsAg
Positive
anti-HBc
Positive
IgM anti-HBc Acute infection
Positive
IgG anti-HBs
Negative

HBsAg Positive
anti-HBc Positive
Chronic infection
IgM anti-HBc Negative
IgG anti-HBs Negative
Adapted from Centre for Diseases Control & Prevention

This Copy is for Dr. Mohamed ElHodiby


Question 4 Hepatitis A virus

Options for Questions 4-4

A Is a DNA virus B Is associated with chronic liver disease


Can be transmitted from mother to fetus during Infection can be prevented using an inactivated
C D
child birth virus vaccine
E Has an incubation period of 3-7 weeks

A(Correct answ er: D)

Explanation
HEPATITIS A
• RNA virus
• Spread by the feco-oral route although sexual transmission may occur
• Not vertically transmissible
• Incubation period 2-3 weeks
• Does not result in chronic carriage or chronic liver disease
• 20-50% of adults in UK have been infected
• formaldehyde inactivated vaccine is available

A healthy 32 year old woman phones the maternity assessment unit because her 5 year old
Question 5
son has developed chickenpox and the woman is 16 weeks pregnant
Options for Questions 5-5

She should be reassured if she has had


A She should be reassured B
chickenpox in the past
She should be reassured and tested for immunity
C D She should be treated with immune globulin
if she has had chickenpox in the past
E She should be treated with acyclovir

A(Correct answ er: C)

Explanation
VARICELLA ZOSTER
• DNA virus, member of the Herpes family of viruses
• Incubation period 10-21 days
• Spread by droplet infection
• Contagious 48h before onset of rash until all vesicles have crusted over
• Severs infection may cause pneumonitis, encephalitis, myocarditis and adrenal insufficiency
• Virus remains dormant in the dorsal root ganglia, causing recurrent infection (shingles, Herpes zoster). Shingles is
contagious
• Infection in pregnancy is more severe, with 10% risk of pneumonitis and up to 6% mortality
• 2% risk of congenital varicella syndrome (dermatomal scarring, cataract, limb hypoplasia, microcephaly, learning
disability..) if infection occurs BEFORE 20 weeks gestation. No risk of congenital anomalies after 28 weeks
• Infection results in life-long immunity.
• There is no effective vaccine currently available
• Infection can be prevented by passive immunisation with varicella zoster immune globulin – not life-long
• 85% of pregnant women are immune (UK)

This Copy is for Dr. Mohamed ElHodiby


• Severe neonatal disease likely (30%) if maternal disease develops within 4 days of delivery – neonate has not
acquired maternal antibodies
• Sensitive to acyclovir – useful in severe disease

In HIV positive women treated with highly active anti-retroviral therapy, which one is not
Question 6
associated with an increased risk of vertical transmission?
Options for Questions 6-6

A High viral load B Birth before 32 weeks gestation


C Birth after 39 weeks gestation D Short duration of anti-retroviral therapy
E Advanced maternal HIV disease

A(Correct answ er: C)

Explanation
Vertical transmission of HIV
• The risk of vertical transmission: 15-20% in untreated, non-breast-feeding woman in Europe and 25-40% in untreated
breast-feeding woman in Africa.
• The risk of vertical transmission in women treated with Highly Active Anti-retroviral Therapy (HAART) in Europe is
less than 2%. In women treated with HAART and in whom the viral load is less than 50 copies / ml, the risk of vertical
transmission is less than 1% irrespective of mode of delivery.
The risk factors for vertical transmission in treated women include:
1. High plasma viraemia at delivery
2. Short duration of HAART therapy
3. Delivery before 32 weeks
• Less than 2% of vertical transmission occurs in the first / second trimesters. Over 80% of vertical transmission occurs
around the time of labour and delivery
Maternal factors associated with increased risk of vertical transmission include
1. Advanced maternal HIV disease
2. Low antenatal CD4 count
3. High plasma viral load – this being the strongest predictor
Obstetric risk factors associated with increased risk of vertical transmission include
1. Vaginal delivery
2. Duration of membrane rupture
3. Chorioamnionitis
4. Pre-term delivery
5. Breast-feeding (associated with a 2-fold increase in vertical transmission rate).

Question 7 The H1N1 virus

Options for Questions 7-7

A Is an influenza B virus B Is transmitted from pigs to humans


C Undergoes antigenic shift D Does not undergo antigenic drift
E Infection does not result in antibody production

A(Correct answ er: C)

Explanation
Classification of influenza viruses
Three main genera: Influenza A, B & C
Influenza A
• Most pathogenic influenza virus in humans

This Copy is for Dr. Mohamed ElHodiby


• Undergoes antigenic shift and antigenic drift, resulting in the production of new variants that evade host immunity with
the potential for pandemics
• Divided into several serotypes based on the antibody binding to the Hemagglutinin and Neuraminidase proteins:
1. H1N1 – Caused 2009 pandemic (Swine flu virus). This virus is not endemic in pigs and is not transmitted from pigs to
humans. It is a recombination of several strains of H1N1 virus found in humans, birds and pigs
2. H2N2 and H3N2 have caused pandemics in Asia
3. H5N1 – Causes highly lethal pneumonia

Question 8 Which drug should be used to treat breast-feeding women infected with H1N1 influenza?

Options for Questions 8-8

A Acyclovir B Oseltamivir
C Zanamivir D Amantadine
E Rimantadine

A(Correct answ er: B)

Explanation
Influenza – Treatment
Symptomatic treatment including paracetamol for fever and as analgesia
Anti-viral agents:
1) Neuraminidase inhibitors: oseltamivir (Tamiflu ®) and zanamivir (Relenza ®)
• Interfere with the release of new viruses from infected cells.
• Most effective if started within a few hours of the onset of symptoms.
• Licensed for use in adults within 48h of the onset of symptoms but may be administered within 7 days of onset.
• Reduce the duration of symptoms by 1-1.5 days and can reduce the risk of complications in high risk patients
Post-exposure prophylaxis: oseltamivir should be given within 48h of exposure while zanamivir should be given within
36h
Drug treatment in pregnancy
• Zanamivir is the preferred drug during pregnancy
• Oseltamivir recommended in severe infection or when zanamivir cannot be used
• Oseltamivir is the preferred drug in women who are breastfeeding
• Oseltamivir: administered orally. Prodrug, hydrolysed by the liver to active metabolite. Extensively metabolised by the
placenta with minimal fetal drug accumulation. Present in low concentration in breast milk
• Use with caution in renal impairment
• Side-effects: Nausea, vomiting, abdominal pain, diarrhoea, headache, conjunctivitis. Less common side-effects
include eczema, hepatitis, GI bleeding, arrhythmia, visual disturbance
• Zanamivir: administered by inhalation of powder (bioavailability is 10-20% after inhaled dose compared to 2% after
oral dose). 90% of absorbed drug excreted unchanged in the urine. Present in breast milk in low concentration.
• Use with caution in patients with asthma and chronic pulmonary disease because of risk of bronchospasm
• Side-effects: bronchospasm, respiratory impairment, angioedema, rticarial, rash
2) M2 protein inhibitors (amantadine, rimantadine): stop the virus infecting cells. High levels of drug resistance have
been observed and not active against influenza B. No longer recommended.

Question 9 Which drug is administered orally to treat H1N1 influenza infection?

Options for Questions 9-9

A Acyclovir B Oseltamivir
C Zanamivir D Amantadine
E Rimantadine

This Copy is for Dr. Mohamed ElHodiby


A(Correct answ er: B)

Explanation
Influenza – Treatment
Symptomatic treatment including paracetamol for fever and as analgesia
Anti-viral agents:
1) Neuraminidase inhibitors: oseltamivir (Tamiflu ®) and zanamivir (Relenza ®)
• Interfere with the release of new viruses from infected cells.
• Most effective if started within a few hours of the onset of symptoms.
• Licensed for use in adults within 48h of the onset of symptoms but may be administered within 7 days of onset.
• Reduce the duration of symptoms by 1-1.5 days and can reduce the risk of complications in high risk patients
Post-exposure prophylaxis: oseltamivir should be given within 48h of exposure while zanamivir should be given within
36h
Drug treatment in pregnancy
• Zanamivir is the preferred drug during pregnancy
• Oseltamivir recommended in severe infection or when zanamivir cannot be used
• Oseltamivir is the preferred drug in women who are breastfeeding
• Oseltamivir: administered orally. Prodrug, hydrolysed by the liver to active metabolite. Extensively metabolised by the
placenta with minimal fetal drug accumulation. Present in low concentration in breast milk
• Use with caution in renal impairment
• Side-effects: Nausea, vomiting, abdominal pain, diarrhoea, headache, conjunctivitis. Less common side-effects
include eczema, hepatitis, GI bleeding, arrhythmia, visual disturbance
• Zanamivir: administered by inhalation of powder (bioavailability is 10-20% after inhaled dose compared to 2% after
oral dose). 90% of absorbed drug excreted unchanged in the urine. Present in breast milk in low concentration.
• Use with caution in patients with asthma and chronic pulmonary disease because of risk of bronchospasm
• Side-effects: bronchospasm, respiratory impairment, angioedema, rticarial, rash
2) M2 protein inhibitors (amantadine, rimantadine): stop the virus infecting cells. High levels of drug resistance have
been observed and not active against influenza B. No longer recommended.

At which stage of pregnancy are women at greatest risk of severe disease following H1N1
Question 10
influenza infection?
Options for Questions 10-10

A First trimester B Second trimester


C Third trimester D 24-48h post-partum
E 4-6 weeks post-partum

A(Correct answ er: C)

Explanation
H1N1 Influenza and pregnancy
• Pregnant women are more likely to have severe disease, be admitted to ITU and die from H1N1 influenza compared
to non-pregnant women. Most reported deaths occurred in women who developed pneumonia and acute respiratory
distress syndrome requiring mechanical ventilation
• Risk of morbidity from seasonal flu is also higher during pregnancy
• The risk is highest in the third trimester compared to early pregnancy or post-partum
• Women with asthma and morbid obesity appear to be at higher risk of severe disease
• Most current data from UKOSS indicates that pregnant women admitted to ITU were less likely to have been treated
with antiviral agents within 2 days of the onset of symptoms when compared with other pregnant women who were
admitted to hospital. This indicates the importance of prompt treatment
• H1N1 infection is associated with pre-term delivery, especially in women admitted to ITU
• Vertical transmission of H1N1 influenza has not been reported and the virus is not known to be present in breast
milk. Breastfeeding should continue in women being treated for H1N1 influenza

This Copy is for Dr. Mohamed ElHodiby


• The H1N1 virus is not terratogenic but there some evidence that pyrexia in early pregnancy may be associated with
an increased risk of neural tube defects

Question 11 With respect to streptococci

Options for Questions 11-11

Strep. Pneumonia is isolated from the nose /


A Streptococci are gram negative bacteria B
throat in up to 70% of healthy individuals
C Group A streptococci are non-haemolytic D Group B streptococci are non-haemolytic
Group A streptococci are isolated from genital
E
tract cultures in 1 in 4 pregnant women

A(Correct answ er: B)

Explanation
STREPTOCOCCI
• Gram positive
• Capsulated, aerobic
• May form chains (S. viridans) or diplococci (S. pneumoniae)
• Grow on blood agar - may cause haemolysis

Alpha-Haemolytic
• Green colour around each colony due to altered haemoglobin
• S. viridans and S. pneumoniae
• Differentiated by Optochin disc test (S. pneumoniae shows zone of inhibition) and bile solubility test (S.
pneumoniae is soluble)
• S. pneumoniae isolated from nose / throat in 70% of healthy individuals
• S. viridans - isolated from the mouth, causes bacterial endocarditis

Beta-Haemolytic
• Complete lysis of red cells
• S. pyogenes, S. agalactiae, S. fecalis
• Differentiated by Lancefield grouping and Bacitracin test
• Lancefield group A: S. pyogenes- causes sore throat, skin and wound infections (erysipelas, impetigo),
rheumatic fever and acute glomerulonephritis. Strains causing scarlet fever produce an exotoxin
• Lancefield group B: S. algalactiae - isolated from vagina, perineum and rectum; causes neonatal
septicaemia and meningitis
• Lancefield group D: S. fecalis - usually non-haemolytic. Isolated from gut, causes UTIs and bacterial
endocarditis

Gamma Haemolytic
• Non haemolytic
• S. fecalis
• Streptococci are characteristically sensitive to penicillin / erythromycin

Question 12 Isolated from the perineum / lower genital tract of 1 in 4 pregnant women

Options for Questions 12-12

A Strep. Viridans B Strep. Pneumonia


C Strep. Algalactiae D Strep. Fecalis
E Strep. Pyogenes

This Copy is for Dr. Mohamed ElHodiby


A(Correct answ er: C)

Explanation
STREPTOCOCCI
• Gram positive
• Capsulated, aerobic
• May form chains (S. viridans) or diplococci (S. pneumoniae)
• Grow on blood agar - may cause haemolysis

Alpha-Haemolytic
• Green colour around each colony due to altered haemoglobin
• S. viridans and S. pneumoniae
• Differentiated by Optochin disc test (S. pneumoniae shows zone of inhibition) and bile solubility test (S.
pneumoniae is soluble)
• S. pneumoniae isolated from nose / throat in 70% of healthy individuals
• S. viridans - isolated from the mouth, causes bacterial endocarditis

Beta-Haemolytic
• Complete lysis of red cells
• S. pyogenes, S. agalactiae, S. fecalis
• Differentiated by Lancefield grouping and Bacitracin test
• Lancefield group A: S. pyogenes- causes sore throat, skin and wound infections (erysipelas, impetigo),
rheumatic fever and acute glomerulonephritis. Strains causing scarlet fever produce an exotoxin
• Lancefield group B: S. algalactiae - isolated from vagina, perineum and rectum; causes neonatal
septicaemia and meningitis
• Lancefield group D: S. fecalis - usually non-haemolytic. Isolated from gut, causes UTIs and bacterial
endocarditis

Gamma Haemolytic
• Non haemolytic
• S. fecalis
Streptococci are characteristically sensitive to penicillin / erythromycin

Question 13 The most likely cause of breast abscess in a breast-feeding mother 7 days post-partum

Options for Questions 13-13

A Staph. Aureus B Staph. Epidermidis


C Strep. Pneumonia D Strep. Fecalis
E E. Coli

A(Correct answ er: A)

Explanation
STAPHYLOCOCCI
• Gram positive
• Grow on blood agar
• Differentiated by coagulase test
• S. aureus: coagulase positive - produce clot when mixed with serum. Also catalase positive and produces
exotoxin (toxic shock syndrome, toxic epidermal necrolysis). Non-motile
• S. aureus produces yellow - orange pigmented colonies
• Plasmid mediated antibiotic resistance - can spread between different strains by transducing phages.
Produces beta-lactamase (penicillinase). Sensitive to flucloxacillin
• Methicillin-resistant S. aureus is resistant to flucloxacillin but usually sensitive to vancomycin
• S. aureus diseases include wound, bone and joint infections, breast abscess, skin and food-borne
infections, toxic shock syndrome. Isolated from the nose in 10-30% of healthy individuals

This Copy is for Dr. Mohamed ElHodiby


S. epidermidis is coagulase negative

Question 14 Which bacteria typically cause toxic shock syndrome?

Options for Questions 14-14

A Group A streptococci B Group B streptococci


C Coagulase negative staphylococci D Coagulase positive staphylococci
E Actinomycetes Israelii

A(Correct answ er: D)

Explanation
TOXIC SHOCK SYNDROME (TSS)
• Rare, life-threatening inflammatory response syndrome characterized by fever, rash, hypotension,
constitutional symptoms, and multi-organ involvement
• Most often associated with the use of super-absorbent tampons and occasionally with the use of
contraceptive sponges.
• Associated with infection with toxigenic strains of Staphylococcus aureus
• The infection can also affect men, children and postmenopausal women.
• Signs and symptoms develop suddenly
• TSS has been linked to many bacterial infections, including pneumonia, osteomyelitis, sinusitis, and
skin and gynecologic infections
• Endotoxin toxic shock syndrome toxin-1 (TSST-1) is the major toxin produced by strains of S aureus
that are responsible for causing TSS.
• Streptococcus pyogenes exotoxin A (SPEA) and S pyogenes exotoxin B (SPEB) are the major
toxins produced by group A beta-hemolytic streptococci.
The toxins activate production of cytokines, such as tumor necrosis factor, interleukin-1, M protein, and gamma-
interferon and induction of nitric oxide production which contributes to hypotension

Question 15 Which organism typically causes IUCD-associated genital tract infections?

Options for Questions 15-15

A Group B streptococcus B Group A streptococcus


C Staphylococcus aureus D Clostridium perfringens
E Actinomycetes israelii

A(Correct answ er: E)

Explanation
ACTINOMYCETE ISRAELII
• Gram positive branching filamentous bacillus
• Anaerobic
• Form part of the normal flora in the mouth, vagina and rectum.
• Non-acid -fast (Nocardia are weakly acid-fast)
• Infection is characterised by sulphur (yellow) granules in pus
• Does not involve the lymphatics
• Causes cervico-facial, pulmonary and abdominal infection, causing multiple discharging sinuses
• Genital tract infection associated with the IUCD
• Sensitive to penicillin

This Copy is for Dr. Mohamed ElHodiby


Question 16 Gas gangrene is caused by

Options for Questions 16-16

A Actinomycetes israelii B Clostridium perfringens


C Clostridium difficile D Bacillus anthracis
E Staphylococcus epidermidis

A(Correct answ er: B)

Explanation
CLOSTRIDIA
• Gram positive anaerobic bacilli
• Spore-forming -the spores are resistant to boiling and chemical disinfectants
• Commonly found in the environment and as part of normal fecal flora
• Produce EXOTOXINS
• Infection common in gangrenous / ischaemic tissue and in association with foreign bodies
• Sensitive to metronidazole and penicillin
• CLOSTRIDIUN DIFFICILE - causes pseudo-membranous colitis - diarrhoea with collapse following broad
spectrum antibiotic treatment - treated with oral metronidazole or vancomycin
• CLOSTRIDIUM PERFRINGENS - Causes gas gangrene (causing crepitus) and septic abortion (rare)
• Produces exotoxin causing septic shock, jaundice, haemolysis and acute renal failure
Sensitive to penicillin - surgical debridement usually required

Question 17 Clostridium tetani is sensitive to the following antibiotics

Options for Questions 17-17

A Penicillin and metronidazole B Cefuroxime and metronidazole


C Penicillin and flucloxacillin D Gentamicin and doxycycline
E Rifampicin and trimrthoprim

A(Correct answ er: C)

Explanation
Clostridium Tetani
• Flagellated bacilli. Spores are resistant to boiling and autoclaving is required
• Incubation period 3-21 days
• Produces EXOTOXIN which is non-immunogenic therefore infection does not confer immunity.
Neurotoxin, inhibits release of inhibitory neurotransmitters in the CNS causing tetany / muscle spasms
• There is no local inflammation associated with tissue infection
• Infection prevented by vaccination with tetanus toxoid in infancy followed by a booster dose every 5-10
years
• Passive immunisation with human tetanus immune globulin + active immunisation required in at-risk
individuals following injury
Antibiotic treatment is an ancillary measure and should include penicillin +/- flucloxacillin. Removal of devitalised
tissue, good tissue perfusion and oxygenation are essential

This Copy is for Dr. Mohamed ElHodiby


Question 18 Endotoxins

Options for Questions 18-18

A Are polypeptide molecules B Are produced by living bacteria


C Stimulate the production of natural anti-toxins D Are converted to non-toxic toxoid by formalin
Are associated with disseminated intra-vascular
E
coagulation

A(Correct answ er: E)

Explanation
EXOTOXINS
• Highly toxic polypeptides produced by living bacteria
• Act at specific target sites
• Are highly antigenic (except Cl. Tetani toxin)
• Are converted into antigenic non-toxic toxoids by formalin
• Are often destroyed by heat
• Are neutralised by anti-toxins
• Typically produced by gram positive bacteria

ENDOTOXINS
• Are lipopolysaccharide molecules
• Present in the outer layer of the cell wall of gram negative bacteria, released on disintegration
• Relatively heat stable
• Not converted into toxoid by formalin
• Act non-specifically on the reticulo-endothelial system causing increased vascular permeability and
prostaglandin release
• May cause DIC and cardiovascular collapse
• Detectable by the Limulus test (A lysate of blood cells from the horseshoe crab is used to test specimens
for the presence of Escherichia coli endotoxin in blood or milk)
• Do not stimulate production of natural anti-toxins
Produced by E. coli, N. meningitides, S. typhi, P. aeruginosa

Question 19 Which antibiotic is used to treat syphilis infection in the first trimester of pregnancy?
Options for Questions 19-19

A Azithromycin B Erythromycin
C Procaine penicillin D Clindamycin
E Tazocin

A(Correct answ er: C)

Explanation
Treatment of syphilis
• Procaine penicillin is first line treatment
• Treponema pallidum is also sensitive to erythromycin and tetracycline although the results of treatment are less
satisfactory
• The Jerisch-Herxheimer reaction is an inflammatory response to the increased release of treponemal antigens during
treatment -managed with steroids

This Copy is for Dr. Mohamed ElHodiby


Question 20 Which infection is not caused by Chlamydia trachomatis?

Options for Questions 20-20

A Salpingitis B Conjunctivitis
C Bartholinitis D Interstitial pneumonitis
E Pharyngitis

A(Correct answ er: E)

Explanation
CHLAMYDIA TRACHOMATIS
• Obligate intracellular organism - bacteria
• Non-motile gram negative or gram variable although gram stain is not useful in diagnosis
• Enter the host through abrasions in mucosal surfaces
• Does not infect squamous cells - the vaginal epithelium and ectocervix are not infected. Endocervical glands are
susceptible
• Produce infectious particles called Elementary Bodies which enter cells by endocytosis
• Differentiate into Reticulate Bodies within cells. These multiply by binary fission and then produce other elementary
bodies which are released from the cell
• Asymptomatic infection is common
• C. psittaci infects birds and non-human animals
Various serovars:
• A - C: Trachoma (blindness)
• D - K: genital infection - cervicitis, urethritis, salpingitis, epididymitis, proctitis, Bartholinitis, conjunctivitis, interstitial
pneumonitis in neonates.
• Consequences of infection include infertility, ectopic pregnancy, Fitz-Hugh-Curtis syndrome, Reiter's syndrome,
arthritis, dermatitis
• L1-3: Lymphogranuloma venereum

A 23 year old woman presents with an itchy vaginal discharge at 22 weeks gestation.
Question 21 Microscopy of high vaginal swab shows trichomonas vaginalis. Which one is the most
appropriate treatment?
Options for Questions 21-21

A Azithromycin B Clotrimazole
C Clindamycin D Metronidazole
E Cefalexine

A(Correct answ er: D)

Explanation
TRICHOMONAS VAGINALIS
• Flagellated protozoan, anaerobic, readily cultured on Diamond’s medium
• Sexually transmitted
• Infection is different from bacterial vaginosis although clinical differentiation is difficult
• Characteristic strawberry cervix only found in 3% of cases
• Diagnosed by microscopy of wet vaginal smear or by culture
• The organism is difficult to detect in males but can be isolated from prostatic fluid
• Also causes urethritis, cystitis and Bartholinitis

This Copy is for Dr. Mohamed ElHodiby


• Vaginal pH > 4.5
Treated with metronidazole

Question 22 Which antibiotic is not used to treat Candida albicans infection?

Options for Questions 22-22

A Fluconazole B Nystatin
C Amphotericin B D Clotrimazole
E Azithromycin

A(Correct answ er: E)

Explanation
CANDIDA ALBICANS
• Commensal organism in the mouth, gut or vagina of 40-80% of individuals
• Causes candidiasis
• Yeast-like fungus, grows by budding and may produce long non-branching filaments called pseudohyphae.
Candida is a polymorphic yeast, i.e., yeast cells, hyphae and pseudohyphae are produced
• Gram positive anaerobic organism
• Causes local infection of the vagina, mouth and skin
• Increased risk of colonisation / infection in pregnancy (especially third trimester), steroid / broad spectrum
antibiotic use, diabetes mellitus. Increased risk in oral contraceptive users is disputed. Causes disseminated
infection in the immunocompromised
• Culture - Sabouraud's medium - pH ~ 5.6
• Vaginal infection associated with a pH at the low end of normal (3-4)
• Sensitive to polyene antibiotics (nystatin / amphotericin B)
• Sensitive to imidazole derivatives - clotrimazole, econazole, floconazole, miconazole
• Rarely isolated from the vagina in pre-pubertal or post-menopausal women

Question 23 With respect to infection with Toxoplasma gondii during pregnancy

Options for Questions 23-23

The risk of fetal infection decreases with The risk of the fetus being affected decreases
A B
increasing gestation age with increasing gestation age
Maternal infection typically presents with Infection in pregnancy is treated with
C D
septicaemia erythromycin
Infection in pregnancy is typically detected during
E
routine screening

A(Correct answ er: B)

Explanation
• Obligate intracellular protozoan
• Completes part of its life cycle in animals (mainly cats but also sheep, cattle and pigs)
• Contracted by contact with cat litter, infected soil or eating undercooked beef, lamb or pork. Prevented by
wearing gloves during gardening, thorough food hygiene and avoiding contact with cat litter
• Oocysts or tissue cysts invades the intestinal epithelium , develop into trophozoites which spread via the
lymphatic system
• Infection is usually mild or asymptomatic with lymphadenopathy (posterior cervical) being the main
presentation. Causes severe disease with encephalitis and chorioretinitis in the immunocompromised
• Risk of fetal infection increases with increasing gestation age (17% first, 25% second and 65% third
trimester)
• Risk of fetus being affected FALLS with increasing gestation age

This Copy is for Dr. Mohamed ElHodiby


• Recognised cause of congenital anomalies with chorioretinitis, intra-cranial calcification, microcephaly,
hydrocephalus. Increased risk of miscarriage with first trimester infection
• Diagnosis can be made by culture in laboratory rodents
• Detection of IgM or a 4 fold rise in IgG titres in samples 3 weeks apart is indicative of acute infection
Sensitive to spiramycin and pyramethamine

Question 24 Which organism is typically detected using the Zeihl-Neelsen stain?

Options for Questions 24-24

A Toxoplasma gondii B Listeria monocytogenes


C Mycobacterium tuberculosis D Treponema pallidum
E Plasmodium falciparum

A(Correct answ er: C)

Explanation
MYCOBACTERIA
• Cause tuberculosis, leprosy in addition to other opportunistic infections
• Present as 2-4 micron long acid-fast bacilli on the Ziehl-Neelsen stain
• Multiply intracellularly in macrophages but also divide extra-cellularly
• Infection results in delayed hypersensitivity response
• Not all mycobacteria are pathogenic in humans
• All mycobacteria are:

1. Acid fast- i.e. they do not de-stain with acid and alcohol once stained with arylmethane dye
2. Aerobic
3. Contain mycolic acids
4. Intracellular pathogens- the wall helps the organism to survive within the macrophage by resisting oxidative
damage

Question 25 Mycobacterium tuberculosis

Options for Questions 25-25

A Is an anaerobic organism B Is an intra-cellular organism


Infection is associated with a Type II hyper-
C D Can only divide within infected cells
sensitivity reaction
E Does not multiply within macrophages

A(Correct answ er: B)

Explanation
MYCOBACTERIA
• Cause tuberculosis, leprosy in addition to other opportunistic infections
• Present as 2-4 micron long acid-fast bacilli on the Ziehl-Neelsen stain
• Multiply intracellularly in macrophages but also divide extra-cellularly
• Infection results in delayed hypersensitivity response
• Not all mycobacteria are pathogenic in humans
• All mycobacteria are:

1. Acid fast- i.e. they do not de-stain with acid and alcohol once stained with arylmethane dye
2. Aerobic
3. Contain mycolic acids

This Copy is for Dr. Mohamed ElHodiby


4. Intracellular pathogens- the wall helps the organism to survive within the macrophage by resisting oxidative
damage

Question 26 Which antibiotic does not act by inhibiting bacterial cell wall synthesis?

Options for Questions 26-26

A Penicillin B Cefalexine
C Vancomycin D Teicoplanin
E Gentamicin

A(Correct answ er: E)

Explanation
ANTIBIOTICS AND CELL WALL SYNTHESIS
• Beta lactams - inhibit transpeptidation (penicillins, monobactams, carbapenams and ceplalosporins)
• Cycloserine - inhibits reactions incorporating alanine into cell wall precursor within the cytoplasm
• Glycopeptides - vancomycin, teicoplanin - bind to terminal d-alanyl-d-alanine residue, preventing
incorporation into growing peptidoglycan chain
• Bacitracin - prevents dephophorylation of phosphilipid carrier, preventing regeneration of carrier required
for continuous synthesis
GLYCOPEPTIDES
• Vancomycin and teicoplanin
• Inhibit bacterial cell wall synthesis by binding to d-alanyl-d-alanine at the end of a pentapeptide chain,
preventing the incorporation of new subunits
• Large molecules which cannot penetrate gram negative cell wall. Only active against gram positive
bacteria
• Not absorbed from the GI tract. Oral administration used to treat Clostridium difficile diarrhoea
• Potentially ototoxic and nephrotoxic - vancomycin should be administered slowly to prevent 'red-man'
syndrome. Teicoplanin is less toxic and can be given bolus

Question 27 Glycopeptide antibiotics

Options for Questions 27-27

A Are not absorbed from the gastro-intestinal tract B Include gentamicin


C Inhibit bacterial cell membrane synthesis D Are effective against E. coli infections
E Cannot be administered by intravenous route

A(Correct answ er: A)

Explanation
GLYCOPEPTIDES
• Vancomycin and teicoplanin
• Inhibit bacterial cell wall synthesis by binding to d-alanyl-d-alanine at the end of a pentapeptide chain,
preventing the incorporation of new subunits
• Large molecules which cannot penetrate gram negative cell wall. Only active against gram positive
bacteria
• Not absorbed from the GI tract. Oral administration used to treat Clostridium difficile diarrhoea
• Potentially ototoxic and nephrotoxic - vancomycin should be administered slowly to prevent 'red-man'
syndrome. Teicoplanin is less toxic and can be given bolus

This Copy is for Dr. Mohamed ElHodiby


Question 28 Which statement regarding aminoglycoside antibiotics is not true?

Options for Questions 28-28

A They are potentially nephrotoxic B They are potentially ototoxic


They should be administered through the
C They cross the blood-brain barrier very well D
intravenous route
E They are effective against gram negative bacteria

A(Correct answ er: C)

Explanation
AMINOGLYCOSIDES
• Gentamicin, tobramycin, netilmicin, streptomycin
• Inhibit bacterial protein synthesis - interfere with the binding of formylmethionyl-transfer RNA to the
ribosomes, preventing the formation of the initiation complex
• Not absorbed from the GI tract. Do not cross the blood-brain barrier
• Not active against streptococci. Active against gram negative bacteria
• Excreted via the kidneys
• Potentially ototoxic and nephrotoxic
• Production of aminoglycoside modifying enzymes is the most important mechanism of acquired bacterial
resistance - usually plasmid mediated

Question 29 Which antibiotic is associated with dental dis-colouration if it is used during pregnancy?

Options for Questions 29-29

A Tetracycline B Azithromycin
C Trimethoprim D Chloramphenicol
E Clindamycin

A(Correct answ er: A)

Explanation
TETRACYCLINES
• Inhibit bacterial protein synthesis by preventing aminoacyl tRNA from entering the acceptor site on
ribosomes
• Absorbed from the gut. Doxycycline and minocycline are better absorbed than the others
• Penetrate tissues well and are active against intracellular bacteria
• Effective against chlamydial, mycoplasma and rickettsiae infections
• Cross placenta - interferes with bone development and cause brown discolouration of teeth -
contraindicated in pregnancy and children
Widespread resistance as a result of their use as growth promoters in animal feed - resistance gene carried on a
transposonv

Question 30 Which statement regarding trimethoprim is not true?

Options for Questions 30-30

A It is a folic acid analogue B It inhibits dihydrofolate reductase


It is mainly used to treat infections with gram
C D It is well absorbed from the GI tract
positive bacteria
E It is predominantly excreted by the kidneys

This Copy is for Dr. Mohamed ElHodiby


A(Correct answ er: C)

Explanation
TRIMETHOPRIM
• Folic acid analogue
• Inhibits dihydrofolate reductase
• Similar structure and mechanism of action to methotrexate and the anti-malarial agent pyrimethamine
• Absorbed from the gut
• Excreted by the kidneys
• Effective against gram negative bacilli EXCEPT pseudomonas
• Resistance is mediated by a plasmid-encoded dihydrofolate reductase which has a much reduced affinity for
trimethoprim

This Copy is for Dr. Mohamed ElHodiby


Question 1 Which one is not a recognized mechanism of action of antibiotics?

Options for Questions 1-1

A Inhibition of bacterial cell wall synthesis B Disruption of bacterial cell membrane function
C Inhibition of mitochondrial function D Inhibition of bacterial protein synthesis
E Inhibition of nucleic acid synthesis

A(Correct answ er: C)

Explanation
Antibiotics act at 4 main sites
• Cell wall synthesis
• Protein synthesis
• Nucleic acid synthesis
• Cell membrane function

Question 2 Which statement regarding beta lactam antibiotics is not true?

Options for Questions 2-2

A Inhibit bacterial cell membrane synthesis B Are analogues of a-alanyl-d-alanine


C Include penicillin D Include cephalosporins
Are typically effective against gram positive
E
bacteria

A(Correct answ er: A)

Explanation
BETA-LACTAM ANTIBIOTICS
• Inhibit bacterial cell wall synthesis
• Analogues of d-alanyl-d-alanine
• Cell wall synthesis is dependent on transpeptidation which links soluble peptidoglycan precursors into the
insoluble growing peptidoglycan chain. This reaction is inhibited by beta-lactam antibiotics - act outside the
cell membrane
• Effective against all gram positive bacteria and gram negative cocci
• Excreted by the kidneys - penicillin excretion inhibited by probenecid
• Resistance mainly via beta-lactamase production
Include penicillins and cephalosporins

Question 3 Beta lactam antibiotics

Options for Questions 3-3

A Are metabolized by the liver B Are analogues of folic acid


C Are effective against gram negative bacteria D Are effective against gram negative cocci
E Include vancomycin

A(Correct answ er: D)

Explanation

This Copy is for Dr. Mohamed ElHodiby


BETA-LACTAM ANTIBIOTICS
• Inhibit bacterial cell wall synthesis
• Analogues of d-alanyl-d-alanine
• Cell wall synthesis is dependent on transpeptidation which links soluble peptidoglycan precursors into the
insoluble growing peptidoglycan chain. This reaction is inhibited by beta-lactam antibiotics - act outside the
cell membrane
• Effective against all gram positive bacteria and gram negative cocci
• Excreted by the kidneys - penicillin excretion inhibited by probenecid
• Resistance mainly via beta-lactamase production
Include penicillins and cephalosporins

Question 4 Which antibiotic does not act by inhibiting bacterial cell wall synthesis?

Options for Questions 4-4

A Penicillin B Cefalexine
C Vancomycin D Teicoplanin
E Gentamicin

A(Correct answ er: E)

Explanation
ANTIBIOTICS AND CELL WALL SYNTHESIS
• Beta lactams - inhibit transpeptidation (penicillins, monobactams, carbapenams and ceplalosporins)
• Cycloserine - inhibits reactions incorporating alanine into cell wall precursor within the cytoplasm
• Glycopeptides - vancomycin, teicoplanin - bind to terminal d-alanyl-d-alanine residue, preventing
incorporation into growing peptidoglycan chain
• Bacitracin - prevents dephophorylation of phosphilipid carrier, preventing regeneration of carrier required
for continuous synthesis
GLYCOPEPTIDES
• Vancomycin and teicoplanin
• Inhibit bacterial cell wall synthesis by binding to d-alanyl-d-alanine at the end of a pentapeptide chain,
preventing the incorporation of new subunits
• Large molecules which cannot penetrate gram negative cell wall. Only active against gram positive
bacteria
• Not absorbed from the GI tract. Oral administration used to treat Clostridium difficile diarrhoea
• Potentially ototoxic and nephrotoxic - vancomycin should be administered slowly to prevent 'red-man'
syndrome. Teicoplanin is less toxic and can be given bolus

Question 5 Vancomycin

Options for Questions 5-5

A Is a beta lactam antibiotic B Is effective against gram negative bacteria


C Is ineffective if administered orally D Is not absorbed from the gastro-intestinal tract
E Is typically associated with hepatotoxicity

A(Correct answ er: D)

Explanation
GLYCOPEPTIDES
• Vancomycin and teicoplanin

This Copy is for Dr. Mohamed ElHodiby


• Inhibit bacterial cell wall synthesis by binding to d-alanyl-d-alanine at the end of a pentapeptide chain,
preventing the incorporation of new subunits
• Large molecules which cannot penetrate gram negative cell wall. Only active against gram positive
bacteria
• Not absorbed from the GI tract. Oral administration used to treat Clostridium difficile diarrhoea
• Potentially ototoxic and nephrotoxic - vancomycin should be administered slowly to prevent 'red-man'
syndrome. Teicoplanin is less toxic and can be given bolus

Question 6 Which antibiotic is typically associated with red man syndrome?

Options for Questions 6-6

A Penicillin B Gentamicin
C Vancomycin D Flucloxacillin
E Metronidazole

A(Correct answ er: C)

Explanation
GLYCOPEPTIDES
• Vancomycin and teicoplanin
• Inhibit bacterial cell wall synthesis by binding to d-alanyl-d-alanine at the end of a pentapeptide chain,
preventing the incorporation of new subunits
• Large molecules which cannot penetrate gram negative cell wall. Only active against gram positive
bacteria
• Not absorbed from the GI tract. Oral administration used to treat Clostridium difficile diarrhoea
Potentially ototoxic and nephrotoxic - vancomycin should be administered slowly to prevent 'red-man' syndrome.
Teicoplanin is less toxic and can be given bolus

Question 7 Which antibiotic should not be administered by an intravenous bolus?

Options for Questions 7-7

A Benzyl penicillin B Vancomycin


C Cefuroxime D Teicoplanin
E Flucloxacillin

A(Correct answ er: B)

Explanation
GLYCOPEPTIDES
• Vancomycin and teicoplanin
• Inhibit bacterial cell wall synthesis by binding to d-alanyl-d-alanine at the end of a pentapeptide chain,
preventing the incorporation of new subunits
• Large molecules which cannot penetrate gram negative cell wall. Only active against gram positive
bacteria
• Not absorbed from the GI tract. Oral administration used to treat Clostridium difficile diarrhoea
• Potentially ototoxic and nephrotoxic - vancomycin should be administered slowly to prevent 'red-man'
syndrome. Teicoplanin is less toxic and can be given bolus

This Copy is for Dr. Mohamed ElHodiby


Question 8 Which antibiotic is not an aminoglycoside?

Options for Questions 8-8

A Gentamicin B Tobramycin
C Vancomycin D Netilmicin
E Streptomycin

A(Correct answ er: C)

Explanation
AMINOGLYCOSIDES
• Gentamicin, tobramycin, netilmicin, streptomycin
• Inhibit bacterial protein synthesis - interfere with the binding of formylmethionyl-transfer RNA to the
ribosomes, preventing the formation of the initiation complex
• Not absorbed from the GI tract. Do not cross the blood-brain barrier
• Not active against streptococci. Active against gram negative bacteria
• Excreted via the kidneys
• Potentially ototoxic and nephrotoxic
• Production of aminoglycoside modifying enzymes is the most important mechanism of acquired bacterial
resistance - usually plasmid mediated

Question 9 Aminoglycoside antibiotics

Options for Questions 9-9

A Inhibit bacterial protein synthesis B Are effective against streptococci


C Are well absorbed from the GI tract D Are mainly metabolized by the liver
E Include vancomycin

A(Correct answ er: A)

Explanation
AMINOGLYCOSIDES
• Gentamicin, tobramycin, netilmicin, streptomycin
• Inhibit bacterial protein synthesis - interfere with the binding of formylmethionyl-transfer RNA to the
ribosomes, preventing the formation of the initiation complex
• Not absorbed from the GI tract. Do not cross the blood-brain barrier
• Not active against streptococci. Active against gram negative bacteria
• Excreted via the kidneys
• Potentially ototoxic and nephrotoxic
• Production of aminoglycoside modifying enzymes is the most important mechanism of acquired bacterial
resistance - usually plasmid mediated

Question 10 Which antibiotic does not act by inhibiting bacterial protein synthesis?

Options for Questions 10-10

A Tetracycline B Gentamicin
C Netilmicin D Vancomycin
E Chloramplenicol

This Copy is for Dr. Mohamed ElHodiby


A(Correct answ er: D)

Explanation
AMINOGLYCOSIDES
• Gentamicin, tobramycin, netilmicin, streptomycin
• Inhibit bacterial protein synthesis - interfere with the binding of formylmethionyl-transfer RNA to the
ribosomes, preventing the formation of the initiation complex
• Not absorbed from the GI tract. Do not cross the blood-brain barrier
• Not active against streptococci. Active against gram negative bacteria
• Excreted via the kidneys
• Potentially ototoxic and nephrotoxic
• Production of aminoglycoside modifying enzymes is the most important mechanism of acquired bacterial
resistance - usually plasmid mediated
TETRACYCLINES
• Inhibit bacterial protein synthesis by preventing aminoacyl tRNA from entering the acceptor site on
ribosomes
• Absorbed from the gut. Doxycycline and minocycline are better absorbed than the others
• Penetrate tissues well and are active against intracellular bacteria
• Effective against chlamydial, mycoplasma and rickettsiae infections
• Cross placenta - interferes with bone development and cause brown discolouration of teeth -
contraindicated in pregnancy and children
• Widespread resistance as a result of their use as growth promoters in animal feed - resistance gene
carried on a transposon
CHLORAMPHENICOL
• Inhibits bacterial protein synthesis by blocking peptidyl transferase
• Well absorbed from the gut
• Active against a wide range of bacteria. Main indication is the treatment of Salmonella typhi
• Causes REVERSIBLE dose-dependent bone marrow suppression

Question 11 Which antibiotic is associated with dental dis-colouration if it is used during pregnancy?

Options for Questions 11-11

A Tetracycline B Azithromycin
C Trimethoprim D Chloramphenicol
E Clindamycin

A(Correct answ er: A)

Explanation
TETRACYCLINES
• Inhibit bacterial protein synthesis by preventing aminoacyl tRNA from entering the acceptor site on
ribosomes
• Absorbed from the gut. Doxycycline and minocycline are better absorbed than the others
• Penetrate tissues well and are active against intracellular bacteria
• Effective against chlamydial, mycoplasma and rickettsiae infections
• Cross placenta - interferes with bone development and cause brown discolouration of teeth -
contraindicated in pregnancy and children
Widespread resistance as a result of their use as growth promoters in animal feed - resistance gene carried on a
transposonv

This Copy is for Dr. Mohamed ElHodiby


Question 12 Which antibiotic does not inhibit bacterial protein synthesis?

Options for Questions 12-12

A Erythromycin B Clindamycin
C Cefuroxime D Azithromycin
E Tetracycline

A(Correct answ er: C)

Explanation
MACROGLIDES
• Erythromycin, azithromycin, clarithromycin and spiramycin
• Inhibit bacterial protein synthesis by binding to 23S rRNA, blocking translocation
• Well absorbed from the gut; penetrate tissues well and are active against intracellular organisms
• Erythromycin is concentrated in the liver and excreted via bile
CLINDAMYCIN
• Clindamycin belongs to the group of Lincosamides
• Binds to 50S ribosomal subunit, inhibiting peptide bond formation.
• Similar antibiotic spectrum to erythromycin, excreted by the liver, active in faeces for up to 5 days after a
dose
• More active against anaerobes than erythromycin. Clostridium difficile is resistant and may be selected,
causing pseudomembranous colitis

Question 13 Which statement regarding clindamycin is not true?

Options for Questions 13-13

A It acts by inhibiting bacterial protein synthesis B It is mainly excreted by the liver


C It is effective against Clostridium difficile D It can be administered orally
E It is effective against anaerobic bacteria

A(Correct answ er: C)

Explanation
CLINDAMYCIN
• Clindamycin belongs to the group of Lincosamides
• Binds to 50S ribosomal subunit, inhibiting peptide bond formation.
• Similar antibiotic spectrum to erythromycin, excreted by the liver, active in faeces for up to 5 days after a
dose
• More active against anaerobes than erythromycin. Clostridium difficile is resistant and may be selected,
causing pseudomembranous colitis

Question 14 Sulphonamide antibiotics

Options for Questions 14-14

A Are analogues of d-alanyl-d-alanine B Inhibit bacterial cell wall synthesis


Are a recognized cause of Stevens-Johnson
C Inhibit bacterial protein synthesis D
syndrome
E Are metabolized by beta-lactamase

This Copy is for Dr. Mohamed ElHodiby


A(Correct answ er: D)

Explanation
SULPHONAMIDES
• Derivatives of p-aminobenzoic acid
• Inhibit bacterial synthesis of tetrahydrofolic acid
• Active orally, metabolised by the liver and excreted by the kidneys
• Recognised cause of Stevens-Johnson syndrome
• Resistance is widespread - plasmid mediated
• Administered in combination with trimethoprim as COTRIMOXAZOLE

Question 15 Which antibiotic is a folic acid analogue?

Options for Questions 15-15

A Cefuroxime B Trimethoprim
C Erythromycin D Clindamycin
E Tetracycline

A(Correct answ er: B)

Explanation
TRIMETHOPRIM
• Folic acid analogue
• Inhibits dihydrofolate reductase
• Similar structure and mechanism of action to methotrexate and the anti-malarial agent pyrimethamine
• Absorbed from the gut
• Excreted by the kidneys
• Effective against gram negative bacilli EXCEPT pseudomonas
• Resistance is mediated by a plasmid-encoded dihydrofolate reductase which has a much reduced affinity for
trimethoprim

Question 16 Which statement regarding metronidazole is not true?

Options for Questions 16-16

A It is a pro-drug B It is mainly excreted by the liver


C It is effective against anaerobes D It is effective against Clostridium difficile
E It can be administered rectally

A(Correct answ er: B)

Explanation
METRONIDAZOLE
• A nitroimidazole
• Pro-drug, converted to active metabolite by reduction after entering microbial cell
• Well absorbed from the gut / rectum
• Effective only against anaerobes and protozoa such as Entamoeba histolitica, trichomonas vaginalis, Gardnerella
vaginalis, and Giardia lamblia
• Used for surgical prophylaxis following abdominal / pelvic surgery
• Effective treatment for pseudomembranous colitis

This Copy is for Dr. Mohamed ElHodiby


• Rectal administration is an effective alternative to iv therapy
• Causes a disulfiram-like reaction with alcohol
• Excreted by the kidneys

Question 17 Which one is not an anti-fungal drug?

Options for Questions 17-17

A Clotrimazole B Cotrimoxazole
C Flucytosine D Nystatin
E Fluconazole

A(Correct answ er: B)

Explanation
ANTI-FUNGAL AGENTS
• Azole compounds - clotrimazole, ketoconazole, fluconazole: inhibit lanosterol C14 demethylase,an important enzyme
in sterol biosynthesis
• Polyene compounds - amphotericin B, nystatin: bind ergosterol in cell membrane resulting in leakage of cellular
contents
• Flucytosine: pro-drug, taken up by fungal cells and converted into 5-fluorouracil. Inhibits DNA synthesis
• Griseofulvin: inhibits mitosis and nucleic acid synthesis

Question 18 With respect to the sterilization / disinfection of surgical equipment

Options for Questions 18-18

Ultra-violet irradiation is the most effective method


A B Autoclaving utilizes dry heat
of sterilization
Autoclaving is effective against the spores of
C D Boiling is more effective than autoclaving
clostridium botulinum
E Pasteurization uses temperatures of 160-180C

A(Correct answ er: C)

Explanation
STERILISATION / DISINFECTION
• Ultr-violet irradiation - inefficient as a sterilant. Used to inhibit bacterial growth in complex apparatus
• Dry heat -160-180C. Useful for heat resistant equipment
• Moist heat (autoclaving) - most effective method of sterilisation. Usual cycle is 121C for 15 minutes - sufficient to kill
spores of Clostridium botulinum. Heat-sensitive instruments such as endoscopes -use low temperature steam +
formaldehyde
• Boiling - disinfects but does not sterilise -some spores are not killed
• Pasteurisation - reduce microbial load in fluids - 62.8 - 65.6C for 30 minutes
• Gamma irradiation - useful for sterilising large batches of small items such as needles
• Filtration - produces particle-free fluid
• Ethylene oxide -effective sterilant but toxic and explosive
• Formaldehyde -disinfectant. Irritant.
• Iodine / iodophors -kills some spores, inactivated by blood and other organic matter; iodine stains skin and can cause
local sensitivity
• Mercuric chloride - topical skin preparation

This Copy is for Dr. Mohamed ElHodiby


Question 19 Which surgical disinfectant is effective against endospores?

Options for Questions 19-19

A Iodine (3%) B Iodophores (7.5-10%)


C Isopropyl alcohol (70%) D Chlorhexidine (2-4%)
E Triclosan

A(Correct answ er: A)

Explanation
Iodine preparations (3%)
• Excellent activity against gram positive and most gram negative bacteria
• Excellent activity against TB, viruses
• Good activity against fungi
• Only fair activity against endospores
• Activity is markedly affected by organic matter
• Not suitable for use as surgical scrub but can be used for skin prep. Can burn skin so should be removed after
several minutes
• Not suitable for use on mucous membranes
• Iodine may be absorbed by the neonate and affect thyroid function

Question 20 Iodophores used for surgical disinfection

Options for Questions 20-20

A Are aqueous solutions of iodine B Have no activity against endospores


Have no activity against Mycobacterium
C D Have no activity against fungi
tuberculosis
E Activity is unaffected by organic matter

A(Correct answ er: B)

Explanation
Iodophores (7.5 – 10%)
• Preparation containing iodine complexed with a solubilizing agent, such as a surfactant or povidone (povidone-iodine)
• Result in a solution that releases free iodine
• Excellent activity against gram positive and most gram negative bacteria
• Fair activity against TB
• No activity against endospores
• Good activity against viruses and fungi
• Activity is moderately affected by organic matter
• Useful as surgical scrub and skin prep
• Can be used on mucous membranes
• Iodine may be absorbed by the neonate and affect thyroid function

This Copy is for Dr. Mohamed ElHodiby


Five pregnant women (A, B, C, D, E) have been screened for hepatitis B during pregnancy. The
Question 1
results are shown in the table (click icon to view). The hepatitis B status of patient A is
Five pregnant women (A, B, C, D, E) have been screened for hepatitis B during pregnancy. The
Question 2
results are shown in the table (click icon to view). The hepatitis B status of patient E is
Options for Questions 1-2

A Previously vaccinated B Previous infection with immunity


C Chronic carrier D Acute infection
E Susceptible

A(Correct answ er: E)

A(Correct answ er: C)

Explanation
Hepatitis B Serology
• HBsAg detected before HBeAg - both detectable before the onset of jaundice. HBeAg levels fall rapidly
• Anti-HBs antibody detectable at the onset of jaundice and before anti-HBe antibodies
• Anti HBs antibody detected about 4 weeks after the onset of jaundice
• Detection of HBeAg indicates high infectivity
• Detection of anti-HBs IgG indicates previous infection or vaccination
• Chronic carriage associated with persistence of HBsAg
Tests Results Interpretation
Negative
HBsAg
Negative
anti-HBc Susceptible
Negative
IgG anti-HBs

HBsAg Negative
anti-HBc Positive Immune due to natural infection
IgG anti-HBs Positive
HBsAg Negative
anti-HBc Negative Immune due to vaccination
IgG anti-HBs Positive
HBsAg
Positive
anti-HBc
Positive
IgM anti-HBc Acute infection
Positive
IgG anti-HBs
Negative

HBsAg Positive
anti-HBc Positive
Chronic infection
IgM anti-HBc Negative
IgG anti-HBs Negative
Adapted from Centre for Diseases Control & Prevention

A 24 year old woman has been referred to the antenatal clinic at 16 weeks gestation. She is
Question 3
known to be HBs antigen positive and anti-HBs IgG negative
Options for Questions 3-3

This Copy is for Dr. Mohamed ElHodiby


She has previously been vaccinated against Fetal blood sampling should be avoided during
A B
hepatitis B labour
The woman should be vaccinated against
C Caesarean section should be recommended D
hepatitis B after delivery
Hepatitis B viral load should be monitored during
E
pregnancy

A(Correct answ er: B)

Explanation
HbsAg Positive
anti-HBc Positive
IgM anti-HBc Negative
IgG anti-HBs Negative
Chronic carrier.

Aim for vaginal birth but keep membranes intact for as long as possible, avoid FSE and FBS. Neonatal passive
followed by passive and active immunisation

Question 4 Terratogenic virus

Options for Questions 4-4

A Hepatitis B B Herpes simplex


C Varicella zoster D Human papilloma virus
E Parvovirus B 19

A(Correct answ er: C)

Explanation

• Varicella zoster associated with 2% risk of congenital varicella syndrome (dermatomal scarring, cataract, limb
hypoplasia, microcephaly, learning disability..) if infection occurs BEFORE 20 weeks gestation
• Parvovirus B 19 causes fetal anaemia and hydrops

A 17 year old woman attends the antenatal clinic at 20 weeks gestation. She has a history of
Question 5
genital herpes
Options for Questions 5-5

There is an increased risk of congenital


A The virus is an RNA virus B
anomalies
C The virus has an incubation period of 10-21 days D Herpetic lesions may be present on her cervix
In the absence of vesicles or a prodrom, there is
E
no risk to the neonate following vaginal birth

A(Correct answ er: D)

Explanation
HERPES SIMPLEX VIRUS
• DNA virus
• Type I classically causes oro-labial, and type II genital infection .

This Copy is for Dr. Mohamed ElHodiby


• However, the prevalence of oro-genital sexual practices means that this distinction is less evident
• Incubation period 3-7 days
• Primary infection may be asymptomatic. Symptoms include genital vesicles which progress into painful ulcers that
eventually crust over. Dysuria, urinary retention, bilateral inguinal lymphadenopathy, systemic illness may occur.
Aseptic meningitis / encephalitis are rare complications
• Primary infection involves cervix in 70% of symptomatic cases, causing purulent discharge and necrotic ulceration
• Virus remains dormant in dorsal root ganglia, causing recurrent infection. Average number of recurrences per year =
4. Less severe symptoms
• Genital infection can be acquired from oro-genital contact.
• Virus can be shed from the genital tract and transmit infection in the absence of symptoms or a prodrome
• Trauma (such as dental treatment, facial peels), exposure to sunlight or ultra-violet light (sun beds) may trigger
recurrent cold sores
• Vertical transmission causes neonatal disseminated herpes which has high mortality / morbidity. Does not cause
congenital anomalies.
Diagnosis
• Clinical
• Viral culture – negative culture does not exclude infection. The longer the duration of symptoms, the less likely is a
swab to be positive
• Electron microscopy
• ELISA / PCR
• Serology – unreliable cannot distinguish between oro-labial and genital infection
Treatment
• Symptomatic
• Acyclovir
• Prophylactic acyclovir reduces the frequency of recurrence.

A 21 year old woman is thought to have primary genital herpes at 6 weeks gestation. The
Question 6
diagnosis should be confirmed using
Options for Questions 6-6

A Clinical history and examination B Viral culture


C PCR D Serology
E Electron microscopy

A(Correct answ er: A)

Explanation
RPES SIMPLEX VIRUS
• DNA virus
• Type I classically causes oro-labial, and type II genital infection .
• However, the prevalence of oro-genital sexual practices means that this distinction is less evident
• Incubation period 3-7 days
• Primary infection may be asymptomatic. Symptoms include genital vesicles which progress into painful ulcers that
eventually crust over. Dysuria, urinary retention, bilateral inguinal lymphadenopathy, systemic illness may occur.
Aseptic meningitis / encephalitis are rare complications
• Primary infection involves cervix in 70% of symptomatic cases, causing purulent discharge and necrotic ulceration
• Virus remains dormant in dorsal root ganglia, causing recurrent infection. Average number of recurrences per year =
4. Less severe symptoms
• Genital infection can be acquired from oro-genital contact.
• Virus can be shed from the genital tract and transmit infection in the absence of symptoms or a prodrome
• Trauma (such as dental treatment, facial peels), exposure to sunlight or ultra-violet light (sun beds) may trigger
recurrent cold sores
• Vertical transmission causes neonatal disseminated herpes which has high mortality / morbidity. Does not cause
congenital anomalies.
Diagnosis

This Copy is for Dr. Mohamed ElHodiby


• Clinical
• Viral culture – negative culture does not exclude infection. The longer the duration of symptoms, the less likely is a
swab to be positive
• Electron microscopy
• ELISA / PCR
• Serology – unreliable cannot distinguish between oro-labial and genital infection
Treatment
• Symptomatic
• Acyclovir
• Prophylactic acyclovir reduces the frequency of recurrence.

Question 7 Vertical transmission of HIV infection mostly occurs

Options for Questions 7-7

A In the first trimester B In the second trimester


C In the third trimester D During labour and delivery
E In the neonatal period through breastfeeding

A(Correct answ er: D)

Explanation
Vertical transmission of HIV
• The risk of vertical transmission: 15-20% in untreated, non-breast-feeding woman in Europe and 25-40% in untreated
breast-feeding woman in Africa.
• The risk of vertical transmission in women treated with Highly Active Anti-retroviral Therapy (HAART) in Europe is
less than 2%. In women treated with HAART and in whom the viral load is less than 50 copies / ml, the risk of vertical
transmission is less than 1% irrespective of mode of delivery.
The risk factors for vertical transmission in treated women include:
1. High plasma viraemia at delivery
2. Short duration of HAART therapy
3. Delivery before 32 weeks
• Less than 2% of vertical transmission occurs in the first / second trimesters. Over 80% of vertical transmission occurs
around the time of labour and delivery
Maternal factors associated with increased risk of vertical transmission include
1. Advanced maternal HIV disease
2. Low antenatal CD4 count
3. High plasma viral load – this being the strongest predictor
Obstetric risk factors associated with increased risk of vertical transmission include
1. Vaginal delivery
2. Duration of membrane rupture
3. Chorioamnionitis
4. Pre-term delivery
5. Breast-feeding (associated with a 2-fold increase in vertical transmission rate).

In HIV positive women treated with highly active anti-retroviral therapy, which one is not
Question 8
associated with an increased risk of vertical transmission?
Options for Questions 8-8

A High viral load B Birth before 32 weeks gestation


C Birth after 39 weeks gestation D Short duration of anti-retroviral therapy
E Advanced maternal HIV disease

This Copy is for Dr. Mohamed ElHodiby


A(Correct answ er: C)

Explanation
Vertical transmission of HIV
• The risk of vertical transmission: 15-20% in untreated, non-breast-feeding woman in Europe and 25-40% in untreated
breast-feeding woman in Africa.
• The risk of vertical transmission in women treated with Highly Active Anti-retroviral Therapy (HAART) in Europe is
less than 2%. In women treated with HAART and in whom the viral load is less than 50 copies / ml, the risk of vertical
transmission is less than 1% irrespective of mode of delivery.
The risk factors for vertical transmission in treated women include:
1. High plasma viraemia at delivery
2. Short duration of HAART therapy
3. Delivery before 32 weeks
• Less than 2% of vertical transmission occurs in the first / second trimesters. Over 80% of vertical transmission occurs
around the time of labour and delivery
Maternal factors associated with increased risk of vertical transmission include
1. Advanced maternal HIV disease
2. Low antenatal CD4 count
3. High plasma viral load – this being the strongest predictor
Obstetric risk factors associated with increased risk of vertical transmission include
1. Vaginal delivery
2. Duration of membrane rupture
3. Chorioamnionitis
4. Pre-term delivery
5. Breast-feeding (associated with a 2-fold increase in vertical transmission rate).

Question 9 Which drug is administered orally to treat H1N1 influenza infection?

Options for Questions 9-9

A Acyclovir B Oseltamivir
C Zanamivir D Amantadine
E Rimantadine

A(Correct answ er: B)

Explanation
Influenza – Treatment
Symptomatic treatment including paracetamol for fever and as analgesia
Anti-viral agents:
1) Neuraminidase inhibitors: oseltamivir (Tamiflu ®) and zanamivir (Relenza ®)
• Interfere with the release of new viruses from infected cells.
• Most effective if started within a few hours of the onset of symptoms.
• Licensed for use in adults within 48h of the onset of symptoms but may be administered within 7 days of onset.
• Reduce the duration of symptoms by 1-1.5 days and can reduce the risk of complications in high risk patients
Post-exposure prophylaxis: oseltamivir should be given within 48h of exposure while zanamivir should be given within
36h
Drug treatment in pregnancy
• Zanamivir is the preferred drug during pregnancy
• Oseltamivir recommended in severe infection or when zanamivir cannot be used
• Oseltamivir is the preferred drug in women who are breastfeeding
• Oseltamivir: administered orally. Prodrug, hydrolysed by the liver to active metabolite. Extensively metabolised by the
placenta with minimal fetal drug accumulation. Present in low concentration in breast milk
• Use with caution in renal impairment
• Side-effects: Nausea, vomiting, abdominal pain, diarrhoea, headache, conjunctivitis. Less common side-effects
include eczema, hepatitis, GI bleeding, arrhythmia, visual disturbance

This Copy is for Dr. Mohamed ElHodiby


• Zanamivir: administered by inhalation of powder (bioavailability is 10-20% after inhaled dose compared to 2% after
oral dose). 90% of absorbed drug excreted unchanged in the urine. Present in breast milk in low concentration.
• Use with caution in patients with asthma and chronic pulmonary disease because of risk of bronchospasm
• Side-effects: bronchospasm, respiratory impairment, angioedema, rticarial, rash
2) M2 protein inhibitors (amantadine, rimantadine): stop the virus infecting cells. High levels of drug resistance have
been observed and not active against influenza B. No longer recommended.

Question 10 Which drug is administered by inhalation to treat infection with H1N1 influenza?

Options for Questions 10-10

A Acyclovir B Oseltamivir
C Zanamivir D Amantadine
E Rimantadine

A(Correct answ er: C)

Explanation
Influenza – Treatment
Symptomatic treatment including paracetamol for fever and as analgesia
Anti-viral agents:
1) Neuraminidase inhibitors: oseltamivir (Tamiflu ®) and zanamivir (Relenza ®)
• Interfere with the release of new viruses from infected cells.
• Most effective if started within a few hours of the onset of symptoms.
• Licensed for use in adults within 48h of the onset of symptoms but may be administered within 7 days of onset.
• Reduce the duration of symptoms by 1-1.5 days and can reduce the risk of complications in high risk patients
Post-exposure prophylaxis: oseltamivir should be given within 48h of exposure while zanamivir should be given within
36h
Drug treatment in pregnancy
• Zanamivir is the preferred drug during pregnancy
• Oseltamivir recommended in severe infection or when zanamivir cannot be used
• Oseltamivir is the preferred drug in women who are breastfeeding
• Oseltamivir: administered orally. Prodrug, hydrolysed by the liver to active metabolite. Extensively metabolised by the
placenta with minimal fetal drug accumulation. Present in low concentration in breast milk
• Use with caution in renal impairment
• Side-effects: Nausea, vomiting, abdominal pain, diarrhoea, headache, conjunctivitis. Less common side-effects
include eczema, hepatitis, GI bleeding, arrhythmia, visual disturbance
• Zanamivir: administered by inhalation of powder (bioavailability is 10-20% after inhaled dose compared to 2% after
oral dose). 90% of absorbed drug excreted unchanged in the urine. Present in breast milk in low concentration.
• Use with caution in patients with asthma and chronic pulmonary disease because of risk of bronchospasm
• Side-effects: bronchospasm, respiratory impairment, angioedema, rticarial, rash
2) M2 protein inhibitors (amantadine, rimantadine): stop the virus infecting cells. High levels of drug resistance have
been observed and not active against influenza B. No longer recommended.

At which stage of pregnancy are women at greatest risk of severe disease following H1N1
Question 11
influenza infection?
Options for Questions 11-11

A First trimester B Second trimester


C Third trimester D 24-48h post-partum
E 4-6 weeks post-partum

A(Correct answ er: C)

This Copy is for Dr. Mohamed ElHodiby


Explanation
N1 Influenza and pregnancy
• Pregnant women are more likely to have severe disease, be admitted to ITU and die from H1N1 influenza compared
to non-pregnant women. Most reported deaths occurred in women who developed pneumonia and acute respiratory
distress syndrome requiring mechanical ventilation
• Risk of morbidity from seasonal flu is also higher during pregnancy
• The risk is highest in the third trimester compared to early pregnancy or post-partum
• Women with asthma and morbid obesity appear to be at higher risk of severe disease
• Most current data from UKOSS indicates that pregnant women admitted to ITU were less likely to have been treated
with antiviral agents within 2 days of the onset of symptoms when compared with other pregnant women who were
admitted to hospital. This indicates the importance of prompt treatment
• H1N1 infection is associated with pre-term delivery, especially in women admitted to ITU
• Vertical transmission of H1N1 influenza has not been reported and the virus is not known to be present in breast
milk. Breastfeeding should continue in women being treated for H1N1 influenza
• The H1N1 virus is not terratogenic but there some evidence that pyrexia in early pregnancy may be associated with
an increased risk of neural tube defects

Question 12 Infection with the H1N1 influenza virus

Options for Questions 12-12

Occurring in the third trimester is associated with Occurring in the first trimester is associated with
A B
a 10-25% risk of vertical transmission an increased risk of congenital anomalies
Is associated with an increased risk of pre-term Is associated with an increased risk of pre-
C D
delivery eclampsia
E Causes less severe disease in obese women

A(Correct answ er: C)

Explanation
H1N1 Influenza and pregnancy
• Pregnant women are more likely to have severe disease, be admitted to ITU and die from H1N1 influenza compared
to non-pregnant women. Most reported deaths occurred in women who developed pneumonia and acute respiratory
distress syndrome requiring mechanical ventilation
• Risk of morbidity from seasonal flu is also higher during pregnancy
• The risk is highest in the third trimester compared to early pregnancy or post-partum
• Women with asthma and morbid obesity appear to be at higher risk of severe disease
• Most current data from UKOSS indicates that pregnant women admitted to ITU were less likely to have been treated
with antiviral agents within 2 days of the onset of symptoms when compared with other pregnant women who were
admitted to hospital. This indicates the importance of prompt treatment
• H1N1 infection is associated with pre-term delivery, especially in women admitted to ITU
• Vertical transmission of H1N1 influenza has not been reported and the virus is not known to be present in breast
milk. Breastfeeding should continue in women being treated for H1N1 influenza
• The H1N1 virus is not terratogenic but there some evidence that pyrexia in early pregnancy may be associated with
an increased risk of neural tube defects

Question 13 Causes Rheumatic fever

Options for Questions 13-13

A Strep. Viridans B Strep. Pneumonia

This Copy is for Dr. Mohamed ElHodiby


C Strep. Algalactiae D Strep. Fecalis
E Strep. Pyogenes

A(Correct answ er: E)

Explanation
STREPTOCOCCI
• Gram positive
• Capsulated, aerobic
• May form chains (S. viridans) or diplococci (S. pneumoniae)
• Grow on blood agar - may cause haemolysis

Alpha-Haemolytic
• Green colour around each colony due to altered haemoglobin
• S. viridans and S. pneumoniae
• Differentiated by Optochin disc test (S. pneumoniae shows zone of inhibition) and bile solubility test (S.
pneumoniae is soluble)
• S. pneumoniae isolated from nose / throat in 70% of healthy individuals
• S. viridans - isolated from the mouth, causes bacterial endocarditis

Beta-Haemolytic
• Complete lysis of red cells
• S. pyogenes, S. agalactiae, S. fecalis
• Differentiated by Lancefield grouping and Bacitracin test
• Lancefield group A: S. pyogenes- causes sore throat, skin and wound infections (erysipelas, impetigo),
rheumatic fever and acute glomerulonephritis. Strains causing scarlet fever produce an exotoxin
• Lancefield group B: S. algalactiae - isolated from vagina, perineum and rectum; causes neonatal
septicaemia and meningitis
• Lancefield group D: S. fecalis - usually non-haemolytic. Isolated from gut, causes UTIs and bacterial
endocarditis

Gamma Haemolytic
• Non haemolytic
• S. fecalis
Streptococci are characteristically sensitive to penicillin / erythromycin

A healthy 34 year old woman develops a wound infection 5 days after emergency caesarean
Question 14
section. Which organism is the most likely cause?
Options for Questions 14-14

A E. Coli B Staph. Aureus


C Group B Strep D Anaerobes
E Group A Strep

A(Correct answ er: B)

Explanation
STAPHYLOCOCCI
• Gram positive
• Grow on blood agar
• Differentiated by coagulase test
• S. aureus: coagulase positive - produce clot when mixed with serum. Also catalase positive and produces
exotoxin (toxic shock syndrome, toxic epidermal necrolysis). Non-motile
• S. aureus produces yellow - orange pigmented colonies

This Copy is for Dr. Mohamed ElHodiby


• Plasmid mediated antibiotic resistance - can spread between different strains by transducing phages.
Produces beta-lactamase (penicillinase). Sensitive to flucloxacillin
• Methicillin-resistant S. aureus is resistant to flucloxacillin but usually sensitive to vancomycin
• S. aureus diseases include wound, bone and joint infections, breast abscess, skin and food-borne
infections, toxic shock syndrome. Isolated from the nose in 10-30% of healthy individuals
S. epidermidis is coagulase negative

Question 15 Which bacteria typically cause toxic shock syndrome?

Options for Questions 15-15

A Group A streptococci B Group B streptococci


C Coagulase negative staphylococci D Coagulase positive staphylococci
E Actinomycetes Israelii

A(Correct answ er: D)

Explanation
TOXIC SHOCK SYNDROME (TSS)
• Rare, life-threatening inflammatory response syndrome characterized by fever, rash, hypotension,
constitutional symptoms, and multi-organ involvement
• Most often associated with the use of super-absorbent tampons and occasionally with the use of
contraceptive sponges.
• Associated with infection with toxigenic strains of Staphylococcus aureus
• The infection can also affect men, children and postmenopausal women.
• Signs and symptoms develop suddenly
• TSS has been linked to many bacterial infections, including pneumonia, osteomyelitis, sinusitis, and
skin and gynecologic infections
• Endotoxin toxic shock syndrome toxin-1 (TSST-1) is the major toxin produced by strains of S aureus
that are responsible for causing TSS.
• Streptococcus pyogenes exotoxin A (SPEA) and S pyogenes exotoxin B (SPEB) are the major
toxins produced by group A beta-hemolytic streptococci.
The toxins activate production of cytokines, such as tumor necrosis factor, interleukin-1, M protein, and gamma-
interferon and induction of nitric oxide production which contributes to hypotension.

Question 16 Clostridium tetani is sensitive to the following antibiotics

Options for Questions 16-16

A Penicillin and metronidazole B Cefuroxime and metronidazole


C Penicillin and flucloxacillin D Gentamicin and doxycycline
E Rifampicin and trimrthoprim

A(Correct answ er: C)

Explanation
Clostridium Tetani
• Flagellated bacilli. Spores are resistant to boiling and autoclaving is required
• Incubation period 3-21 days
• Produces EXOTOXIN which is non-immunogenic therefore infection does not confer immunity.
Neurotoxin, inhibits release of inhibitory neurotransmitters in the CNS causing tetany / muscle spasms
• There is no local inflammation associated with tissue infection

This Copy is for Dr. Mohamed ElHodiby


• Infection prevented by vaccination with tetanus toxoid in infancy followed by a booster dose every 5-10
years
• Passive immunisation with human tetanus immune globulin + active immunisation required in at-risk
individuals following injury
Antibiotic treatment is an ancillary measure and should include penicillin +/- flucloxacillin. Removal of devitalised
tissue, good tissue perfusion and oxygenation are essential

Question 17 Test results in a pregnant woman with treated syphilis

Options for Questions 17-17

VDRL and fluorescent treponemal antibody tests


A VDRL test is negative B
are negative
VDRL test negative and fluorescent treponemal VDRL test positive and fluorescent treponemal
C D
antibody tests positive antibody tests negative
E Fluorescent treponemal antibody tests negative

A(Correct answ er: C)

Explanation
DIAGNOSIS OF SYPHILIS
Cardiolipin antibody tests(such as Wassermann reaction, rapid plasma regain test and VDRL - Venereal
Disease Reference Laboratory) which are relatively non-specific
Treponemal antibody testssuch as the Fluorescent Treponemal Antibody test -Absorbed (FTA-Abs) which
are specific and used as verification tests
• Cardiolipin antibody tests are positive in the serum in untreated secondary, latent and tertiary syphilis and
may be false positive
• Cardiolipin antibody tests become negative with treatment
• Primary: VDRL (+ve or -ve), FTA-Abs (+ve). FTA-Abs usually becomes positive before VDRL
• Secondary : VDRL and FTA-Abs (+ve)
• Latent : VDRL (usually +ve), FTA-Abs (+ve)
• Tertiary / quaternary : VDRL (+ve or -ve), FTA-Abs (+ve)
• Treated : VDRL (-ve), FTA-Abs (+ve)
• The VDRL test becomes negative with treatment and can be used to monitor treatment. The FTA-Abs test
can remain positive for several years after adequate treatment

Question 18 With respect to serological tests for syphilis

Options for Questions 18-18

The VDRL test becomes positive before the


A B The VDRL test remains positive after treatment
fluorescent treponemal antibody test
The fluorescent treponemal antibody test is used The VDRL test is more specific than the
C D
to monitor treatment treponemal antibody test
The fluorescent treponemal antibody test can
E
remain positive for several years after treatment

A(Correct answ er: E)

Explanation
DIAGNOSIS OF SYPHILIS
• Cardiolipin antibody tests(such as Wassermann reaction, rapid plasma regain test and VDRL - Venereal
Disease Reference Laboratory) which are relatively non-specific

This Copy is for Dr. Mohamed ElHodiby


• Treponemal antibody testssuch as the Fluorescent Treponemal Antibody test -Absorbed (FTA-Abs) which
are specific and used as verification tests
• Cardiolipin antibody tests are positive in the serum in untreated secondary, latent and tertiary syphilis and
may be false positive
• Cardiolipin antibody tests become negative with treatment
• Primary: VDRL (+ve or -ve), FTA-Abs (+ve). FTA-Abs usually becomes positive before VDRL
• Secondary : VDRL and FTA-Abs (+ve)
• Latent : VDRL (usually +ve), FTA-Abs (+ve)
• Tertiary / quaternary : VDRL (+ve or -ve), FTA-Abs (+ve)
• Treated : VDRL (-ve), FTA-Abs (+ve)
• The VDRL test becomes negative with treatment and can be used to monitor treatment. The FTA-Abs test
can remain positive for several years after adequate treatment

Question 19 Which infection is not caused by Chlamydia trachomatis?

Options for Questions 19-19

A Salpingitis B Conjunctivitis
C Bartholinitis D Interstitial pneumonitis
E Pharyngitis

A(Correct answ er: E)

Explanation
AMYDIA TRACHOMATIS
• Obligate intracellular organism - bacteria
• Non-motile gram negative or gram variable although gram stain is not useful in diagnosis
• Enter the host through abrasions in mucosal surfaces
• Does not infect squamous cells - the vaginal epithelium and ectocervix are not infected. Endocervical glands are
susceptible
• Produce infectious particles called Elementary Bodies which enter cells by endocytosis
• Differentiate into Reticulate Bodies within cells. These multiply by binary fission and then produce other elementary
bodies which are released from the cell
• Asymptomatic infection is common
• C. psittaci infects birds and non-human animals
Various serovars:
• A - C: Trachoma (blindness)
• D - K: genital infection - cervicitis, urethritis, salpingitis, epididymitis, proctitis, Bartholinitis, conjunctivitis, interstitial
pneumonitis in neonates.
• Consequences of infection include infertility, ectopic pregnancy, Fitz-Hugh-Curtis syndrome, Reiter's syndrome,
arthritis, dermatitis
• L1-3: Lymphogranuloma venereum

Question 20 Which infection is not caused by Neisseria gonorrhoea?

Options for Questions 20-20

A Proctitis B Vaginitis
C Pharyngitis D Cervicitis
E Urethritis

A(Correct answ er: B)

This Copy is for Dr. Mohamed ElHodiby


Explanation
Neisseria gonorrhoea
• Incubation period 2-7 days, majority of infected females are asymptomatic
• Males: 20% chance; Females: 50% chance of being infected after one episode of unprotected intercourse with
infected partner
• Causes urethritis, cervicitis, epididymitis, pharyngitis, proctitis, endocarditis, skin infection, arthritis and vertical
neonatal infection (ophthalmia neonatorum). Salpingitis (PID) may lead to infertility.

Question 21 The normal range of vaginal pH

Options for Questions 21-21

A 2.0 – 3.5 B 3.8 – 4.5


C 4.8 – 5.7 D 5.0 – 7.0
E 7.2 – 8.0

A(Correct answ er: B)

Explanation
BACTERIAL VAGINOSIS
Polymicrobial infection characterised by:
• Copious frothy vaginal discharge with a fishy odour
• Increased vaginal pH (>4.5). Normal vaginal pH = 3.8 – 4.5
• Clue cells - vaginal epithelial cells coated with gram-variable bacteria on wet vaginal smear
• Vaginosis - there is minimal inflammation
• Positive amine test - pungent fishy odour on adding the vaginal discharge to sodium hydroxide
• Associated with an alteration in vaginal flora with decreased lactobacilli and increased anaerobes and
Gaednerella vaginalis
• Not sexually transmitted
• Treated with metronidazole

Question 22 Vaginal infection characterised by the presence strawberry cervis

Options for Questions 22-22

A Bacterial vaginosis B Gardnerella vaginalis infection


C Trichomonas vaginalis infection D Candida albicans infection
E Chlamydia trachomatis infection

A(Correct answ er: C)

Explanation
TRICHOMONAS VAGINALIS
• Flagellated protozoan, anaerobic, readily cultured on Diamond’s medium
• Sexually transmitted
• Infection is different from bacterial vaginosis although clinical differentiation is difficult
• Characteristic strawberry cervix only found in 3% of cases
• Diagnosed by microscopy of wet vaginal smear or by culture
• The organism is difficult to detect in males but can be isolated from prostatic fluid
• Also causes urethritis, cystitis and Bartholinitis
• Vaginal pH > 4.5
• Treated with metronidazole

This Copy is for Dr. Mohamed ElHodiby


Question 23 Which organism typically causes opportunistic infections?

Options for Questions 23-23

A E. Coli B P. Aeruginosa
C S. Typhi D P. Mirabilis
E N. Meningitides

A(Correct answ er: B)

Explanation
PSEUDOMONAS AERUGINOSA
• Gram negative bacilli, indistinguishable from coliforms on gram staining
• Strict aerobes
• The bacterium is ubiquitous in soil and water, and on surfaces in contact with soil or water. They occur
regularly on the surfaces of plants and occasionally on the surfaces of animals.
• Its metabolism is respiratory and never fermentative
• Almost all strains are motile by means of a single polar flagellum
• The bacterium almost never infects uncompromised tissues, yet there is hardly any tissue that it cannot
infect if the tissue defenses are compromised in some manner
• P. aeruginosa strains produce two types of soluble pigments, the fluorescent pigment pyoverdin and the
blue pigment pyocyanin. The latter is produced abundantly in media of low-iron content and functions in iron
metabolism in the bacterium
• Grow well on selective agar containing the disinfectant cetrimide
• Strongly oxidase positive (coliforms are oxidase negative)
• Metabolises glucose by oxidation
• Not part of normal gut flora
• Causes chronic UTIs, wound infections, chronic osteomyelitis and opportunistic infections
• Resistant to ampicillin, trimethoprim, tetracycline, sulphonamides
• Sensitive to aminoglycosides gentamicin and netilmicin
• Sensitive to anti-pseudomonas penicillins carbenicillin, ticarcillin, piperacillin
Most are sensitive to ceftazidime, ciprofloxacin and imipenam

A 34 year old woman is found to have a pseudomonas aeruginosa urinary tract infection. Which
Question 24
antibiotic is unsuitable for treatment?
Options for Questions 24-24

A Gentamicin B Ciprofloxacin
C Carbenicillin D Ceftazidime
E Ampicillin

A(Correct answ er: E)

Explanation

PSEUDOMONAS AERUGINOSA
• Gram negative bacilli, indistinguishable from coliforms on gram staining
• Strict aerobes
• The bacterium is ubiquitous in soil and water, and on surfaces in contact with soil or water. They occur
regularly on the surfaces of plants and occasionally on the surfaces of animals.
• Its metabolism is respiratory and never fermentative
• Almost all strains are motile by means of a single polar flagellum
• The bacterium almost never infects uncompromised tissues, yet there is hardly any tissue that it cannot
infect if the tissue defenses are compromised in some manner

This Copy is for Dr. Mohamed ElHodiby


• P. aeruginosa strains produce two types of soluble pigments, the fluorescent pigment pyoverdin and the
blue pigment pyocyanin. The latter is produced abundantly in media of low-iron content and functions in iron
metabolism in the bacterium
• Grow well on selective agar containing the disinfectant cetrimide
• Strongly oxidase positive (coliforms are oxidase negative)
• Metabolises glucose by oxidation
• Not part of normal gut flora
• Causes chronic UTIs, wound infections, chronic osteomyelitis and opportunistic infections
• Resistant to ampicillin, trimethoprim, tetracycline, sulphonamides
• Sensitive to aminoglycosides gentamicin and netilmicin
• Sensitive to anti-pseudomonas penicillins carbenicillin, ticarcillin, piperacillin
Most are sensitive to ceftazidime, ciprofloxacin and imipenam

Question 25 Which organism undergoes part of its life cycle in sheep?

Options for Questions 25-25

A Toxoplasma gondii B Bacteriodes anthracis


C Listeria monocytogenes D Plasmodium falciparum
E Treponema pallidum

A(Correct answ er: A)

Explanation
TOXOPLASMA GONDII
• Obligate intracellular protozoan
• Completes part of its life cycle in animals (mainly cats but also sheep, cattle and pigs)
• Contracted by contact with cat litter, infected soil or eating undercooked beef, lamb or pork. Prevented by
wearing gloves during gardening, thorough food hygiene and avoiding contact with cat litter
• Oocysts or tissue cysts invades the intestinal epithelium , develop into trophozoites which spread via the
lymphatic system
• Infection is usually mild or asymptomatic with lymphadenopathy (posterior cervical) being the main
presentation. Causes severe disease with encephalitis and chorioretinitis in the immunocompromised
• Risk of fetal infection increases with increasing gestation age (17% first, 25% second and 65% third
trimester)
• Risk of fetus being affected FALLS with increasing gestation age
• Recognised cause of congenital anomalies with chorioretinitis, intra-cranial calcification, microcephaly,
hydrocephalus. Increased risk of miscarriage with first trimester infection
• Diagnosis can be made by culture in laboratory rodents
• Detection of IgM or a 4 fold rise in IgG titres in samples 3 weeks apart is indicative of acute infection
Sensitive to spiramycin and pyramethamine

A 23 year old woman presents with a 3 months history of cough and weight loss. She is
Question 26
suspected of having pulmonary tuberculosis
Options for Questions 26-26

Sputum culture results are typically available


A B A negative sputum culture excludes tuberculosis
within 3-7 days
Chest X-ray is contra-indicated in the first The Zeihl-Neelsen stain can be used to make the
C D
trimester diagnosis
E A positive tuberculin test confirms the diagnosis

A(Correct answ er: D)

This Copy is for Dr. Mohamed ElHodiby


Explanation
Mycobacterium tuberculosis
• Slender, straight or slightly curved bacillus, non-motile, non-encapsulated and does not form spores
• Acid fast bacillus (AFB)
• Aerobic
• Slow growing- divides every 18-24 hr
• Resistant to drying and chemical disinfectants
• Sensitive to heat (Pasteurization) and UV light

Tuberculosis
• Transmission occurs aerosol / droplet infection
• One organism may be enough to establish an infection
• TB is a chronic disease and an infected person can spread the infective organism, M. tuberculosis, to
several contacts before being diagnosed
• The main site of infection is the lung (>80%)
• Any organ can be infected via lymphatic drainage or haematological spread
• Symptoms include severe weight loss, night sweats, chronic cough, haemoptis
• A person may be infected without developing disease or remain healthy and free of disease for decades.
The infecting organisms remain viable. Disease may be activated by malignancy, immune suppression, old
age or chronic ill-health
• Diagnosis based on:

1) Clinical presentation
2) Radiology -CXR
3)Microbiology - demonstration of M. tuberculosis in a clinical specimen: microscopic (Ziehl-Neelsen or Auramine
O stain for acid-fast bacilli) and / or Culture- may take 2-6 weeks for the isolation of M. tuberculosis
4)Negative culture does not exclude tuberculosis

Question 27 Which antibiotic does not act by inhibiting bacterial cell wall synthesis?

Options for Questions 27-27

A Penicillin B Cefalexine
C Vancomycin D Teicoplanin
E Gentamicin

A(Correct answ er: E)

Explanation
ANTIBIOTICS AND CELL WALL SYNTHESIS
• Beta lactams - inhibit transpeptidation (penicillins, monobactams, carbapenams and ceplalosporins)
• Cycloserine - inhibits reactions incorporating alanine into cell wall precursor within the cytoplasm
• Glycopeptides - vancomycin, teicoplanin - bind to terminal d-alanyl-d-alanine residue, preventing
incorporation into growing peptidoglycan chain
• Bacitracin - prevents dephophorylation of phosphilipid carrier, preventing regeneration of carrier required
for continuous synthesis
GLYCOPEPTIDES
• Vancomycin and teicoplanin
• Inhibit bacterial cell wall synthesis by binding to d-alanyl-d-alanine at the end of a pentapeptide chain,
preventing the incorporation of new subunits
• Large molecules which cannot penetrate gram negative cell wall. Only active against gram positive
bacteria
• Not absorbed from the GI tract. Oral administration used to treat Clostridium difficile diarrhoea
• Potentially ototoxic and nephrotoxic - vancomycin should be administered slowly to prevent 'red-man'
syndrome. Teicoplanin is less toxic and can be given bolus

This Copy is for Dr. Mohamed ElHodiby


Question 28 Which statement regarding aminoglycoside antibiotics is not true?

Options for Questions 28-28

A They are potentially nephrotoxic B They are potentially ototoxic


They should be administered through the
C They cross the blood-brain barrier very well D
intravenous route
E They are effective against gram negative bacteria

A(Correct answ er: C)

Explanation
AMINOGLYCOSIDES
• Gentamicin, tobramycin, netilmicin, streptomycin
• Inhibit bacterial protein synthesis - interfere with the binding of formylmethionyl-transfer RNA to the
ribosomes, preventing the formation of the initiation complex
• Not absorbed from the GI tract. Do not cross the blood-brain barrier
• Not active against streptococci. Active against gram negative bacteria
• Excreted via the kidneys
• Potentially ototoxic and nephrotoxic
• Production of aminoglycoside modifying enzymes is the most important mechanism of acquired bacterial
resistance - usually plasmid mediated

Question 29 Which statement regarding trimethoprim is not true?

Options for Questions 29-29

A It is a folic acid analogue B It inhibits dihydrofolate reductase


It is mainly used to treat infections with gram
C D It is well absorbed from the GI tract
positive bacteria
E It is predominantly excreted by the kidneys

A(Correct answ er: C)

Explanation
TRIMETHOPRIM
• Folic acid analogue
• Inhibits dihydrofolate reductase
• Similar structure and mechanism of action to methotrexate and the anti-malarial agent pyrimethamine
• Absorbed from the gut
• Excreted by the kidneys
• Effective against gram negative bacilli EXCEPT pseudomonas
• Resistance is mediated by a plasmid-encoded dihydrofolate reductase which has a much reduced affinity for
trimethoprim

Question 30 Which disinfectant may be used to prepare the vagina prior to vaginal hysterectomy?

Options for Questions 30-30

A Iodine (3%) solution B Povidone-iodine solution


C Isopropyl alcohol 70% D Chlorhexidine 2% + alcohol 70%
E Chlorhexidine 0.2%

This Copy is for Dr. Mohamed ElHodiby


A(Correct answ er: B)

Explanation
Iodophores (7.5 – 10%)
• Preparation containing iodine complexed with a solubilizing agent, such as a surfactant or povidone (povidone-iodine)
• Result in a solution that releases free iodine
• Excellent activity against gram positive and most gram negative bacteria
• Fair activity against TB
• No activity against endospores
• Good activity against viruses and fungi
• Activity is moderately affected by organic matter
• Useful as surgical scrub and skin prep
• Can be used on mucous membranes
• Iodine may be absorbed by the neonate and affect thyroid function

This Copy is for Dr. Mohamed ElHodiby


Question 1 Streptococci

Options for Questions 1-1

A Are differentiated using the coagulase test B Produce beta lactamase


C Are typically resistant to penicillin D Typically cause post-operative wound infection
E Grow on blood agar causing haemolysis

A(Correct answ er: E)

Explanation
STREPTOCOCCI
• Gram positive
• Capsulated, aerobic
• May form chains (S. viridans) or diplococci (S. pneumoniae)
• Grow on blood agar - may cause haemolysis

Alpha-Haemolytic
• Green colour around each colony due to altered haemoglobin
• S. viridans and S. pneumoniae
• Differentiated by Optochin disc test (S. pneumoniae shows zone of inhibition) and bile solubility test (S.
pneumoniae is soluble)
• S. pneumoniae isolated from nose / throat in 70% of healthy individuals
• S. viridans - isolated from the mouth, causes bacterial endocarditis

Beta-Haemolytic
• Complete lysis of red cells
• S. pyogenes, S. agalactiae, S. fecalis
• Differentiated by Lancefield grouping and Bacitracin test
• Lancefield group A: S. pyogenes- causes sore throat, skin and wound infections (erysipelas, impetigo),
rheumatic fever and acute glomerulonephritis. Strains causing scarlet fever produce an exotoxin
• Lancefield group B: S. algalactiae - isolated from vagina, perineum and rectum; causes neonatal
septicaemia and meningitis
• Lancefield group D: S. fecalis - usually non-haemolytic. Isolated from gut, causes UTIs and bacterial
endocarditis

Gamma Haemolytic
• Non haemolytic
• S. fecalis
• Streptococci are characteristically sensitive to penicillin / erythromycin

Question 2 Causes sore throat, wound and skin infections

Question 3 Causes Rheumatic fever

Options for Questions 2-3

A Strep. Viridans B Strep. Pneumonia


C Strep. Algalactiae D Strep. Fecalis
E Strep. Pyogenes

This Copy is for Dr. Mohamed ElHodiby


A(Correct answ er: E)

A(Correct answ er: E)

Explanation
STREPTOCOCCI
• Gram positive
• Capsulated, aerobic
• May form chains (S. viridans) or diplococci (S. pneumoniae)
• Grow on blood agar - may cause haemolysis

Alpha-Haemolytic
• Green colour around each colony due to altered haemoglobin
• S. viridans and S. pneumoniae
• Differentiated by Optochin disc test (S. pneumoniae shows zone of inhibition) and bile solubility test (S.
pneumoniae is soluble)
• S. pneumoniae isolated from nose / throat in 70% of healthy individuals
• S. viridans - isolated from the mouth, causes bacterial endocarditis

Beta-Haemolytic
• Complete lysis of red cells
• S. pyogenes, S. agalactiae, S. fecalis
• Differentiated by Lancefield grouping and Bacitracin test
• Lancefield group A: S. pyogenes- causes sore throat, skin and wound infections (erysipelas, impetigo),
rheumatic fever and acute glomerulonephritis. Strains causing scarlet fever produce an exotoxin
• Lancefield group B: S. algalactiae - isolated from vagina, perineum and rectum; causes neonatal
septicaemia and meningitis
• Lancefield group D: S. fecalis - usually non-haemolytic. Isolated from gut, causes UTIs and bacterial
endocarditis

Gamma Haemolytic
• Non haemolytic
• S. fecalis
Streptococci are characteristically sensitive to penicillin / erythromycin

A healthy 34 year old woman develops a wound infection 5 days after emergency caesarean
Question 4
section. Which is the most appropriate antibiotic therapy?

Options for Questions 4-4

A Amoxicillin B Cephalexin
C Flucloxacillin D Metronidazole + cefalexine
E Gentamicin

A(Correct answ er: C)

Explanation
STAPHYLOCOCCI
• Gram positive
• Grow on blood agar
• Differentiated by coagulase test
• S. aureus: coagulase positive - produce clot when mixed with serum. Also catalase positive and produces
exotoxin (toxic shock syndrome, toxic epidermal necrolysis). Non-motile
• S. aureus produces yellow - orange pigmented colonies

This Copy is for Dr. Mohamed ElHodiby


• Plasmid mediated antibiotic resistance - can spread between different strains by transducing phages.
Produces beta-lactamase (penicillinase). Sensitive to flucloxacillin
• Methicillin-resistant S. aureus is resistant to flucloxacillin but usually sensitive to vancomycin
• S. aureus diseases include wound, bone and joint infections, breast abscess, skin and food-borne
infections, toxic shock syndrome. Isolated from the nose in 10-30% of healthy individuals
S. epidermidis is coagulase negative

Question 5 Which one is a spore-forming bacterium?

Options for Questions 5-5

A Actinomyces israelii B Corynebacterium diphtheria


C Listeria monocytogenes D Pseudomonas aeruginosa
E Bacillus anthracis

A(Correct answ er: E)

Explanation
BACILLUS SPECIES
• B. anthracis - gram positive aerobic spore-forming bacilli; capsulated
• Grows on blood agar
• Spores may remain dormant in the environment for several years
• Infects many animals including sheep
• Causes anthrax
• Produces an exotoxin
• B. cereus contaminates food such as boiled rice, causing food poisoning
Sensitive to penicillin

Question 6 A spore-forming organism that infects sheep and causes sepsis in iv drug users

Options for Questions 6-6

A Actinomyces israelii B Clostridium perfringens


C Listeria monocytogenes D Pseudomonas aeruginosa
E Bacillus anthracis

A(Correct answ er: E)

Explanation
BACILLUS SPECIES
• B. anthracis - gram positive aerobic spore-forming bacilli; capsulated
• Grows on blood agar
• Spores may remain dormant in the environment for several years
• Infects many animals including sheep
• Causes anthrax
• Produces an exotoxin
• B. cereus contaminates food such as boiled rice, causing food poisoning
Sensitive to penicillin

Question 7 Which organism is typically sensitive to penicillin?

Options for Questions 7-7

A E Coli B Staph. Aureus

This Copy is for Dr. Mohamed ElHodiby


C Bacillus anthracis D C. Difficile
E Staph. Epidermides

A(Correct answ er: C)

Explanation
BACILLUS SPECIES
• B. anthracis - gram positive aerobic spore-forming bacilli; capsulated
• Grows on blood agar
• Spores may remain dormant in the environment for several years
• Infects many animals including sheep
• Causes anthrax
• Produces an exotoxin
• B. cereus contaminates food such as boiled rice, causing food poisoning
Sensitive to penicillin

Question 8 Actinomycete israelii

Options for Questions 8-8

A Is gram negative B Is acid-fast


C Infection is characterized by sulphur granules D Infection typically involves the lymphatic system
E Infection is characterized by gas formation

A(Correct answ er: C)

Explanation
ACTINOMYCETE ISRAELII
• Gram positive branching filamentous bacillus
• Anaerobic
• Form part of the normal flora in the mouth, vagina and rectum.
• Non-acid -fast (Nocardia are weakly acid-fast)
• Infection is characterised by sulphur (yellow) granules in pus
• Does not involve the lymphatics
• Causes cervico-facial, pulmonary and abdominal infection, causing multiple discharging sinuses
• Genital tract infection associated with the IUCD
• Sensitive to penicillin

Question 9 Which organism typically causes IUCD-associated genital tract infections?

Options for Questions 9-9

A Group B streptococcus B Group A streptococcus


C Staphylococcus aureus D Clostridium perfringens
E Actinomycetes israelii

A(Correct answ er: E)

Explanation
ACTINOMYCETE ISRAELII
• Gram positive branching filamentous bacillus

This Copy is for Dr. Mohamed ElHodiby


• Anaerobic
• Form part of the normal flora in the mouth, vagina and rectum.
• Non-acid -fast (Nocardia are weakly acid-fast)
• Infection is characterised by sulphur (yellow) granules in pus
• Does not involve the lymphatics
• Causes cervico-facial, pulmonary and abdominal infection, causing multiple discharging sinuses
• Genital tract infection associated with the IUCD
• Sensitive to penicillin

Question 10 Which one is a spore-forming bacterium?

Options for Questions 10-10

A Clostridium difficile B Actinomyces israelii


C Corynebacterium diphtheria D Listeria monocytogenes
E Pseudomonas aeruginosa

A(Correct answ er: A)

Explanation
CLOSTRIDIA
• Gram positive anaerobic bacilli
• Spore-forming -the spores are resistant to boiling and chemical disinfectants
• Commonly found in the environment and as part of normal fecal flora
• Produce EXOTOXINS
• Infection common in gangrenous / ischaemic tissue and in association with foreign bodies
• Sensitive to metronidazole and penicillin
• CLOSTRIDIUN DIFFICILE - causes pseudo-membranous colitis - diarrhoea with collapse following broad
spectrum antibiotic treatment - treated with oral metronidazole or vancomycin
• CLOSTRIDIUM PERFRINGENS - Causes gas gangrene (causing crepitus) and septic abortion (rare)
• Produces exotoxin causing septic shock, jaundice, haemolysis and acute renal failure
Sensitive to penicillin - surgical debridement usually required

Question 11 Which antibiotics should be used to treat clostridium difficile infection?

Options for Questions 11-11

A Metronidazole or vancomycin B Cefuroxime or ampicillin


C Mentonidazole or gentamicin D Vancomycin or tetracycline
E Vancomycin or cefuroxime

A(Correct answ er: A)

Explanation
CLOSTRIDIA
• Gram positive anaerobic bacilli
• Spore-forming -the spores are resistant to boiling and chemical disinfectants
• Commonly found in the environment and as part of normal fecal flora
• Produce EXOTOXINS
• Infection common in gangrenous / ischaemic tissue and in association with foreign bodies
• Sensitive to metronidazole and penicillin

This Copy is for Dr. Mohamed ElHodiby


• CLOSTRIDIUN DIFFICILE - causes pseudo-membranous colitis - diarrhoea with collapse following broad
spectrum antibiotic treatment - treated with oral metronidazole or vancomycin
• CLOSTRIDIUM PERFRINGENS - Causes gas gangrene (causing crepitus) and septic abortion (rare)
• Produces exotoxin causing septic shock, jaundice, haemolysis and acute renal failure
Sensitive to penicillin - surgical debridement usually required

Question 12 Infection with listeria monocytogenes

Options for Questions 12-12

A Is best treated with cephalosporins B Is sensitive to metronidazole


C Is resistant to ampicillin D Is associated with the formation of granulomas
E Is more common in the first trimester

A(Correct answ er: D)

Explanation
LISTERIA MONOCYTOGENES
• Gram positive coccobaccilus
• Motile. Ubiquitous in the environment but is not a gut commensal
• Wide-spread among animals and in the environment
• Facultative anaerobe, non-spore-forming
• Beta-haemolytic
• Produces exotoxin (haemolysin) which damages leukoytes
• May grow within macrophages
• Multiplies slowly at refrigeration temperatures
• Present in undercooked meats, soft cheeses, pate, coleslaw, contaminated milk. Can be contracted
from animal manure, silage and abortus material
• Produces disease in circumstances associated with depressed T-cell immunity - malignancy,
steroids, AIDS, pregnancy
• Disease more common in third trimester of pregnancy. Infection in first trimester associated with
miscarriage. Third trimester infection associated with fetal death, meconium stained liquor, chorio-amnionitis
• Crosses placenta
• Causes early (vertically acquired) and late (horizontally acquired) onset neonatal disease
• May be isolated from vagina in infected women
• Listeriosis involves many organs with micro-abscesses and granulomas
• Sensitive to ampicillin - main therapy
• Also sensitive to trimethoprim, gentamicin, terracycline, rifampicin and chloramphenicol. Resistant to
cephalosporins
In non-pregnant, typically causes meningitis

Infection with which micro-organism is associated with eating undercooked meats or soft
Question 13
cheeses?
Options for Questions 13-13

A E. Coli B Strep. Fecalis


C L. Monocytogenes D C. Dificile
E M. Tuberculosis

A(Correct answ er: C)

Explanation
LISTERIA MONOCYTOGENES
• Gram positive coccobaccilus
• Motile. Ubiquitous in the environment but is not a gut commensal

This Copy is for Dr. Mohamed ElHodiby


• Wide-spread among animals and in the environment
• Facultative anaerobe, non-spore-forming
• Beta-haemolytic
• Produces exotoxin (haemolysin) which damages leukoytes
• May grow within macrophages
• Multiplies slowly at refrigeration temperatures
• Present in undercooked meats, soft cheeses, pate, coleslaw, contaminated milk. Can be contracted
from animal manure, silage and abortus material
• Produces disease in circumstances associated with depressed T-cell immunity - malignancy,
steroids, AIDS, pregnancy
• Disease more common in third trimester of pregnancy. Infection in first trimester associated with
miscarriage. Third trimester infection associated with fetal death, meconium stained liquor, chorio-amnionitis
• Crosses placenta
• Causes early (vertically acquired) and late (horizontally acquired) onset neonatal disease
• May be isolated from vagina in infected women
• Listeriosis involves many organs with micro-abscesses and granulomas
• Sensitive to ampicillin - main therapy
• Also sensitive to trimethoprim, gentamicin, terracycline, rifampicin and chloramphenicol. Resistant to
cephalosporins
In non-pregnant, typically causes meningitis

Question 14 Exotoxins

Options for Questions 14-14

A Are lipopolysaccharide molecules B Act at specific target sites


C Are heat-stable D Are typically produced by gram negative bacteria
Are typically released when the bacterial cell wall
E
disintegrates

A(Correct answ er: B)

Explanation
TOXINS
• Highly toxic polypeptides produced by living bacteria
• Act at specific target sites
• Are highly antigenic (except Cl. Tetani toxin)
• Are converted into antigenic non-toxic toxoids by formalin
• Are often destroyed by heat
• Are neutralised by anti-toxins
• Typically produced by gram positive bacteria

ENDOTOXINS
• Are lipopolysaccharide molecules
• Present in the outer layer of the cell wall of gram negative bacteria, released on disintegration
• Relatively heat stable
• Not converted into toxoid by formalin
• Act non-specifically on the reticulo-endothelial system causing increased vascular permeability and
prostaglandin release
• May cause DIC and cardiovascular collapse
• Detectable by the Limulus test (A lysate of blood cells from the horseshoe crab is used to test specimens
for the presence of Escherichia coli endotoxin in blood or milk)
• Do not stimulate production of natural anti-toxins
Produced by E. coli, N. meningitides, S. typhi, P. aeruginosa

This Copy is for Dr. Mohamed ElHodiby


Question 15 Which statement regarding coliforms is not true?

Options for Questions 15-15

A Typically produce endotoxins B Are oxidase positive


C Grow on blood agar D Typically sensitive to gentamicin
E May be flagellated

A(Correct answ er: B)

Explanation
COLIFORMS
• Include E. coli, Klebsiella, Proteus, Salmonella and Shigella
• Gram negative bacill
• Facultative anaerobes
• Grow on blood agar or selective media
• Oxidase negative (differentiates from P. aeruginosa which is oxidase positive)
• E. coli and Klebsiella are lactose fermenting
• E. coli and Salmonella are motile (flagellated)
• Produce endotoxins (gram positive bacteria produce exotoxins) but E. coli also produces an enterotoxin
(exotoxin) which causes diarrhoea by stimulating epithelial adenylate cyclase
• E. coli is part of normal fecal flora
• Salmonella and shigella are not part of normal flora
• Salmonella typhi may be present permanently in the biliary tract of carriers
• Klebsiella is resistant to ampicillin. Other community acquired colifirms are generally sensitive to ampicillin
Hospital acquired coloforms are generally resistant to ampicillin but sensitive to gentamicin and third generation
cephalosporins

Question 16 Which organism typically causes opportunistic infections?

Options for Questions 16-16

A E. Coli B P. Aeruginosa
C S. Typhi D P. Mirabilis
E N. Meningitides

A(Correct answ er: B)

Explanation

PSEUDOMONAS AERUGINOSA
• Gram negative bacilli, indistinguishable from coliforms on gram staining
• Strict aerobes
• The bacterium is ubiquitous in soil and water, and on surfaces in contact with soil or water. They occur
regularly on the surfaces of plants and occasionally on the surfaces of animals.
• Its metabolism is respiratory and never fermentative
• Almost all strains are motile by means of a single polar flagellum
• The bacterium almost never infects uncompromised tissues, yet there is hardly any tissue that it cannot
infect if the tissue defenses are compromised in some manner
• P. aeruginosa strains produce two types of soluble pigments, the fluorescent pigment pyoverdin and the
blue pigment pyocyanin. The latter is produced abundantly in media of low-iron content and functions in iron
metabolism in the bacterium
• Grow well on selective agar containing the disinfectant cetrimide
• Strongly oxidase positive (coliforms are oxidase negative)

This Copy is for Dr. Mohamed ElHodiby


• Metabolises glucose by oxidation
• Not part of normal gut flora
• Causes chronic UTIs, wound infections, chronic osteomyelitis and opportunistic infections
• Resistant to ampicillin, trimethoprim, tetracycline, sulphonamides
• Sensitive to aminoglycosides gentamicin and netilmicin
• Sensitive to anti-pseudomonas penicillins carbenicillin, ticarcillin, piperacillin
Most are sensitive to ceftazidime, ciprofloxacin and imipenam

Question 17 Bordetella pertussis

Options for Questions 17-17

A Is a gram positive organism B Infection has an incubation period of 2-4 weeks


C Infection is more common in girls than in boys D Produces an exotoxin
E Infection is typically treated with anti-toxin

A(Correct answ er: C)

Explanation
BORDETELLA PERTUSSIS
• Gram negative bacilli; member of family of Parvobacteria which includes H. influenzae
• Non-motile
• Does not grow on blood agar
• Causes whooping cough
• Incubation period 5-14 days
• Spread by droplet infection
• Infected individual is infectious for up to three weeks after the onset of symptoms
• Infection may occur at any age but most common and more severe in infants below the age of 1 year
• In children, lymphocytosis is often profound (>70% of the total WBC count). The WBC count often rises to
2 2
20-40,000 cells/mm ; and may rise to 100,000 cells/mm
• Infection is more common in females than in males
• Disease has Catarrhal, spasmodic (paroxysmal) and recovery stages
• Cultures are typically negative soon after the onset of the paroxysmal stage
• Infection is preventable by vaccination using killed organism. Does not produce a toxin
Severity of infection may be reduced by erythromycin

Question 18 Which antibiotic(s) may be used to treat infection with Bacteriodes fragilis?
Options for Questions 18-18

A Metronidazole B Metronidazole or clindamycin


C Gentamicin D Cefalexin or gentamicin
E Metronidazole or cefalexine

A(Correct answ er: B)

Explanation
BACTERIODES
• Gram negative bacilli, strict ANAEROBES
• Non-spore forming
• Readily killed by heat or chemical disinfectants
• Do not produce exotoxins

This Copy is for Dr. Mohamed ElHodiby


• Part of normal mouth, vaginal and rectal flora
• Some are motile while others are non-motile
• Important cause of post-operative infection, intra-abdominal and pelvic sepsis. Synergistic action with
aerobic organisms such as coliforms, coliforms being responsible for acute infection, septicaemia and shock
while bacteriodes contribute to abscess formation
Sensitive to metronidazole and clindamycin

Question 19 Which organism undergoes part of its life cycle in sheep?

Options for Questions 19-19

A Toxoplasma gondii B Bacteriodes anthracis


C Listeria monocytogenes D Plasmodium falciparum
E Treponema pallidum

A(Correct answ er: A)

Explanation
TOXOPLASMA GONDII
• Obligate intracellular protozoan
• Completes part of its life cycle in animals (mainly cats but also sheep, cattle and pigs)
• Contracted by contact with cat litter, infected soil or eating undercooked beef, lamb or pork. Prevented by
wearing gloves during gardening, thorough food hygiene and avoiding contact with cat litter
• Oocysts or tissue cysts invades the intestinal epithelium , develop into trophozoites which spread via the
lymphatic system
• Infection is usually mild or asymptomatic with lymphadenopathy (posterior cervical) being the main
presentation. Causes severe disease with encephalitis and chorioretinitis in the immunocompromised
• Risk of fetal infection increases with increasing gestation age (17% first, 25% second and 65% third
trimester)
• Risk of fetus being affected FALLS with increasing gestation age
• Recognised cause of congenital anomalies with chorioretinitis, intra-cranial calcification, microcephaly,
hydrocephalus. Increased risk of miscarriage with first trimester infection
• Diagnosis can be made by culture in laboratory rodents
• Detection of IgM or a 4 fold rise in IgG titres in samples 3 weeks apart is indicative of acute infection
Sensitive to spiramycin and pyramethamine

Question 20 Which drug is not used to treat tuberculosis?

Options for Questions 20-20

A Rifampicin B Isoniazid
C Pyrazinamide D Ethambutol
E Spiramycin

A(Correct answ er: E)

Explanation
Treatment of tuberculosis
• Treatment is for 6 months
• Combinations of at least 3 anti-tuberculous agents are used
First-line treatment: Isoniazid, Rifampicin and Pyrazinamide +/-ethambutol

This Copy is for Dr. Mohamed ElHodiby


A 17 year old woman attends the antenatal clinic at 20 weeks gestation. She has a history of
Question 1
genital herpes
Options for Questions 1-1

There is an increased risk of congenital


A The virus is an RNA virus B
anomalies
C The virus has an incubation period of 10-21 days D Herpetic lesions may be present on her cervix
In the absence of vesicles or a prodrom, there is
E
no risk to the neonate following vaginal birth

A(Correct answ er: D)

Explanation
HERPES SIMPLEX VIRUS
• DNA virus
• Type I classically causes oro-labial, and type II genital infection .
• However, the prevalence of oro-genital sexual practices means that this distinction is less evident
• Incubation period 3-7 days
• Primary infection may be asymptomatic. Symptoms include genital vesicles which progress into painful ulcers that
eventually crust over. Dysuria, urinary retention, bilateral inguinal lymphadenopathy, systemic illness may occur.
Aseptic meningitis / encephalitis are rare complications
• Primary infection involves cervix in 70% of symptomatic cases, causing purulent discharge and necrotic ulceration
• Virus remains dormant in dorsal root ganglia, causing recurrent infection. Average number of recurrences per year =
4. Less severe symptoms
• Genital infection can be acquired from oro-genital contact.
• Virus can be shed from the genital tract and transmit infection in the absence of symptoms or a prodrome
• Trauma (such as dental treatment, facial peels), exposure to sunlight or ultra-violet light (sun beds) may trigger
recurrent cold sores
• Vertical transmission causes neonatal disseminated herpes which has high mortality / morbidity. Does not cause
congenital anomalies.
Diagnosis
• Clinical
• Viral culture – negative culture does not exclude infection. The longer the duration of symptoms, the less likely is a
swab to be positive
• Electron microscopy
• ELISA / PCR
• Serology – unreliable cannot distinguish between oro-labial and genital infection
Treatment
• Symptomatic
• Acyclovir
• Prophylactic acyclovir reduces the frequency of recurrence.

A healthy 32 year old woman phones the maternity assessment unit because her 5 year old
Question 2
son has developed chickenpox and the woman is 16 weeks pregnant
Options for Questions 2-2

She should be reassured if she has had


A She should be reassured B
chickenpox in the past
She should be reassured and tested for immunity
C D She should be treated with immune globulin
if she has had chickenpox in the past
E She should be treated with acyclovir

This Copy is for Dr. Mohamed ElHodiby


A(Correct answ er: C)

Explanation
VARICELLA ZOSTER
• DNA virus, member of the Herpes family of viruses
• Incubation period 10-21 days
• Spread by droplet infection
• Contagious 48h before onset of rash until all vesicles have crusted over
• Severs infection may cause pneumonitis, encephalitis, myocarditis and adrenal insufficiency
• Virus remains dormant in the dorsal root ganglia, causing recurrent infection (shingles, Herpes zoster). Shingles is
contagious
• Infection in pregnancy is more severe, with 10% risk of pneumonitis and up to 6% mortality
• 2% risk of congenital varicella syndrome (dermatomal scarring, cataract, limb hypoplasia, microcephaly, learning
disability..) if infection occurs BEFORE 20 weeks gestation. No risk of congenital anomalies after 28 weeks
• Infection results in life-long immunity.
• There is no effective vaccine currently available
• Infection can be prevented by passive immunisation with varicella zoster immune globulin – not life-long
• 85% of pregnant women are immune (UK)
• Severe neonatal disease likely (30%) if maternal disease develops within 4 days of delivery – neonate has not
acquired maternal antibodies
• Sensitive to acyclovir – useful in severe disease

Question 3 Human papilloma virus

Options for Questions 3-3

A Is a DNA virus B Is terratogenic


Primary infection at term is an indication for
C Infection in pregnancy is treated with ganciclovir D
caesarean section
E Infection always results in a chronic carrier state

A(Correct answ er: A)

Explanation
HUMAN PAPILLOMA VIRUS
• DNA virus
• Over 80 different genotypes
• Cannot be cultured in-vitro
• Commonest sexually transmitted infection
• Types 2,6,11 cause exophytic epithelial proliferation (genital warts; condylomata accuminata)
• Types 16 and 18 are associated with CIN and cervical cancer – high risk HPV. Also associated with oral and vulval
carcinoma and VIN
• Some women are able to clear the infection – dependent on host immunity
• Vertically acquired infection can occur, causing juvenile laryngeal papilomatosis
• There is no recognised anti-viral treatment

A doctor has suffered a needle-stick injury during caesarean section and the patient is known to
Question 4
be HIV positive. The risk of the doctor becoming infected is
Options for Questions 4-4

This Copy is for Dr. Mohamed ElHodiby


A < 1% B 2-5%
C 5-10% D 10-15%
E 15-20%

A(Correct answ er: A)

Explanation
HUMAN IMMUNO-DEFICIENCY VIRUS
• RNA virus
• Infects CD4 positive cells – uses CD4 antigen as receptor
• Sero-conversion may take up to three months
• Transmitted by breastfeeding, sexual intercourse, needle stick injuries (including sharing needles), blood and blood
products and vertival transmission mainly during passage through the genital tract
• Risk of infection after a needle stick injury involving an infected patient is <1% and lower than risk of hepatitis B
transmission
• Risk of infection after single episode of unprotected intercourse with infected partner <1%

Question 5 Vertical transmission of HIV infection mostly occurs

Options for Questions 5-5

A In the first trimester B In the second trimester


C In the third trimester D During labour and delivery
E In the neonatal period through breastfeeding

A(Correct answ er: D)

Explanation
Vertical transmission of HIV
• The risk of vertical transmission: 15-20% in untreated, non-breast-feeding woman in Europe and 25-40% in untreated
breast-feeding woman in Africa.
• The risk of vertical transmission in women treated with Highly Active Anti-retroviral Therapy (HAART) in Europe is
less than 2%. In women treated with HAART and in whom the viral load is less than 50 copies / ml, the risk of vertical
transmission is less than 1% irrespective of mode of delivery.
The risk factors for vertical transmission in treated women include:
1. High plasma viraemia at delivery
2. Short duration of HAART therapy
3. Delivery before 32 weeks
• Less than 2% of vertical transmission occurs in the first / second trimesters. Over 80% of vertical transmission occurs
around the time of labour and delivery
Maternal factors associated with increased risk of vertical transmission include
1. Advanced maternal HIV disease
2. Low antenatal CD4 count
3. High plasma viral load – this being the strongest predictor
Obstetric risk factors associated with increased risk of vertical transmission include
1. Vaginal delivery
2. Duration of membrane rupture
3. Chorioamnionitis
4. Pre-term delivery
5. Breast-feeding (associated with a 2-fold increase in vertical transmission rate).

In HIV positive women treated with highly active anti-retroviral therapy, which one is not
Question 6
associated with an increased risk of vertical transmission?
Options for Questions 6-6

This Copy is for Dr. Mohamed ElHodiby


A High viral load B Birth before 32 weeks gestation
C Birth after 39 weeks gestation D Short duration of anti-retroviral therapy
E Advanced maternal HIV disease

A(Correct answ er: C)

Explanation
Vertical transmission of HIV
• The risk of vertical transmission: 15-20% in untreated, non-breast-feeding woman in Europe and 25-40% in untreated
breast-feeding woman in Africa.
• The risk of vertical transmission in women treated with Highly Active Anti-retroviral Therapy (HAART) in Europe is
less than 2%. In women treated with HAART and in whom the viral load is less than 50 copies / ml, the risk of vertical
transmission is less than 1% irrespective of mode of delivery.
The risk factors for vertical transmission in treated women include:
1. High plasma viraemia at delivery
2. Short duration of HAART therapy
3. Delivery before 32 weeks
• Less than 2% of vertical transmission occurs in the first / second trimesters. Over 80% of vertical transmission occurs
around the time of labour and delivery
Maternal factors associated with increased risk of vertical transmission include
1. Advanced maternal HIV disease
2. Low antenatal CD4 count
3. High plasma viral load – this being the strongest predictor
Obstetric risk factors associated with increased risk of vertical transmission include
1. Vaginal delivery
2. Duration of membrane rupture
3. Chorioamnionitis
4. Pre-term delivery
5. Breast-feeding (associated with a 2-fold increase in vertical transmission rate).

Question 7 The H1N1 virus

Options for Questions 7-7

A Is an influenza B virus B Is transmitted from pigs to humans


C Undergoes antigenic shift D Does not undergo antigenic drift
E Infection does not result in antibody production

A(Correct answ er: C)

Explanation
Classification of influenza viruses
Three main genera: Influenza A, B & C
Influenza A
• Most pathogenic influenza virus in humans
• Undergoes antigenic shift and antigenic drift, resulting in the production of new variants that evade host immunity with
the potential for pandemics
• Divided into several serotypes based on the antibody binding to the Hemagglutinin and Neuraminidase proteins:
1. H1N1 – Caused 2009 pandemic (Swine flu virus). This virus is not endemic in pigs and is not transmitted from pigs to
humans. It is a recombination of several strains of H1N1 virus found in humans, birds and pigs
2. H2N2 and H3N2 have caused pandemics in Asia
3. H5N1 – Causes highly lethal pneumonia

Question 8 Which drug is administered orally to treat H1N1 influenza infection?

This Copy is for Dr. Mohamed ElHodiby


Options for Questions 8-8

A Acyclovir B Oseltamivir
C Zanamivir D Amantadine
E Rimantadine

A(Correct answ er: B)

Explanation
Influenza – Treatment
Symptomatic treatment including paracetamol for fever and as analgesia
Anti-viral agents:
1) Neuraminidase inhibitors: oseltamivir (Tamiflu ®) and zanamivir (Relenza ®)
• Interfere with the release of new viruses from infected cells.
• Most effective if started within a few hours of the onset of symptoms.
• Licensed for use in adults within 48h of the onset of symptoms but may be administered within 7 days of onset.
• Reduce the duration of symptoms by 1-1.5 days and can reduce the risk of complications in high risk patients
Post-exposure prophylaxis: oseltamivir should be given within 48h of exposure while zanamivir should be given within
36h
Drug treatment in pregnancy
• Zanamivir is the preferred drug during pregnancy
• Oseltamivir recommended in severe infection or when zanamivir cannot be used
• Oseltamivir is the preferred drug in women who are breastfeeding
• Oseltamivir: administered orally. Prodrug, hydrolysed by the liver to active metabolite. Extensively metabolised by the
placenta with minimal fetal drug accumulation. Present in low concentration in breast milk
• Use with caution in renal impairment
• Side-effects: Nausea, vomiting, abdominal pain, diarrhoea, headache, conjunctivitis. Less common side-effects
include eczema, hepatitis, GI bleeding, arrhythmia, visual disturbance
• Zanamivir: administered by inhalation of powder (bioavailability is 10-20% after inhaled dose compared to 2% after
oral dose). 90% of absorbed drug excreted unchanged in the urine. Present in breast milk in low concentration.
• Use with caution in patients with asthma and chronic pulmonary disease because of risk of bronchospasm
• Side-effects: bronchospasm, respiratory impairment, angioedema, rticarial, rash
2) M2 protein inhibitors (amantadine, rimantadine): stop the virus infecting cells. High levels of drug resistance have
been observed and not active against influenza B. No longer recommended.

At which stage of pregnancy are women at greatest risk of severe disease following H1N1
Question 9
influenza infection?
Options for Questions 9-9

A First trimester B Second trimester


C Third trimester D 24-48h post-partum
E 4-6 weeks post-partum

A(Correct answ er: C)

Explanation
H1N1 Influenza and pregnancy
• Pregnant women are more likely to have severe disease, be admitted to ITU and die from H1N1 influenza compared
to non-pregnant women. Most reported deaths occurred in women who developed pneumonia and acute respiratory
distress syndrome requiring mechanical ventilation
• Risk of morbidity from seasonal flu is also higher during pregnancy

This Copy is for Dr. Mohamed ElHodiby


• The risk is highest in the third trimester compared to early pregnancy or post-partum
• Women with asthma and morbid obesity appear to be at higher risk of severe disease
• Most current data from UKOSS indicates that pregnant women admitted to ITU were less likely to have been treated
with antiviral agents within 2 days of the onset of symptoms when compared with other pregnant women who were
admitted to hospital. This indicates the importance of prompt treatment
• H1N1 infection is associated with pre-term delivery, especially in women admitted to ITU
• Vertical transmission of H1N1 influenza has not been reported and the virus is not known to be present in breast
milk. Breastfeeding should continue in women being treated for H1N1 influenza
• The H1N1 virus is not terratogenic but there some evidence that pyrexia in early pregnancy may be associated with
an increased risk of neural tube defects

Question 10 Infection with the H1N1 influenza virus

Options for Questions 10-10

Occurring in the third trimester is associated with Occurring in the first trimester is associated with
A B
a 10-25% risk of vertical transmission an increased risk of congenital anomalies
Is associated with an increased risk of pre-term Is associated with an increased risk of pre-
C D
delivery eclampsia
E Causes less severe disease in obese women

A(Correct answ er: C)

Explanation
H1N1 Influenza and pregnancy
• Pregnant women are more likely to have severe disease, be admitted to ITU and die from H1N1 influenza compared
to non-pregnant women. Most reported deaths occurred in women who developed pneumonia and acute respiratory
distress syndrome requiring mechanical ventilation
• Risk of morbidity from seasonal flu is also higher during pregnancy
• The risk is highest in the third trimester compared to early pregnancy or post-partum
• Women with asthma and morbid obesity appear to be at higher risk of severe disease
• Most current data from UKOSS indicates that pregnant women admitted to ITU were less likely to have been treated
with antiviral agents within 2 days of the onset of symptoms when compared with other pregnant women who were
admitted to hospital. This indicates the importance of prompt treatment
• H1N1 infection is associated with pre-term delivery, especially in women admitted to ITU
• Vertical transmission of H1N1 influenza has not been reported and the virus is not known to be present in breast
milk. Breastfeeding should continue in women being treated for H1N1 influenza
• The H1N1 virus is not terratogenic but there some evidence that pyrexia in early pregnancy may be associated with
an increased risk of neural tube defects

Question 11 Which bacteria typically cause toxic shock syndrome?

Options for Questions 11-11

A Group A streptococci B Group B streptococci


C Coagulase negative staphylococci D Coagulase positive staphylococci
E Actinomycetes Israelii

A(Correct answ er: D)

Explanation

This Copy is for Dr. Mohamed ElHodiby


TOXIC SHOCK SYNDROME (TSS)
• Rare, life-threatening inflammatory response syndrome characterized by fever, rash, hypotension,
constitutional symptoms, and multi-organ involvement
• Most often associated with the use of super-absorbent tampons and occasionally with the use of
contraceptive sponges.
• Associated with infection with toxigenic strains of Staphylococcus aureus
• The infection can also affect men, children and postmenopausal women.
• Signs and symptoms develop suddenly
• TSS has been linked to many bacterial infections, including pneumonia, osteomyelitis, sinusitis, and
skin and gynecologic infections
• Endotoxin toxic shock syndrome toxin-1 (TSST-1) is the major toxin produced by strains of S aureus
that are responsible for causing TSS.
• Streptococcus pyogenes exotoxin A (SPEA) and S pyogenes exotoxin B (SPEB) are the major
toxins produced by group A beta-hemolytic streptococci.
The toxins activate production of cytokines, such as tumor necrosis factor, interleukin-1, M protein, and gamma-
interferon and induction of nitric oxide production which contributes to hypotension.

Question 12 Which organism is typically sensitive to penicillin?

Options for Questions 12-12

A E Coli B Staph. Aureus


C Bacillus anthracis D C. Difficile
E Staph. Epidermides

A(Correct answ er: C)

Explanation
BACILLUS SPECIES
• B. anthracis - gram positive aerobic spore-forming bacilli; capsulated
• Grows on blood agar
• Spores may remain dormant in the environment for several years
• Infects many animals including sheep
• Causes anthrax
• Produces an exotoxin
• B. cereus contaminates food such as boiled rice, causing food poisoning
Sensitive to penicillin

Question 13 Gas gangrene is caused by

Options for Questions 13-13

A Actinomycetes israelii B Clostridium perfringens


C Clostridium difficile D Bacillus anthracis
E Staphylococcus epidermidis

A(Correct answ er: B)

Explanation
CLOSTRIDIA
• Gram positive anaerobic bacilli
• Spore-forming -the spores are resistant to boiling and chemical disinfectants
• Commonly found in the environment and as part of normal fecal flora
• Produce EXOTOXINS
• Infection common in gangrenous / ischaemic tissue and in association with foreign bodies
• Sensitive to metronidazole and penicillin

This Copy is for Dr. Mohamed ElHodiby


• CLOSTRIDIUN DIFFICILE - causes pseudo-membranous colitis - diarrhoea with collapse following broad
spectrum antibiotic treatment - treated with oral metronidazole or vancomycin
• CLOSTRIDIUM PERFRINGENS - Causes gas gangrene (causing crepitus) and septic abortion (rare)
• Produces exotoxin causing septic shock, jaundice, haemolysis and acute renal failure
Sensitive to penicillin - surgical debridement usually required

Question 14 Infection with Clostridium tetani

Options for Questions 14-14

A Has an incubation period of 3-6 months B Results in life-long immunity


Is characterized by an extensive local
C D Is treated with tetanus immune globulin
inflammatory response
E Is treated with metronidazole

A(Correct answ er: D)

Explanation
Clostridium Tetani
• Flagellated bacilli. Spores are resistant to boiling and autoclaving is required
• Incubation period 3-21 days
• Produces EXOTOXIN which is non-immunogenic therefore infection does not confer immunity.
Neurotoxin, inhibits release of inhibitory neurotransmitters in the CNS causing tetany / muscle spasms
• There is no local inflammation associated with tissue infection
• Infection prevented by vaccination with tetanus toxoid in infancy followed by a booster dose every 5-10
years
• Passive immunisation with human tetanus immune globulin + active immunisation required in at-risk
individuals following injury
Antibiotic treatment is an ancillary measure and should include penicillin +/- flucloxacillin. Removal of devitalised
tissue, good tissue perfusion and oxygenation are essential

Question 15 Clostridium tetani is sensitive to the following antibiotics

Options for Questions 15-15

A Penicillin and metronidazole B Cefuroxime and metronidazole


C Penicillin and flucloxacillin D Gentamicin and doxycycline
E Rifampicin and trimrthoprim
A(Correct answ er: C)

Explanation
Clostridium Tetani
• Flagellated bacilli. Spores are resistant to boiling and autoclaving is required
• Incubation period 3-21 days
• Produces EXOTOXIN which is non-immunogenic therefore infection does not confer immunity.
Neurotoxin, inhibits release of inhibitory neurotransmitters in the CNS causing tetany / muscle spasms
• There is no local inflammation associated with tissue infection
• Infection prevented by vaccination with tetanus toxoid in infancy followed by a booster dose every 5-10
years
• Passive immunisation with human tetanus immune globulin + active immunisation required in at-risk
individuals following injury
Antibiotic treatment is an ancillary measure and should include penicillin +/- flucloxacillin. Removal of devitalised
tissue, good tissue perfusion and oxygenation are essential

This Copy is for Dr. Mohamed ElHodiby


Question 16 Endotoxins

Options for Questions 16-16

A Are polypeptide molecules B Are produced by living bacteria


C Stimulate the production of natural anti-toxins D Are converted to non-toxic toxoid by formalin
Are associated with disseminated intra-vascular
E
coagulation

A(Correct answ er: E)

Explanation
EXOTOXINS
• Highly toxic polypeptides produced by living bacteria
• Act at specific target sites
• Are highly antigenic (except Cl. Tetani toxin)
• Are converted into antigenic non-toxic toxoids by formalin
• Are often destroyed by heat
• Are neutralised by anti-toxins
• Typically produced by gram positive bacteria

ENDOTOXINS
• Are lipopolysaccharide molecules
• Present in the outer layer of the cell wall of gram negative bacteria, released on disintegration
• Relatively heat stable
• Not converted into toxoid by formalin
• Act non-specifically on the reticulo-endothelial system causing increased vascular permeability and
prostaglandin release
• May cause DIC and cardiovascular collapse
• Detectable by the Limulus test (A lysate of blood cells from the horseshoe crab is used to test specimens
for the presence of Escherichia coli endotoxin in blood or milk)
• Do not stimulate production of natural anti-toxins
Produced by E. coli, N. meningitides, S. typhi, P. aeruginosa

Question 17 Which antibiotic is used to treat syphilis infection in the first trimester of pregnancy?

Options for Questions 17-17

A Azithromycin B Erythromycin
C Procaine penicillin D Clindamycin
E Tazocin

A(Correct answ er: C)

Explanation
Treatment of syphilis
• Procaine penicillin is first line treatment
• Treponema pallidum is also sensitive to erythromycin and tetracycline although the results of treatment are less
satisfactory
• The Jerisch-Herxheimer reaction is an inflammatory response to the increased release of treponemal antigens during
treatment -managed with steroids.

This Copy is for Dr. Mohamed ElHodiby


Question 18 Which infection is not caused by Chlamydia trachomatis?

Options for Questions 18-18

A Salpingitis B Conjunctivitis
C Bartholinitis D Interstitial pneumonitis
E Pharyngitis

A(Correct answ er: E)

Explanation
CHLAMYDIA TRACHOMATIS
• Obligate intracellular organism - bacteria
• Non-motile gram negative or gram variable although gram stain is not useful in diagnosis
• Enter the host through abrasions in mucosal surfaces
• Does not infect squamous cells - the vaginal epithelium and ectocervix are not infected. Endocervical glands are
susceptible
• Produce infectious particles called Elementary Bodies which enter cells by endocytosis
• Differentiate into Reticulate Bodies within cells. These multiply by binary fission and then produce other elementary
bodies which are released from the cell
• Asymptomatic infection is common
• C. psittaci infects birds and non-human animals
Various serovars:
• A - C: Trachoma (blindness)
• D - K: genital infection - cervicitis, urethritis, salpingitis, epididymitis, proctitis, Bartholinitis, conjunctivitis, interstitial
pneumonitis in neonates.
• Consequences of infection include infertility, ectopic pregnancy, Fitz-Hugh-Curtis syndrome, Reiter's syndrome,
arthritis, dermatitis
• L1-3: Lymphogranuloma venereum

A 23 year old woman is found to have chlamydia cervicitis at 6 weeks gestation. If untreated,
Question 19
infection is associated with
Options for Questions 19-19

A 1-2% risk of congenital anomalies B 1-2% risk of neonatal chlamydia infection


C 5-10% risk of neonatal chlamydia infection D 15-20% risk of neonatal chlamydia infection
E Over 50% risk of neonatal chlamydia infection

A(Correct answ er: E)

Explanation
Chlamydia infection in pregnancy
• 60% risk of neonatal chlamydial infection including conjunctivitis and neonatal pneumonia
• Antenatal treatment with Erythromycin, 500mg qds for seven days. Amoxicillin 500mg tds for 7 days or Azithromycin
1g stat are alternatives
• Consider referral to GUM clinic for contact tracing
• Test of cure recommended 5-6 weeks after treatment

This Copy is for Dr. Mohamed ElHodiby


Question 20 Which infection is not caused by Neisseria gonorrhoea?

Options for Questions 20-20

A Arthritis B Endocarditis
C Epididymitis D Salpingitis
E Stomatitis

A(Correct answ er: E)

Explanation
Neisseria gonorrhoea
• Incubation period 2-7 days, majority of infected females are asymptomatic
• Males: 20% chance; Females: 50% chance of being infected after one episode of unprotected intercourse with
infected partner
• Causes urethritis, cervicitis, epididymitis, pharyngitis, proctitis, endocarditis, skin infection, arthritis and vertical
neonatal infection (ophthalmia neonatorum). Salpingitis (PID) may lead to infertility.
TREATMENT
• Procaine penicillin im + probenecid
• Third generation cephalosporin
• Ciprofloxacin
• Spectinomycin
• Azythromycin
Contact tracing required

Question 21 Which organism typically causes opportunistic infections?

Options for Questions 21-21

A E. Coli B P. Aeruginosa
C S. Typhi D P. Mirabilis
E N. Meningitides

A(Correct answ er: B)

Explanation
PSEUDOMONAS AERUGINOSA
• Gram negative bacilli, indistinguishable from coliforms on gram staining
• Strict aerobes
• The bacterium is ubiquitous in soil and water, and on surfaces in contact with soil or water. They occur
regularly on the surfaces of plants and occasionally on the surfaces of animals.
• Its metabolism is respiratory and never fermentative
• Almost all strains are motile by means of a single polar flagellum
• The bacterium almost never infects uncompromised tissues, yet there is hardly any tissue that it cannot
infect if the tissue defenses are compromised in some manner
• P. aeruginosa strains produce two types of soluble pigments, the fluorescent pigment pyoverdin and the
blue pigment pyocyanin. The latter is produced abundantly in media of low-iron content and functions in iron
metabolism in the bacterium
• Grow well on selective agar containing the disinfectant cetrimide
• Strongly oxidase positive (coliforms are oxidase negative)
• Metabolises glucose by oxidation
• Not part of normal gut flora

This Copy is for Dr. Mohamed ElHodiby


• Causes chronic UTIs, wound infections, chronic osteomyelitis and opportunistic infections
• Resistant to ampicillin, trimethoprim, tetracycline, sulphonamides
• Sensitive to aminoglycosides gentamicin and netilmicin
• Sensitive to anti-pseudomonas penicillins carbenicillin, ticarcillin, piperacillin
Most are sensitive to ceftazidime, ciprofloxacin and imipenam

Question 22 With respect to infection with Toxoplasma gondii during pregnancy

Options for Questions 22-22

The risk of fetal infection decreases with The risk of the fetus being affected decreases
A B
increasing gestation age with increasing gestation age
Maternal infection typically presents with Infection in pregnancy is treated with
C D
septicaemia erythromycin
Infection in pregnancy is typically detected during
E
routine screening

A(Correct answ er: B)

Explanation
TOXOPLASMA GONDII
• Obligate intracellular protozoan
• Completes part of its life cycle in animals (mainly cats but also sheep, cattle and pigs)
• Contracted by contact with cat litter, infected soil or eating undercooked beef, lamb or pork. Prevented by
wearing gloves during gardening, thorough food hygiene and avoiding contact with cat litter
• Oocysts or tissue cysts invades the intestinal epithelium , develop into trophozoites which spread via the
lymphatic system
• Infection is usually mild or asymptomatic with lymphadenopathy (posterior cervical) being the main
presentation. Causes severe disease with encephalitis and chorioretinitis in the immunocompromised
• Risk of fetal infection increases with increasing gestation age (17% first, 25% second and 65% third
trimester)
• Risk of fetus being affected FALLS with increasing gestation age
• Recognised cause of congenital anomalies with chorioretinitis, intra-cranial calcification, microcephaly,
hydrocephalus. Increased risk of miscarriage with first trimester infection
• Diagnosis can be made by culture in laboratory rodents
• Detection of IgM or a 4 fold rise in IgG titres in samples 3 weeks apart is indicative of acute infection
Sensitive to spiramycin and pyramethamine

Question 23 Mycobacterium tuberculosis

Options for Questions 23-23

A Is an anaerobic organism B Is an intra-cellular organism


Infection is associated with a Type II hyper-
C D Can only divide within infected cells
sensitivity reaction
E Does not multiply within macrophages

A(Correct answ er: B)

Explanation
MYCOBACTERIA
• Cause tuberculosis, leprosy in addition to other opportunistic infections
• Present as 2-4 micron long acid-fast bacilli on the Ziehl-Neelsen stain
• Multiply intracellularly in macrophages but also divide extra-cellularly

This Copy is for Dr. Mohamed ElHodiby


• Infection results in delayed hypersensitivity response
• Not all mycobacteria are pathogenic in humans
• All mycobacteria are:

1. Acid fast- i.e. they do not de-stain with acid and alcohol once stained with arylmethane dye
2. Aerobic
3. Contain mycolic acids
4. Intracellular pathogens- the wall helps the organism to survive within the macrophage by resisting oxidative
damage

A 23 year old woman presents with a 3 months history of cough and weight loss. She is
Question 24
suspected of having pulmonary tuberculosis
Options for Questions 24-24

Sputum culture results are typically available


A B A negative sputum culture excludes tuberculosis
within 3-7 days
Chest X-ray is contra-indicated in the first The Zeihl-Neelsen stain can be used to make the
C D
trimester diagnosis
E A positive tuberculin test confirms the diagnosis

A(Correct answ er: D)

Explanation
Mycobacterium tuberculosis
• Slender, straight or slightly curved bacillus, non-motile, non-encapsulated and does not form spores
• Acid fast bacillus (AFB)
• Aerobic
• Slow growing- divides every 18-24 hr
• Resistant to drying and chemical disinfectants
• Sensitive to heat (Pasteurization) and UV light

Tuberculosis
• Transmission occurs aerosol / droplet infection
• One organism may be enough to establish an infection
• TB is a chronic disease and an infected person can spread the infective organism, M. tuberculosis, to
several contacts before being diagnosed
• The main site of infection is the lung (>80%)
• Any organ can be infected via lymphatic drainage or haematological spread
• Symptoms include severe weight loss, night sweats, chronic cough, haemoptis
• A person may be infected without developing disease or remain healthy and free of disease for decades.
The infecting organisms remain viable. Disease may be activated by malignancy, immune suppression, old
age or chronic ill-health
• Diagnosis based on:

1) Clinical presentation
2) Radiology -CXR
3)Microbiology - demonstration of M. tuberculosis in a clinical specimen: microscopic (Ziehl-Neelsen or Auramine
O stain for acid-fast bacilli) and / or Culture- may take 2-6 weeks for the isolation of M. tuberculosis
4)Negative culture does not exclude tuberculosis

Question 25 Which antibiotic does not act by inhibiting bacterial cell wall synthesis?

Options for Questions 25-25

A Penicillin B Cefalexine
C Vancomycin D Teicoplanin
E Gentamicin

This Copy is for Dr. Mohamed ElHodiby


A(Correct answ er: E)

Explanation
ANTIBIOTICS AND CELL WALL SYNTHESIS
• Beta lactams - inhibit transpeptidation (penicillins, monobactams, carbapenams and ceplalosporins)
• Cycloserine - inhibits reactions incorporating alanine into cell wall precursor within the cytoplasm
• Glycopeptides - vancomycin, teicoplanin - bind to terminal d-alanyl-d-alanine residue, preventing
incorporation into growing peptidoglycan chain
• Bacitracin - prevents dephophorylation of phosphilipid carrier, preventing regeneration of carrier required
for continuous synthesis
GLYCOPEPTIDES
• Vancomycin and teicoplanin
• Inhibit bacterial cell wall synthesis by binding to d-alanyl-d-alanine at the end of a pentapeptide chain,
preventing the incorporation of new subunits
• Large molecules which cannot penetrate gram negative cell wall. Only active against gram positive
bacteria
• Not absorbed from the GI tract. Oral administration used to treat Clostridium difficile diarrhoea
• Potentially ototoxic and nephrotoxic - vancomycin should be administered slowly to prevent 'red-man'
syndrome. Teicoplanin is less toxic and can be given bolus

Question 26 Aminoglycoside antibiotics

Options for Questions 26-26

A Inhibit bacterial protein synthesis B Are effective against streptococci


C Are well absorbed from the GI tract D Are mainly metabolized by the liver
E Include vancomycin

A(Correct answ er: A)

Explanation
AMINOGLYCOSIDES
• Gentamicin, tobramycin, netilmicin, streptomycin
• Inhibit bacterial protein synthesis - interfere with the binding of formylmethionyl-transfer RNA to the
ribosomes, preventing the formation of the initiation complex
• Not absorbed from the GI tract. Do not cross the blood-brain barrier
• Not active against streptococci. Active against gram negative bacteria
• Excreted via the kidneys
• Potentially ototoxic and nephrotoxic
• Production of aminoglycoside modifying enzymes is the most important mechanism of acquired bacterial
resistance - usually plasmid mediated

Question 27 Which statement regarding aminoglycoside antibiotics is not true?

Options for Questions 27-27

A They are potentially nephrotoxic B They are potentially ototoxic


They should be administered through the
C They cross the blood-brain barrier very well D
intravenous route
E They are effective against gram negative bacteria

This Copy is for Dr. Mohamed ElHodiby


A(Correct answ er: C)

Explanation
AMINOGLYCOSIDES
• Gentamicin, tobramycin, netilmicin, streptomycin
• Inhibit bacterial protein synthesis - interfere with the binding of formylmethionyl-transfer RNA to the
ribosomes, preventing the formation of the initiation complex
• Not absorbed from the GI tract. Do not cross the blood-brain barrier
• Not active against streptococci. Active against gram negative bacteria
• Excreted via the kidneys
• Potentially ototoxic and nephrotoxic
• Production of aminoglycoside modifying enzymes is the most important mechanism of acquired bacterial
resistance - usually plasmid mediated

Question 28 Which statement regarding trimethoprim is not true?

Options for Questions 28-28

A It is a folic acid analogue B It inhibits dihydrofolate reductase


It is mainly used to treat infections with gram
C D It is well absorbed from the GI tract
positive bacteria
E It is predominantly excreted by the kidneys

A(Correct answ er: C)

Explanation
TRIMETHOPRIM
• Folic acid analogue
• Inhibits dihydrofolate reductase
• Similar structure and mechanism of action to methotrexate and the anti-malarial agent pyrimethamine
• Absorbed from the gut
• Excreted by the kidneys
• Effective against gram negative bacilli EXCEPT pseudomonas
• Resistance is mediated by a plasmid-encoded dihydrofolate reductase which has a much reduced affinity for
trimethoprim

Question 29 Which one is not an anti-fungal drug?

Options for Questions 29-29

A Clotrimazole B Cotrimoxazole
C Flucytosine D Nystatin
E Fluconazole

A(Correct answ er: B)

Explanation
ANTI-FUNGAL AGENTS
• Azole compounds - clotrimazole, ketoconazole, fluconazole: inhibit lanosterol C14 demethylase,an important enzyme
in sterol biosynthesis
• Polyene compounds - amphotericin B, nystatin: bind ergosterol in cell membrane resulting in leakage of cellular
contents

This Copy is for Dr. Mohamed ElHodiby


• Flucytosine: pro-drug, taken up by fungal cells and converted into 5-fluorouracil. Inhibits DNA synthesis
• Griseofulvin: inhibits mitosis and nucleic acid synthesis

Question 30 Which disinfectant may be used to prepare the vagina prior to vaginal hysterectomy?

Options for Questions 30-30

A Iodine (3%) solution B Povidone-iodine solution


C Isopropyl alcohol 70% D Chlorhexidine 2% + alcohol 70%
E Chlorhexidine 0.2%

A(Correct answ er: B)

Explanation
Iodophores (7.5 – 10%)
• Preparation containing iodine complexed with a solubilizing agent, such as a surfactant or povidone (povidone-iodine)
• Result in a solution that releases free iodine
• Excellent activity against gram positive and most gram negative bacteria
• Fair activity against TB
• No activity against endospores
• Good activity against viruses and fungi
• Activity is moderately affected by organic matter
• Useful as surgical scrub and skin prep
• Can be used on mucous membranes
• Iodine may be absorbed by the neonate and affect thyroid function

This Copy is for Dr. Mohamed ElHodiby


Question 1 Characteristic feature of primary syphilis

Options for Questions 1-1

A Chancre B Gumma
C Tabes dorsalis D Serpiginous ulcers
E Maculo-papular rash

A(Correct answ er: A)

Explanation
SYPHILIS
• Sexually transmitted infection caused by Treponema pallidum
• Spirocheate which cannot be cultured in-vitro. Can be visualised by dark-ground microscopy, silver staining or
immunofluorescence
• Incubation period 10-90 days
• PRIMARY: Painless chancre on genital or extra-genital sites. Regional lymphadenopathy may occur
• SECONDARY - generalised malaise, sore throat, serpiginous ulcers in the mouth, maculo-papular rash and
generalised lymphadenopathy. Primary and secondary syphilis are infectious. Occurs 6-8 weeks after the primary
stage
• LATENT - may last 3-30 years
• TERTIARY - characteristic lesion is the GUMMA - a granuloma involving the skin, mucous membranes, bones and
joints (Charcot’s joints)
• QUATERNARY

1) CVS - aortic incompetence, coronary ostia stenosis, aortic aneurysm

2) CNS - meningo-vascular syphilis, Tabes Dorsalis, Generalised paralysis of the insane

Question 2 Characteristic feature of secondary syphilis

Options for Questions 2-2

A Gumma B Regional lymphadenopathy


C Generalised lymphadenopathy D Charcot’s joints
E Aortic aneurysm

A(Correct answ er: C)

Explanation
SYPHILIS
• Sexually transmitted infection caused by Treponema pallidum
• Spirocheate which cannot be cultured in-vitro. Can be visualised by dark-ground microscopy, silver staining or
immunofluorescence
• Incubation period 10-90 days
• PRIMARY: Painless chancre on genital or extra-genital sites. Regional lymphadenopathy may occur
• SECONDARY - generalised malaise, sore throat, serpiginous ulcers in the mouth, maculo-papular rash and
generalised lymphadenopathy. Primary and secondary syphilis are infectious. Occurs 6-8 weeks after the primary
stage
• LATENT - may last 3-30 years

This Copy is for Dr. Mohamed ElHodiby


• TERTIARY - characteristic lesion is the GUMMA - a granuloma involving the skin, mucous membranes, bones and
joints (Charcot’s joints)
• QUATERNARY

1) CVS - aortic incompetence, coronary ostia stenosis, aortic aneurysm

2) CNS - meningo-vascular syphilis, Tabes Dorsalis, Generalised paralysis of the insane

Question 3 Characteristic feature of tertiary syphilis

Options for Questions 3-3

A Gumma B Regional lymphadenopathy


C Generalised lymphadenopathy D Tabes dorsalis
E Aortic aneurysm
A(Correct answ er: A)

Explanation
SYPHILIS
• Sexually transmitted infection caused by Treponema pallidum
• Spirocheate which cannot be cultured in-vitro. Can be visualised by dark-ground microscopy, silver staining or
immunofluorescence
• Incubation period 10-90 days
• PRIMARY: Painless chancre on genital or extra-genital sites. Regional lymphadenopathy may occur
• SECONDARY - generalised malaise, sore throat, serpiginous ulcers in the mouth, maculo-papular rash and
generalised lymphadenopathy. Primary and secondary syphilis are infectious. Occurs 6-8 weeks after the primary
stage
• LATENT - may last 3-30 years
• TERTIARY - characteristic lesion is the GUMMA - a granuloma involving the skin, mucous membranes, bones and
joints (Charcot’s joints)
QUATERNARY

1) CVS - aortic incompetence, coronary ostia stenosis, aortic aneurysm

2) CNS - meningo-vascular syphilis, Tabes Dorsalis, Generalised paralysis of the insane

Question 4 With respect to syphilis

Options for Questions 4-4

Secondary syphilis is characterized by central


A Secondary syphilis is not infectious B
nervous system involvement
Both primary and secondary syphilis are
C Mouth ulcers typically occur in latent syphilis D
infectious
Tertiary syphilis is characterized by generalized
E
lymphadenopathy

A(Correct answ er: D)

Explanation
SYPHILIS
• Sexually transmitted infection caused by Treponema pallidum
• Spirocheate which cannot be cultured in-vitro. Can be visualised by dark-ground microscopy, silver staining or
immunofluorescence

This Copy is for Dr. Mohamed ElHodiby


• Incubation period 10-90 days
• PRIMARY: Painless chancre on genital or extra-genital sites. Regional lymphadenopathy may occur
• SECONDARY - generalised malaise, sore throat, serpiginous ulcers in the mouth, maculo-papular rash and
generalised lymphadenopathy. Primary and secondary syphilis are infectious. Occurs 6-8 weeks after the primary
stage
• LATENT - may last 3-30 years
• TERTIARY - characteristic lesion is the GUMMA - a granuloma involving the skin, mucous membranes, bones and
joints (Charcot’s joints)
• QUATERNARY

1) CVS - aortic incompetence, coronary ostia stenosis, aortic aneurysm

2) CNS - meningo-vascular syphilis, Tabes Dorsalis, Generalised paralysis of the insane

Question 5 Test results in a pregnant woman with secondary syphilis

Options for Questions 5-5

VDRL positive and fluorescent treponemal


A VDRL test is negative B
antibody tests positive
VDRL test negative and fluorescent treponemal VDRL test positive and fluorescent treponemal
C D
antibody tests positive antibody tests negative
E Fluorescent treponemal antibody tests negative

A(Correct answ er: B)

Explanation
DIAGNOSIS OF SYPHILIS
• Cardiolipin antibody tests(such as Wassermann reaction, rapid plasma regain test and VDRL - Venereal
Disease Reference Laboratory) which are relatively non-specific
• Treponemal antibody testssuch as the Fluorescent Treponemal Antibody test -Absorbed (FTA-Abs) which
are specific and used as verification tests
• Cardiolipin antibody tests are positive in the serum in untreated secondary, latent and tertiary syphilis and
may be false positive
• Cardiolipin antibody tests become negative with treatment
• Primary: VDRL (+ve or -ve), FTA-Abs (+ve). FTA-Abs usually becomes positive before VDRL
• Secondary : VDRL and FTA-Abs (+ve)
• Latent : VDRL (usually +ve), FTA-Abs (+ve)
• Tertiary / quaternary : VDRL (+ve or -ve), FTA-Abs (+ve)
• Treated : VDRL (-ve), FTA-Abs (+ve)
• The VDRL test becomes negative with treatment and can be used to monitor treatment. The FTA-Abs test
can remain positive for several years after adequate treatment

Question 6 Test results in a pregnant woman with latent syphilis

Options for Questions 6-6

VDRL positive and fluorescent treponemal


A VDRL test is negative B
antibody tests positive
VDRL test negative and fluorescent treponemal VDRL test positive and fluorescent treponemal
C D
antibody tests positive antibody tests negative
E Fluorescent treponemal antibody tests negative

This Copy is for Dr. Mohamed ElHodiby


A(Correct answ er: B)

Explanation
DIAGNOSIS OF SYPHILIS
• Cardiolipin antibody tests(such as Wassermann reaction, rapid plasma regain test and VDRL - Venereal
Disease Reference Laboratory) which are relatively non-specific
• Treponemal antibody testssuch as the Fluorescent Treponemal Antibody test -Absorbed (FTA-Abs) which
are specific and used as verification tests
• Cardiolipin antibody tests are positive in the serum in untreated secondary, latent and tertiary syphilis and
may be false positive
• Cardiolipin antibody tests become negative with treatment
• Primary: VDRL (+ve or -ve), FTA-Abs (+ve). FTA-Abs usually becomes positive before VDRL
• Secondary : VDRL and FTA-Abs (+ve)
• Latent : VDRL (usually +ve), FTA-Abs (+ve)
• Tertiary / quaternary : VDRL (+ve or -ve), FTA-Abs (+ve)
• Treated : VDRL (-ve), FTA-Abs (+ve)
• The VDRL test becomes negative with treatment and can be used to monitor treatment. The FTA-Abs test
can remain positive for several years after adequate treatment

Question 7 Which one is not a recognised cause of biological false positive syphilis serology?

Options for Questions 7-7

A Treatment with methyl dopa B Pregnancy


C Systemic lupus erythematosus D Malaria infection
E Recent immunisation

A(Correct answ er: A)

Explanation
Causes of biological false positive test for syphilis
• Recent immunisation, pneumonia, malaria, pregnancy - cause acute false positives (last less than 6 months)
• SLE, and other auto-immune disorders, acquired haemolytic anaemia, leprosy and drug addiction cause chronic
biological false positives

Question 8 Chlamydia trachomatis

Options for Questions 8-8

A Multiplies outside infected cells B Infects the squamous epithelium of the ectocervix
C Can infect the vaginal mucosa D Can infect the lower third of the anal canal
E Infects the glandular epithelium of the endocervix

A(Correct answ er: E)

Explanation
CHLAMYDIA TRACHOMATIS
• Obligate intracellular organism - bacteria
• Non-motile gram negative or gram variable although gram stain is not useful in diagnosis
• Enter the host through abrasions in mucosal surfaces

This Copy is for Dr. Mohamed ElHodiby


• Does not infect squamous cells - the vaginal epithelium and ectocervix are not infected. Endocervical glands are
susceptible
• Produce infectious particles called Elementary Bodies which enter cells by endocytosis
• Differentiate into Reticulate Bodies within cells. These multiply by binary fission and then produce other elementary
bodies which are released from the cell
• Asymptomatic infection is common
• C. psittaci infects birds and non-human animals

Question 9 Which infection is not caused by Chlamydia trachomatis?

Options for Questions 9-9

A Cervicitis B Urethritis
C Vaginitis D Proctitis
E Epididymitis

A(Correct answ er: C)

Explanation
CHLAMYDIA TRACHOMATIS
• Obligate intracellular organism - bacteria
• Non-motile gram negative or gram variable although gram stain is not useful in diagnosis
• Enter the host through abrasions in mucosal surfaces
• Does not infect squamous cells - the vaginal epithelium and ectocervix are not infected. Endocervical glands are
susceptible
• Produce infectious particles called Elementary Bodies which enter cells by endocytosis
• Differentiate into Reticulate Bodies within cells. These multiply by binary fission and then produce other elementary
bodies which are released from the cell
• Asymptomatic infection is common
• C. psittaci infects birds and non-human animals
Various serovars:
• A - C: Trachoma (blindness)
• D - K: genital infection - cervicitis, urethritis, salpingitis, epididymitis, proctitis, Bartholinitis, conjunctivitis, interstitial
pneumonitis in neonates.
• Consequences of infection include infertility, ectopic pregnancy, Fitz-Hugh-Curtis syndrome, Reiter's syndrome,
arthritis, dermatitis
• L1-3: Lymphogranuloma venereum

Question 10 Which technique is not used to diagnose Chlamydia trachomatis infection ?

Options for Questions 10-10

A ELISA B PCR
C LCR D Immuno-fluorescence
E Serology

A(Correct answ er: E)

Explanation
Diagnosis of Chlamydia trachomatis
• ELISA test - endocervical (or other) swab - sensitivity ~70%
• Immuno-fluorescence
• PCR or LCR (Ligase Chain Reaction) - early morning urine specimen - most sensitive test

This Copy is for Dr. Mohamed ElHodiby


• Culture - requires special (McCoy) cells
• Serology - unreliable

A 23 year old woman is found to have chlamydia cervicitis at 6 weeks gestation. If untreated,
Question 11
infection is associated with
Options for Questions 11-11

A 1-2% risk of congenital anomalies B 1-2% risk of neonatal chlamydia infection


C 5-10% risk of neonatal chlamydia infection D 15-20% risk of neonatal chlamydia infection
E Over 50% risk of neonatal chlamydia infection

A(Correct answ er: E)

Explanation
Chlamydia infection in pregnancy
• 60% risk of neonatal chlamydial infection including conjunctivitis and neonatal pneumonia
• Antenatal treatment with Erythromycin, 500mg qds for seven days. Amoxicillin 500mg tds for 7 days or Azithromycin
1g stat are alternatives
• Consider referral to GUM clinic for contact tracing
• Test of cure recommended 5-6 weeks after treatment

Question 12 Neisseria gonorrhoea

Options for Questions 12-12

Infects mucous membranes lined by squamous


A B Is a gram positive diplococcus
epithelium
C Is lactose- fermenting D Infection has an incubation period of 2-7 days
E Typically causes asymptomatic infection in males

A(Correct answ er: D)

Explanation
NEISSERIA GONORRHOEA
• Gram negative intracellular diplococcus
• Fragile organism - very sensitive to drying and low temperature- swabs should be collected in Stuart's transport
medium, maintained at room temperature and cultured on selective VCT medium (vancomycin, collistin,
trimethoprim).
• Grows on blood or chocolate agar
• Infects mucous membranes lined by non-squamous epithelium
• Glucose fermenting (N. meningitides is Maltose fermenting)
• Serological diagnosis is unreliable.
• Has pili which are important in virulence
• Neisseria meningitides is carried in the nasopharynx of 5-30%of adults and is one of the three primary pathogens
causing bacterial meningitis
INFECTION
• Incubation period 2-7 days, majority of infected females are asymptomatic
• Males: 20% chance; Females: 50% chance of being infected after one episode of unprotected intercourse with
infected partner
• Causes urethritis, cervicitis, epididymitis, pharyngitis, proctitis, endocarditis, skin infection, arthritis and vertical
neonatal infection (ophthalmia neonatorum). Salpingitis (PID) may lead to infertility.

This Copy is for Dr. Mohamed ElHodiby


Question 13 Which infection is not caused by Neisseria gonorrhoea?

Options for Questions 13-13

A Arthritis B Endocarditis
C Epididymitis D Salpingitis
E Stomatitis

A(Correct answ er: E)

Explanation
Neisseria gonorrhoea
• Incubation period 2-7 days, majority of infected females are asymptomatic
• Males: 20% chance; Females: 50% chance of being infected after one episode of unprotected intercourse with
infected partner
• Causes urethritis, cervicitis, epididymitis, pharyngitis, proctitis, endocarditis, skin infection, arthritis and vertical
neonatal infection (ophthalmia neonatorum). Salpingitis (PID) may lead to infertility.
TREATMENT
• Procaine penicillin im + probenecid
• Third generation cephalosporin
• Ciprofloxacin
• Spectinomycin
• Azythromycin
Contact tracing required

Question 14 Vaginal infection characterised by the presence of clue cells

Options for Questions 14-14

A Bacterial vaginosis B Gardnerella vaginalis infection


C Trichomonas vaginalis infection D Candida albicans infection
E Chlamydia trachomatis infection

A(Correct answ er: A)

Explanation
BACTERIAL VAGINOSIS
Polymicrobial infection characterised by:
• Copious frothy vaginal discharge with a fishy odour
• Increased vaginal pH (>4.5)
• Clue cells - vaginal epithelial cells coated with gram-variable bacteria on wet vaginal smear
• Vaginosis - there is minimal inflammation
• Positive amine test - pungent fishy odour on adding the vaginal discharge to sodium hydroxide
• Associated with an alteration in vaginal flora with decreased lactobacilli and increased anaerobes and
Gaednerella vaginalis
Gardnerella vaginalis
• Gram variable, can be isolated from the vaginal flora in 20% of asymptomatic women and present in the urethra of
male partners of women with bacterial vaginosis
• Treated with metronidazole

This Copy is for Dr. Mohamed ElHodiby


A 35 year old woman complains of a 2 months history of offensive white vaginal discharge.
Question 15
Addition of potassium hydroxide to a sample of the discharge produced a strong fishy odour.
Options for Questions 15-15

A Bacterial vaginosis B Gardnerella vaginalis infection


C Trichomonas vaginalis infection D Candida albicans infection
E Chlamydia trachomatis infection

A(Correct answ er: A)

Explanation
BACTERIAL VAGINOSIS
Polymicrobial infection characterised by:
• Copious frothy vaginal discharge with a fishy odour
• Increased vaginal pH (>4.5). Normal vaginal pH = 3.8 – 4.5
• Clue cells - vaginal epithelial cells coated with gram-variable bacteria on wet vaginal smear
• Vaginosis - there is minimal inflammation
• Positive amine test - pungent fishy odour on adding the vaginal discharge to sodium hydroxide
• Associated with an alteration in vaginal flora with decreased lactobacilli and increased anaerobes and
Gaednerella vaginalis
• Not sexually transmitted
Treated with metronidazole

Question 16 The normal range of vaginal pH

Options for Questions 16-16

A 2.0 – 3.5 B 3.8 – 4.5


C 4.8 – 5.7 D 5.0 – 7.0
E 7.2 – 8.0

A(Correct answ er: B)

Explanation
BACTERIAL VAGINOSIS
Polymicrobial infection characterised by:
• Copious frothy vaginal discharge with a fishy odour
• Increased vaginal pH (>4.5). Normal vaginal pH = 3.8 – 4.5
• Clue cells - vaginal epithelial cells coated with gram-variable bacteria on wet vaginal smear
• Vaginosis - there is minimal inflammation
• Positive amine test - pungent fishy odour on adding the vaginal discharge to sodium hydroxide
• Associated with an alteration in vaginal flora with decreased lactobacilli and increased anaerobes and
Gaednerella vaginalis
• Not sexually transmitted
• Treated with metronidazole

This Copy is for Dr. Mohamed ElHodiby


Question 17 With respect to the pH of vaginal fluid

Options for Questions 17-17

Vaginal pH is 3.8-4.5 in trichomonal vaginalis


A Vaginal pH is < 3.8 in bacterial vaginosis B
infection
C Vaginal pH is < 3.8 in post-menopausal women D Vaginal pH is > 4.5 in bacterial vaginosis
E Vaginal pH of 7.3 is normal

A(Correct answ er: D)

Explanation
Vaginal pH
• The normal vaginal pH is 3.8 to 4.5
• Bacterial vaginosis, trichomoniasis, and atrophic vaginitis often cause a vaginal pH > 4.5

Question 18 Vaginal infection characterised by the presence strawberry cervis

Options for Questions 18-18

A Bacterial vaginosis B Gardnerella vaginalis infection


C Trichomonas vaginalis infection D Candida albicans infection
E Chlamydia trachomatis infection

A(Correct answ er: C)

Explanation
TRICHOMONAS VAGINALIS
• Flagellated protozoan, anaerobic, readily cultured on Diamond’s medium
• Sexually transmitted
• Infection is different from bacterial vaginosis although clinical differentiation is difficult
• Characteristic strawberry cervix only found in 3% of cases
• Diagnosed by microscopy of wet vaginal smear or by culture
• The organism is difficult to detect in males but can be isolated from prostatic fluid
• Also causes urethritis, cystitis and Bartholinitis
• Vaginal pH > 4.5
• Treated with metronidazole

A 23 year old woman presents with an itchy vaginal discharge at 22 weeks gestation.
Question 19 Microscopy of high vaginal swab shows trichomonas vaginalis. Which one is the most
appropriate treatment?
Options for Questions 19-19

A Azithromycin B Clotrimazole
C Clindamycin D Metronidazole
E Cefalexine

A(Correct answ er: D)

This Copy is for Dr. Mohamed ElHodiby


Explanation
TRICHOMONAS VAGINALIS
• Flagellated protozoan, anaerobic, readily cultured on Diamond’s medium
• Sexually transmitted
• Infection is different from bacterial vaginosis although clinical differentiation is difficult
• Characteristic strawberry cervix only found in 3% of cases
• Diagnosed by microscopy of wet vaginal smear or by culture
• The organism is difficult to detect in males but can be isolated from prostatic fluid
• Also causes urethritis, cystitis and Bartholinitis
• Vaginal pH > 4.5
Treated with metronidazole

Question 20 Trichomonas vaginalis

Options for Questions 20-20

A Spirochete B Gram positive bacteria


C Protozoa D Gram negative bacteria
E Helminth

A(Correct answ er: C)

Explanation
TRICHOMONAS VAGINALIS
• Flagellated protozoan, anaerobic, readily cultured on Diamond’s medium
• Sexually transmitted
• Infection is different from bacterial vaginosis although clinical differentiation is difficult
• Characteristic strawberry cervix only found in 3% of cases
• Diagnosed by microscopy of wet vaginal smear or by culture
• The organism is difficult to detect in males but can be isolated from prostatic fluid
• Also causes urethritis, cystitis and Bartholinitis
• Vaginal pH > 4.5
• Treated with metronidazole

This Copy is for Dr. Mohamed ElHodiby


Five pregnant women (A, B, C, D, E) have been screened for hepatitis B during pregnancy. The
Question 1
results are shown in the table (click icon to view). The hepatitis B status of patient A is
Five pregnant women (A, B, C, D, E) have been screened for hepatitis B during pregnancy. The
Question 2
results are shown in the table (click icon to view). The hepatitis B status of patient E is
Options for Questions 1-2

A Previously vaccinated B Previous infection with immunity


C Chronic carrier D Acute infection
E Susceptible

A(Correct answ er: E)

A(Correct answ er: C)

Explanation
Hepatitis B Serology
• HBsAg detected before HBeAg - both detectable before the onset of jaundice. HBeAg levels fall rapidly
• Anti-HBs antibody detectable at the onset of jaundice and before anti-HBe antibodies
• Anti HBs antibody detected about 4 weeks after the onset of jaundice
• Detection of HBeAg indicates high infectivity
• Detection of anti-HBs IgG indicates previous infection or vaccination
• Chronic carriage associated with persistence of HBsAg
Tests Results Interpretation
Negative
HBsAg
Negative
anti-HBc Susceptible
Negative
IgG anti-HBs

HBsAg Negative
anti-HBc Positive Immune due to natural infection
IgG anti-HBs Positive
HBsAg Negative
anti-HBc Negative Immune due to vaccination
IgG anti-HBs Positive
HBsAg
Positive
anti-HBc
Positive
IgM anti-HBc Acute infection
Positive
IgG anti-HBs
Negative

HBsAg Positive
anti-HBc Positive
Chronic infection
IgM anti-HBc Negative
IgG anti-HBs Negative
Adapted from Centre for Diseases Control & Prevention

A 24 year old woman has been referred to the antenatal clinic at 16 weeks gestation. She is
Question 3
known to be HBs antigen positive and anti-HBs IgG negative

Options for Questions 3-3

This Copy is for Dr. Mohamed ElHodiby


She has previously been vaccinated against Fetal blood sampling should be avoided during
A B
hepatitis B labour
The woman should be vaccinated against
C Caesarean section should be recommended D
hepatitis B after delivery
Hepatitis B viral load should be monitored during
E
pregnancy

A(Correct answ er: B)

Explanation
HbsAg Positive
anti-HBc Positive
IgM anti-HBc Negative
IgG anti-HBs Negative
Chronic carrier.

Aim for vaginal birth but keep membranes intact for as long as possible, avoid FSE and FBS. Neonatal passive
followed by passive and active immunisation

Question 4 Which statement regarding the hepatitis C virus is not true?

Options for Questions 4-4

A RNA virus B Spread by feco-oral route


C Associated with hepatocellular carcinoma D Associated with chronic liver disease
E No treatment is currently available

A(Correct answ er: B)

Explanation
HEPATITIS C
• RNA virus
• Spread by blood products / sexual and vertical transmission
• Incubation period ~2 months
• 50% of patients develop chronic liver disease
• Associated with hepatocellular carcinoma
• treatment with interferon-gamma / Ribavirin may be effective

Question 5 The hepatitis D virus

Options for Questions 5-5

A Is a DNA virus B Is spread by the feco-oral route


C Is not associated with chronic liver disease D Has an incubation period of 10-21 days
E Can be transmitted from mother to fetus

A(Correct answ er: E)

Explanation
HEPATITIS D
• Defective RNA virus

This Copy is for Dr. Mohamed ElHodiby


• Requires co-infection with Hepatitis B for propagation
• Spread by blood products and sexual transmission
• Incubation period 1-3 months
• Associated with chronic carriage and chronic liver disease

Question 6 Terratogenic virus

Options for Questions 6-6

A Hepatitis B B Herpes simplex


C Varicella zoster D Human papilloma virus
E Parvovirus B 19

A(Correct answ er: C)

Explanation
• cella zoster associated with 2% risk of congenital varicella syndrome (dermatomal scarring, cataract, limb hypoplasia,
microcephaly, learning disability..) if infection occurs BEFORE 20 weeks gestation
• Parvovirus B 19 causes fetal anaemia and hydrops

A 17 year old woman attends the antenatal clinic at 20 weeks gestation. She has a history of
Question 7
genital herpes
Options for Questions 7-7

There is an increased risk of congenital


A The virus is an RNA virus B
anomalies
C The virus has an incubation period of 10-21 days D Herpetic lesions may be present on her cervix
In the absence of vesicles or a prodrom, there is
E
no risk to the neonate following vaginal birth

A(Correct answ er: D)

Explanation
HERPES SIMPLEX VIRUS
• DNA virus
• Type I classically causes oro-labial, and type II genital infection .
• However, the prevalence of oro-genital sexual practices means that this distinction is less evident
• Incubation period 3-7 days
• Primary infection may be asymptomatic. Symptoms include genital vesicles which progress into painful ulcers that
eventually crust over. Dysuria, urinary retention, bilateral inguinal lymphadenopathy, systemic illness may occur.
Aseptic meningitis / encephalitis are rare complications
• Primary infection involves cervix in 70% of symptomatic cases, causing purulent discharge and necrotic ulceration
• Virus remains dormant in dorsal root ganglia, causing recurrent infection. Average number of recurrences per year =
4. Less severe symptoms
• Genital infection can be acquired from oro-genital contact.
• Virus can be shed from the genital tract and transmit infection in the absence of symptoms or a prodrome
• Trauma (such as dental treatment, facial peels), exposure to sunlight or ultra-violet light (sun beds) may trigger
recurrent cold sores
• Vertical transmission causes neonatal disseminated herpes which has high mortality / morbidity. Does not cause
congenital anomalies.
Diagnosis

This Copy is for Dr. Mohamed ElHodiby


• Clinical
• Viral culture – negative culture does not exclude infection. The longer the duration of symptoms, the less likely is a
swab to be positive
• Electron microscopy
• ELISA / PCR
• Serology – unreliable cannot distinguish between oro-labial and genital infection
Treatment
• Symptomatic
• Acyclovir
• Prophylactic acyclovir reduces the frequency of recurrence.

Question 8 With respect to HIV infection in pregnancy

Options for Questions 8-8

There is a 3-6 months window between HIV Women booking after 28 weeks should not be
A B
infection and the development of HIV antibodies offered HIV testing
Women presenting in labour with unknown HIV There is an increased risk of congenital
C D
status should not be offered HIV testing anomalies with HIV infection in the first trimester
There is an increased risk of congenital
E anomalies with use of anti-retroviral drugs in the
first trimester

A(Correct answ er: A)

Explanation
Screening for HIV infection in pregnancy
• All pregnant women should be offered HIV testing at booking in every pregnancy
• If HIV testing is declined, this should be documented in the maternity notes and testing offered again at ~ 28 weeks
• HIV negative women who are judged as being at continued high risk of acquiring HIV should be offered repeat testing
later in pregnancy.
• Routine repeat testing to identify the small number of women who sero-convert during pregnancy is not
recommended.
• The results of HIV tests should be clearly documented in the notes and available to all clinicians caring for the
woman. Women should be reassured that their confidentiality will be maintained
• The uptake of HIV screening has increased in England between 2005 and 2009 and is now over 95%
HIV Testing
• HIV testing has traditionally been based on detection of anti-HIV antibodies
• There is a window (3-6 months) between HIV infection and development of antibodies resulting in negative tests in
infected individuals (window period)
• Contemporary HIV tests should detect anti-HIV antibodies and the p24 HIV antigen and reduce the diagnostic
window to ~ 1 month
• Rapid HIV tests can be undertaken with results available within 20 minutes. These tests can be performed using
buccal swabs or finger-prick samples. Most rapid tests only detect anti-HIV antibodies and are more likely to be
negative during the window period. Rapid HIV tests should be offered to women who present in labour with unknown
HIV status
• Women booking at or after 26 weeks should be offered an urgent HIV test with results available within 24h

In HIV positive women treated with highly active anti-retroviral therapy, vaginal birth may be
Question 9
offered if the viral load is
Options for Questions 9-9

This Copy is for Dr. Mohamed ElHodiby


A < 5 copies /ml B < 50 copies / ml
C < 100 copies / ml D < 200 copies / ml
E < 300 copies / ml
A(Correct answ er: B)

Explanation
Vertical transmission of HIV
• The risk of vertical transmission: 15-20% in untreated, non-breast-feeding woman in Europe and 25-40% in untreated
breast-feeding woman in Africa.
• The risk of vertical transmission in women treated with Highly Active Anti-retroviral Therapy (HAART) in Europe is
less than 2%. In women treated with HAART and in whom the viral load is less than 50 copies / ml, the risk of vertical
transmission is less than 1% irrespective of mode of delivery.
The risk factors for vertical transmission in treated women include:
1. High plasma viraemia at delivery
2. Short duration of HAART therapy
3. Delivery before 32 weeks
• Less than 2% of vertical transmission occurs in the first / second trimesters. Over 80% of vertical transmission occurs
around the time of labour and delivery
Maternal factors associated with increased risk of vertical transmission include
1. Advanced maternal HIV disease
2. Low antenatal CD4 count
3. High plasma viral load – this being the strongest predictor
Obstetric risk factors associated with increased risk of vertical transmission include
1. Vaginal delivery
2. Duration of membrane rupture
3. Chorioamnionitis
4. Pre-term delivery
5. Breast-feeding (associated with a 2-fold increase in vertical transmission rate).

Which drug is effective against H1N1 influenza but should not be used in women with severe
Question 10
asthma?
Options for Questions 10-10

A Acyclovir B Oseltamivir
C Zanamivir D Amantadine
E Rimantadine

A(Correct answ er: C)

Explanation
Influenza – Treatment
Symptomatic treatment including paracetamol for fever and as analgesia
Anti-viral agents:
1) Neuraminidase inhibitors: oseltamivir (Tamiflu ®) and zanamivir (Relenza ®)
• Interfere with the release of new viruses from infected cells.
• Most effective if started within a few hours of the onset of symptoms.
• Licensed for use in adults within 48h of the onset of symptoms but may be administered within 7 days of onset.
• Reduce the duration of symptoms by 1-1.5 days and can reduce the risk of complications in high risk patients
Post-exposure prophylaxis: oseltamivir should be given within 48h of exposure while zanamivir should be given within
36h
Drug treatment in pregnancy
• Zanamivir is the preferred drug during pregnancy
• Oseltamivir recommended in severe infection or when zanamivir cannot be used
• Oseltamivir is the preferred drug in women who are breastfeeding

This Copy is for Dr. Mohamed ElHodiby


• Oseltamivir: administered orally. Prodrug, hydrolysed by the liver to active metabolite. Extensively metabolised by the
placenta with minimal fetal drug accumulation. Present in low concentration in breast milk
• Use with caution in renal impairment
• Side-effects: Nausea, vomiting, abdominal pain, diarrhoea, headache, conjunctivitis. Less common side-effects
include eczema, hepatitis, GI bleeding, arrhythmia, visual disturbance
• Zanamivir: administered by inhalation of powder (bioavailability is 10-20% after inhaled dose compared to 2% after
oral dose). 90% of absorbed drug excreted unchanged in the urine. Present in breast milk in low concentration.
• Use with caution in patients with asthma and chronic pulmonary disease because of risk of bronchospasm
• Side-effects: bronchospasm, respiratory impairment, angioedema, rticarial, rash
2) M2 protein inhibitors (amantadine, rimantadine): stop the virus infecting cells. High levels of drug resistance have
been observed and not active against influenza B. No longer recommended.

At which stage of pregnancy are women at greatest risk of severe disease following H1N1
Question 11
influenza infection?
Options for Questions 11-11

A First trimester B Second trimester


C Third trimester D 24-48h post-partum
E 4-6 weeks post-partum

A(Correct answ er: C)

Explanation
H1N1 Influenza and pregnancy
• Pregnant women are more likely to have severe disease, be admitted to ITU and die from H1N1 influenza compared
to non-pregnant women. Most reported deaths occurred in women who developed pneumonia and acute respiratory
distress syndrome requiring mechanical ventilation
• Risk of morbidity from seasonal flu is also higher during pregnancy
• The risk is highest in the third trimester compared to early pregnancy or post-partum
• Women with asthma and morbid obesity appear to be at higher risk of severe disease
• Most current data from UKOSS indicates that pregnant women admitted to ITU were less likely to have been treated
with antiviral agents within 2 days of the onset of symptoms when compared with other pregnant women who were
admitted to hospital. This indicates the importance of prompt treatment
• H1N1 infection is associated with pre-term delivery, especially in women admitted to ITU
• Vertical transmission of H1N1 influenza has not been reported and the virus is not known to be present in breast
milk. Breastfeeding should continue in women being treated for H1N1 influenza
• The H1N1 virus is not terratogenic but there some evidence that pyrexia in early pregnancy may be associated with
an increased risk of neural tube defects

Question 12 Infection with the H1N1 influenza virus

Options for Questions 12-12

Occurring in the third trimester is associated with Occurring in the first trimester is associated with
A B
a 10-25% risk of vertical transmission an increased risk of congenital anomalies
Is associated with an increased risk of pre-term Is associated with an increased risk of pre-
C D
delivery eclampsia
E Causes less severe disease in obese women

A(Correct answ er: C)

Explanation

This Copy is for Dr. Mohamed ElHodiby


H1N1 Influenza and pregnancy
• Pregnant women are more likely to have severe disease, be admitted to ITU and die from H1N1 influenza compared
to non-pregnant women. Most reported deaths occurred in women who developed pneumonia and acute respiratory
distress syndrome requiring mechanical ventilation
• Risk of morbidity from seasonal flu is also higher during pregnancy
• The risk is highest in the third trimester compared to early pregnancy or post-partum
• Women with asthma and morbid obesity appear to be at higher risk of severe disease
• Most current data from UKOSS indicates that pregnant women admitted to ITU were less likely to have been treated
with antiviral agents within 2 days of the onset of symptoms when compared with other pregnant women who were
admitted to hospital. This indicates the importance of prompt treatment
• H1N1 infection is associated with pre-term delivery, especially in women admitted to ITU
• Vertical transmission of H1N1 influenza has not been reported and the virus is not known to be present in breast
milk. Breastfeeding should continue in women being treated for H1N1 influenza
• The H1N1 virus is not terratogenic but there some evidence that pyrexia in early pregnancy may be associated with
an increased risk of neural tube defects

A 32 year old woman is admitted to hospital at 35 weeks gestation during an H1N1 influenza
Question 13 pandemic. She has a temperature of 39C. Which one is not consistent with a clinical diagnosis
of H1N1 influenza?
Options for Questions 13-13

A She has headache and diarrhoea B She has a cough and widespread muscle aches
C She has diarrhoea and vomiting D She has loin pain and vomiting
E She has a sore throat and diarrhoea

A(Correct answ er: D)

Explanation
Clinical diagnosis of influenza in pregnancy
During a pandemic, clinical diagnosis can be made based on the following:
1. Fever > 38C plus at least 2 of the following symptoms:
• Widespread muscle and joint aches
• Cough
• Headache
• Blocked or runny nose
• Sore throat
• Vomiting
• Diarrhoea
The following are features of severe disease:
• Above symptoms / signs PLUS any of the following:
• Tachypnoea (respiratory rate > 30 / min)
• Dyspnoea
• Hypoxia (SO2 < 92%)
• Persistent tachycardia > 100 / min
• Chest pain ob breathing
• Dehydration & shock
• Rigors
• Purulent or blood-stained sputum

Question 14 Group A streptococci

Options for Questions 14-14

This Copy is for Dr. Mohamed ElHodiby


A Strep. Viridans B Strep. Pneumonia
C Strep. Algalactiae D Strep. Fecalis
E Strep. Pyogenes

A(Correct answ er: E)

Explanation
STREPTOCOCCI
• Gram positive
• Capsulated, aerobic
• May form chains (S. viridans) or diplococci (S. pneumoniae)
• Grow on blood agar - may cause haemolysis

Alpha-Haemolytic
• Green colour around each colony due to altered haemoglobin
• S. viridans and S. pneumoniae
• Differentiated by Optochin disc test (S. pneumoniae shows zone of inhibition) and bile solubility test (S.
pneumoniae is soluble)
• S. pneumoniae isolated from nose / throat in 70% of healthy individuals
• S. viridans - isolated from the mouth, causes bacterial endocarditis

Beta-Haemolytic
• Complete lysis of red cells
• S. pyogenes, S. agalactiae, S. fecalis
• Differentiated by Lancefield grouping and Bacitracin test
• Lancefield group A: S. pyogenes- causes sore throat, skin and wound infections (erysipelas, impetigo),
rheumatic fever and acute glomerulonephritis. Strains causing scarlet fever produce an exotoxin
• Lancefield group B: S. algalactiae - isolated from vagina, perineum and rectum; causes neonatal
septicaemia and meningitis
• Lancefield group D: S. fecalis - usually non-haemolytic. Isolated from gut, causes UTIs and bacterial
endocarditis

Gamma Haemolytic
• Non haemolytic
• S. fecalis
• Streptococci are characteristically sensitive to penicillin / erythromycin

Question 15 Causes Rheumatic fever

Options for Questions 15-15

A Strep. Viridans B Strep. Pneumonia


C Strep. Algalactiae D Strep. Fecalis
E Strep. Pyogenes

A(Correct answ er: E)

Explanation
STREPTOCOCCI
• Gram positive
• Capsulated, aerobic
• May form chains (S. viridans) or diplococci (S. pneumoniae)
• Grow on blood agar - may cause haemolysis

Alpha-Haemolytic
• Green colour around each colony due to altered haemoglobin
• S. viridans and S. pneumoniae

This Copy is for Dr. Mohamed ElHodiby


• Differentiated by Optochin disc test (S. pneumoniae shows zone of inhibition) and bile solubility test (S.
pneumoniae is soluble)
• S. pneumoniae isolated from nose / throat in 70% of healthy individuals
• S. viridans - isolated from the mouth, causes bacterial endocarditis

Beta-Haemolytic
• Complete lysis of red cells
• S. pyogenes, S. agalactiae, S. fecalis
• Differentiated by Lancefield grouping and Bacitracin test
• Lancefield group A: S. pyogenes- causes sore throat, skin and wound infections (erysipelas, impetigo),
rheumatic fever and acute glomerulonephritis. Strains causing scarlet fever produce an exotoxin
• Lancefield group B: S. algalactiae - isolated from vagina, perineum and rectum; causes neonatal
septicaemia and meningitis
• Lancefield group D: S. fecalis - usually non-haemolytic. Isolated from gut, causes UTIs and bacterial
endocarditis

Gamma Haemolytic
• Non haemolytic
• S. fecalis
Streptococci are characteristically sensitive to penicillin / erythromycin

Question 16 Actinomycete israelii

Options for Questions 16-16

A Is part of the normal flora in the mouth B Is a common cause of vaginal discharge
Causes genital tract infection associated with the
C Is an aerobic organism D
use of tampons
Causes infection that spreads through the
E
lymphatic system

A(Correct answ er: A)

Explanation
ACTINOMYCETE ISRAELII
• Gram positive branching filamentous bacillus
• Anaerobic
• Form part of the normal flora in the mouth, vagina and rectum.
• Non-acid -fast (Nocardia are weakly acid-fast)
• Infection is characterised by sulphur (yellow) granules in pus
• Does not involve the lymphatics
• Causes cervico-facial, pulmonary and abdominal infection, causing multiple discharging sinuses
• Genital tract infection associated with the IUCD
• Sensitive to penicillin

Question 17 Which organism typically causes infection following treatment with broad-spectrum antibiotics?

Options for Questions 17-17

A E. Coli B Staph. Epidermidis


C Strep. Fecalis D C. Difficile
E A. Israelii

A(Correct answ er: D)

Explanation

This Copy is for Dr. Mohamed ElHodiby


CLOSTRIDIA
• Gram positive anaerobic bacilli
• Spore-forming -the spores are resistant to boiling and chemical disinfectants
• Commonly found in the environment and as part of normal fecal flora
• Produce EXOTOXINS
• Infection common in gangrenous / ischaemic tissue and in association with foreign bodies
• Sensitive to metronidazole and penicillin
• CLOSTRIDIUN DIFFICILE - causes pseudo-membranous colitis - diarrhoea with collapse following broad
spectrum antibiotic treatment - treated with oral metronidazole or vancomycin
• CLOSTRIDIUM PERFRINGENS - Causes gas gangrene (causing crepitus) and septic abortion (rare)
• Produces exotoxin causing septic shock, jaundice, haemolysis and acute renal failure
Sensitive to penicillin - surgical debridement usually required

Question 18 Which organism is not a spirochaete?

Options for Questions 18-18

A Treponema pallidum B Leptospira icterohaemorrhagica


C Chlamydia trachomatis D Borrelia vincenti
E Treponema pertenue

A(Correct answ er: C)

Explanation
SPIROCHAETES
• Thin-walled spiralled flexible motile organisms
• Not seen on gram stain except Borrelia vincenti
• Seen on dark ground microscopy or silver stain
• Include Treponema (T. pallidum - syphilis, T. pertenue - Yaws, T. carateum - Pinta), Leptospira (L.
icterohaemorrhagica – Weil’s disease) and Borrelia (B. recurrentis - relapsing fever, B. vincenti – Vincent’s infection)

Question 19 With respect to serological tests for syphilis

Options for Questions 19-19

The VDRL test becomes positive before the


A B The VDRL test remains positive after treatment
fluorescent treponemal antibody test
The fluorescent treponemal antibody test is used The VDRL test is more specific than the
C D
to monitor treatment treponemal antibody test
The fluorescent treponemal antibody test can
E
remain positive for several years after treatment

A(Correct answ er: E)

Explanation
DIAGNOSIS OF SYPHILIS
• Cardiolipin antibody tests(such as Wassermann reaction, rapid plasma regain test and VDRL - Venereal
Disease Reference Laboratory) which are relatively non-specific
• Treponemal antibody testssuch as the Fluorescent Treponemal Antibody test -Absorbed (FTA-Abs) which
are specific and used as verification tests
• Cardiolipin antibody tests are positive in the serum in untreated secondary, latent and tertiary syphilis and
may be false positive
• Cardiolipin antibody tests become negative with treatment
• Primary: VDRL (+ve or -ve), FTA-Abs (+ve). FTA-Abs usually becomes positive before VDRL
• Secondary : VDRL and FTA-Abs (+ve)

This Copy is for Dr. Mohamed ElHodiby


• Latent : VDRL (usually +ve), FTA-Abs (+ve)
• Tertiary / quaternary : VDRL (+ve or -ve), FTA-Abs (+ve)
• Treated : VDRL (-ve), FTA-Abs (+ve)
• The VDRL test becomes negative with treatment and can be used to monitor treatment. The FTA-Abs test
can remain positive for several years after adequate treatment

Question 20 Which antibiotic is used to treat syphilis infection in the first trimester of pregnancy?

Options for Questions 20-20

A Azithromycin B Erythromycin
C Procaine penicillin D Clindamycin
E Tazocin

A(Correct answ er: C)

Explanation
Treatment of syphilis
• Procaine penicillin is first line treatment
• Treponema pallidum is also sensitive to erythromycin and tetracycline although the results of treatment are less
satisfactory
• The Jerisch-Herxheimer reaction is an inflammatory response to the increased release of treponemal antigens during
treatment -managed with steroids.

Question 21 Which infection is not caused by Chlamydia trachomatis?

Options for Questions 21-21

A Salpingitis B Conjunctivitis
C Bartholinitis D Interstitial pneumonitis
E Pharyngitis
A(Correct answ er: E)

Explanation
CHLAMYDIA TRACHOMATIS
• Obligate intracellular organism - bacteria
• Non-motile gram negative or gram variable although gram stain is not useful in diagnosis
• Enter the host through abrasions in mucosal surfaces
• Does not infect squamous cells - the vaginal epithelium and ectocervix are not infected. Endocervical glands are
susceptible
• Produce infectious particles called Elementary Bodies which enter cells by endocytosis
• Differentiate into Reticulate Bodies within cells. These multiply by binary fission and then produce other elementary
bodies which are released from the cell
• Asymptomatic infection is common
• C. psittaci infects birds and non-human animals
Various serovars:
• A - C: Trachoma (blindness)
• D - K: genital infection - cervicitis, urethritis, salpingitis, epididymitis, proctitis, Bartholinitis, conjunctivitis, interstitial
pneumonitis in neonates.
• Consequences of infection include infertility, ectopic pregnancy, Fitz-Hugh-Curtis syndrome, Reiter's syndrome,
arthritis, dermatitis
• L1-3: Lymphogranuloma venereum

This Copy is for Dr. Mohamed ElHodiby


A 23 year old woman with Neisseria gonorrhoea cervicitis develops aseptic arthritis and
Question 22
conjunctivitis
Options for Questions 22-22

A Sjogren syndrome B Sicca syndrome


C Reiter’s syndrome D Fitz-Hugh-Curtis syndrome
E Jarisch-Herxheimer reaction

A(Correct answ er: C)

Explanation
REITER’S SYNDROME
• Arthritis +/- conjunctivitis following GI or GU infection
• Commoner in males
• 50% associated with Chlamydia infection
• Arthritis is reactive, aseptic and sero-negative
• Abnormal response to the infection in genetically predisposed individuals

Question 23 The normal range of vaginal pH

Options for Questions 23-23

A 2.0 – 3.5 B 3.8 – 4.5


C 4.8 – 5.7 D 5.0 – 7.0
E 7.2 – 8.0

A(Correct answ er: B)

Explanation
BACTERIAL VAGINOSIS
Polymicrobial infection characterised by:
• Copious frothy vaginal discharge with a fishy odour
• Increased vaginal pH (>4.5). Normal vaginal pH = 3.8 – 4.5
• Clue cells - vaginal epithelial cells coated with gram-variable bacteria on wet vaginal smear
• Vaginosis - there is minimal inflammation
• Positive amine test - pungent fishy odour on adding the vaginal discharge to sodium hydroxide
• Associated with an alteration in vaginal flora with decreased lactobacilli and increased anaerobes and
Gaednerella vaginalis
• Not sexually transmitted
• Treated with metronidazole

Question 24 Vaginal infection characterised by the presence strawberry cervis

Options for Questions 24-24

A Bacterial vaginosis B Gardnerella vaginalis infection


C Trichomonas vaginalis infection D Candida albicans infection
E Chlamydia trachomatis infection

This Copy is for Dr. Mohamed ElHodiby


A(Correct answ er: C)

Explanation
TRICHOMONAS VAGINALIS
• Flagellated protozoan, anaerobic, readily cultured on Diamond’s medium
• Sexually transmitted
• Infection is different from bacterial vaginosis although clinical differentiation is difficult
• Characteristic strawberry cervix only found in 3% of cases
• Diagnosed by microscopy of wet vaginal smear or by culture
• The organism is difficult to detect in males but can be isolated from prostatic fluid
• Also causes urethritis, cystitis and Bartholinitis
• Vaginal pH > 4.5
• Treated with metronidazole

Question 25 Which organism typically causes opportunistic infections?

Options for Questions 25-25

A E. Coli B P. Aeruginosa
C S. Typhi D P. Mirabilis
E N. Meningitides

A(Correct answ er: B)

Explanation
PSEUDOMONAS AERUGINOSA
• Gram negative bacilli, indistinguishable from coliforms on gram staining
• Strict aerobes
• The bacterium is ubiquitous in soil and water, and on surfaces in contact with soil or water. They occur
regularly on the surfaces of plants and occasionally on the surfaces of animals.
• Its metabolism is respiratory and never fermentative
• Almost all strains are motile by means of a single polar flagellum
• The bacterium almost never infects uncompromised tissues, yet there is hardly any tissue that it cannot
infect if the tissue defenses are compromised in some manner
• P. aeruginosa strains produce two types of soluble pigments, the fluorescent pigment pyoverdin and the
blue pigment pyocyanin. The latter is produced abundantly in media of low-iron content and functions in iron
metabolism in the bacterium
• Grow well on selective agar containing the disinfectant cetrimide
• Strongly oxidase positive (coliforms are oxidase negative)
• Metabolises glucose by oxidation
• Not part of normal gut flora
• Causes chronic UTIs, wound infections, chronic osteomyelitis and opportunistic infections
• Resistant to ampicillin, trimethoprim, tetracycline, sulphonamides
• Sensitive to aminoglycosides gentamicin and netilmicin
• Sensitive to anti-pseudomonas penicillins carbenicillin, ticarcillin, piperacillin
Most are sensitive to ceftazidime, ciprofloxacin and imipenam

A 34 year old woman is found to have a pseudomonas aeruginosa urinary tract infection. Which
Question 26
antibiotic is unsuitable for treatment?
Options for Questions 26-26

A Gentamicin B Ciprofloxacin
C Carbenicillin D Ceftazidime

This Copy is for Dr. Mohamed ElHodiby


E Ampicillin

A(Correct answ er: E)

Explanation
PSEUDOMONAS AERUGINOSA
• Gram negative bacilli, indistinguishable from coliforms on gram staining
• Strict aerobes
• The bacterium is ubiquitous in soil and water, and on surfaces in contact with soil or water. They occur
regularly on the surfaces of plants and occasionally on the surfaces of animals.
• Its metabolism is respiratory and never fermentative
• Almost all strains are motile by means of a single polar flagellum
• The bacterium almost never infects uncompromised tissues, yet there is hardly any tissue that it cannot
infect if the tissue defenses are compromised in some manner
• P. aeruginosa strains produce two types of soluble pigments, the fluorescent pigment pyoverdin and the
blue pigment pyocyanin. The latter is produced abundantly in media of low-iron content and functions in iron
metabolism in the bacterium
• Grow well on selective agar containing the disinfectant cetrimide
• Strongly oxidase positive (coliforms are oxidase negative)
• Metabolises glucose by oxidation
• Not part of normal gut flora
• Causes chronic UTIs, wound infections, chronic osteomyelitis and opportunistic infections
• Resistant to ampicillin, trimethoprim, tetracycline, sulphonamides
• Sensitive to aminoglycosides gentamicin and netilmicin
• Sensitive to anti-pseudomonas penicillins carbenicillin, ticarcillin, piperacillin
Most are sensitive to ceftazidime, ciprofloxacin and imipenam

Question 27 Which infection is typically contracted following contact with cat litter?

Options for Questions 27-27

A Plasmodium falciparum B Listeria monocytogenes


C Toxoplasma gondii D Treponema pallidum
E Bacteriodes anthracis

A(Correct answ er: C)

Explanation
TOXOPLASMA GONDII
• Obligate intracellular protozoan
• Completes part of its life cycle in animals (mainly cats but also sheep, cattle and pigs)
• Contracted by contact with cat litter, infected soil or eating undercooked beef, lamb or pork. Prevented by
wearing gloves during gardening, thorough food hygiene and avoiding contact with cat litter
• Oocysts or tissue cysts invades the intestinal epithelium , develop into trophozoites which spread via the
lymphatic system
• Infection is usually mild or asymptomatic with lymphadenopathy (posterior cervical) being the main
presentation. Causes severe disease with encephalitis and chorioretinitis in the immunocompromised
• Risk of fetal infection increases with increasing gestation age (17% first, 25% second and 65% third
trimester)
• Risk of fetus being affected FALLS with increasing gestation age
• Recognised cause of congenital anomalies with chorioretinitis, intra-cranial calcification, microcephaly,
hydrocephalus. Increased risk of miscarriage with first trimester infection
• Diagnosis can be made by culture in laboratory rodents
• Detection of IgM or a 4 fold rise in IgG titres in samples 3 weeks apart is indicative of acute infection
Sensitive to spiramycin and pyramethamine

This Copy is for Dr. Mohamed ElHodiby


Question 28 Which statement regarding clindamycin is not true?

Options for Questions 28-28

A It acts by inhibiting bacterial protein synthesis B It is mainly excreted by the liver


C It is effective against Clostridium difficile D It can be administered orally
E It is effective against anaerobic bacteria

A(Correct answ er: C)

Explanation
CLINDAMYCIN
• Clindamycin belongs to the group of Lincosamides
• Binds to 50S ribosomal subunit, inhibiting peptide bond formation.
• Similar antibiotic spectrum to erythromycin, excreted by the liver, active in faeces for up to 5 days after a
dose
• More active against anaerobes than erythromycin. Clostridium difficile is resistant and may be selected,
causing pseudomembranous colitis

Question 29 Chlorhexidine (2-4%) used for surgical disinfection

Options for Questions 29-29

A Is unsuitable for use as surgical scrub B Is toxic to eyes and ears


Is usually combined with iodine to improve
C Is unsuitable for use on mucous membranes D
efficacy
E Has no persistent activity once the skin is dry

A(Correct answ er: B)

Explanation
Chlorhexidine (2-4%)
• Excellent activity against gram positive bacteria but less effective against most gram negative bacteria
• Excellent activity against viruses
• Poor activity against TB and fungi
• No activity against endospores
• Activity is only slightly affected by organic matter
• Useful as surgical scrub and skin prep – has high persistent activity
• Suitable for use on mucous membranes
• Toxic to ears and eyes
• Combination with detergents highly effective for hand disinfection

Question 30 Which disinfectant may be used to prepare the vagina prior to vaginal hysterectomy?

Options for Questions 30-30

A Iodine (3%) solution B Povidone-iodine solution

This Copy is for Dr. Mohamed ElHodiby


C Isopropyl alcohol 70% D Chlorhexidine 2% + alcohol 70%
E Chlorhexidine 0.2%

A(Correct answ er: B)

Explanation
Iodophores (7.5 – 10%)
• Preparation containing iodine complexed with a solubilizing agent, such as a surfactant or povidone (povidone-iodine)
• Result in a solution that releases free iodine
• Excellent activity against gram positive and most gram negative bacteria
• Fair activity against TB
• No activity against endospores
• Good activity against viruses and fungi
• Activity is moderately affected by organic matter
• Useful as surgical scrub and skin prep
• Can be used on mucous membranes
• Iodine may be absorbed by the neonate and affect thyroid function

This Copy is for Dr. Mohamed ElHodiby


Five pregnant women (A, B, C, D, E) have been screened for hepatitis B during pregnancy. The
Question 1
results are shown in the table (click icon to view). The hepatitis B status of patient B is
Five pregnant women (A, B, C, D, E) have been screened for hepatitis B during pregnancy. The
Question 2
results are shown in the table (click icon to view). The hepatitis B status of patient D is
Options for Questions 1-2

A Previously vaccinated B Previous infection with immunity


C Chronic carrier D Acute infection
E Susceptible

A(Correct answ er: B)

A(Correct answ er: D)

Explanation
Tests Results Interpretation
Negative
HBsAg
Negative
anti-HBc Susceptible
Negative
IgG anti-HBs

HBsAg Negative
anti-HBc Positive Immune due to natural infection
IgG anti-HBs Positive
HBsAg Negative
anti-HBc Negative Immune due to vaccination
IgG anti-HBs Positive
HBsAg
Positive
anti-HBc
Positive
IgM anti-HBc Acute infection
Positive
IgG anti-HBs
Negative

HBsAg Positive
anti-HBc Positive
Chronic infection
IgM anti-HBc Negative
IgG anti-HBs Negative

Question 3 Hepatitis A virus

Options for Questions 3-3

A Is a DNA virus B Is associated with chronic liver disease


Can be transmitted from mother to fetus during Infection can be prevented using an inactivated
C D
child birth virus vaccine
E Has an incubation period of 3-7 weeks

A(Correct answ er: D)

This Copy is for Dr. Mohamed ElHodiby


Explanation
HEPATITIS A
• RNA virus
• Spread by the feco-oral route although sexual transmission may occur
• Not vertically transmissible
• Incubation period 2-3 weeks
• Does not result in chronic carriage or chronic liver disease
• 20-50% of adults in UK have been infected
• formaldehyde inactivated vaccine is available

Question 4 Which virus is typically spread by the feco-oral route?

Options for Questions 4-4

A Hepatitis A B Hepatitis B
C Hepatitis C D Hepatitis D
E Herpes simplex type I

A(Correct answ er: A)

Explanation
• Hepatitis – feco-oral
• Hepatitis – blood & bodily fluids / sexually transmitted
• Hepatitis C – blood & bodily fluids / sexually transmitted
• Hepatitis D – blood & bodily fluids / sexually transmitted
• Herpes simplex type I – direct bodily contact

Question 5 The Hepatitis B virus

Options for Questions 5-5

A Is an RNA virus B Has an incubation period of 10-21 days


Infection can be prevented by passive
C Multiplies in hepatocytes and erythrocytes D
immunisation
Vertical transmission is associated with a 10%
E
risk of chronic carriage

A(Correct answ er: D)

Explanation
HEPATITIS B
• DNA virus
• Spread by sexual intercourse / blood products
• Incubation period 1-5 months
• Multiplies only in the liver
• Vertically transmissible - 90% of vertically acquired infection result in chronic carriage –
preventable by passive + active immunisation
• 10% of horizontally acquired infections result in chronic carriage
• There is no evidence that mosquitoes can transmit hepatitis B
• 60-70% of acute hepatitis B infections are sub-clinical or associated with mild flu-like illness. Severity of symptoms is
similar in pregnancy

This Copy is for Dr. Mohamed ElHodiby


• Chronic infection associated with hepatocellular carcinoma
• Horizontal and vertical infection preventable by active and passive immunisation

Question 6 Which statement regarding the hepatitis C virus is not true?

Options for Questions 6-6

A RNA virus B Spread by feco-oral route


C Associated with hepatocellular carcinoma D Associated with chronic liver disease
E No treatment is currently available
A(Correct answ er: B)

Explanation
HEPATITIS C
• RNA virus
• Spread by blood products / sexual and vertical transmission
• Incubation period ~2 months
• 50% of patients develop chronic liver disease
• Associated with hepatocellular carcinoma
• treatment with interferon-gamma / Ribavirin may be effective

A 17 year old woman attends the antenatal clinic at 20 weeks gestation. She has a history of
Question 7
genital herpes
Options for Questions 7-7

There is an increased risk of congenital


A The virus is an RNA virus B
anomalies
C The virus has an incubation period of 10-21 days D Herpetic lesions may be present on her cervix
In the absence of vesicles or a prodrom, there is
E
no risk to the neonate following vaginal birth

A(Correct answ er: D)

Explanation
HERPES SIMPLEX VIRUS
• DNA virus
• Type I classically causes oro-labial, and type II genital infection .
• However, the prevalence of oro-genital sexual practices means that this distinction is less evident
• Incubation period 3-7 days
• Primary infection may be asymptomatic. Symptoms include genital vesicles which progress into painful ulcers that
eventually crust over. Dysuria, urinary retention, bilateral inguinal lymphadenopathy, systemic illness may occur.
Aseptic meningitis / encephalitis are rare complications
• Primary infection involves cervix in 70% of symptomatic cases, causing purulent discharge and necrotic ulceration
• Virus remains dormant in dorsal root ganglia, causing recurrent infection. Average number of recurrences per year =
4. Less severe symptoms
• Genital infection can be acquired from oro-genital contact.
• Virus can be shed from the genital tract and transmit infection in the absence of symptoms or a prodrome
• Trauma (such as dental treatment, facial peels), exposure to sunlight or ultra-violet light (sun beds) may trigger
recurrent cold sores

This Copy is for Dr. Mohamed ElHodiby


• Vertical transmission causes neonatal disseminated herpes which has high mortality / morbidity. Does not cause
congenital anomalies.
Diagnosis
• Clinical
• Viral culture – negative culture does not exclude infection. The longer the duration of symptoms, the less likely is a
swab to be positive
• Electron microscopy
• ELISA / PCR
• Serology – unreliable cannot distinguish between oro-labial and genital infection
Treatment
• Symptomatic
• Acyclovir
• Prophylactic acyclovir reduces the frequency of recurrence.

Question 8 With respect to screening for HIV infection in pregnancy in the UK, which statement is not true?

Options for Questions 8-8

HIV testing is offered to all pregnant women If a woman declines HIV testing, this should be
A B
routinely documented in her antenatal notes
The HIV test results of HIV positive women
HIV tests detect both the HIV antibody and the
C should not be documented in their hand-held D
p24 HIV antigen
notes
E HIV test can be undertaken using buccal swab

A(Correct answ er: C)

Explanation
Screening for HIV infection in pregnancy
• All pregnant women should be offered HIV testing at booking in every pregnancy
• If HIV testing is declined, this should be documented in the maternity notes and testing offered again at ~ 28 weeks
• HIV negative women who are judged as being at continued high risk of acquiring HIV should be offered repeat testing
later in pregnancy.
• Routine repeat testing to identify the small number of women who sero-convert during pregnancy is not
recommended.
• The results of HIV tests should be clearly documented in the notes and available to all clinicians caring for the
woman. Women should be reassured that their confidentiality will be maintained
• The uptake of HIV screening has increased in England between 2005 and 2009 and is now over 95%
HIV Testing
• HIV testing has traditionally been based on detection of anti-HIV antibodies
• There is a window (3-6 months) between HIV infection and development of antibodies resulting in negative tests in
infected individuals (window period)
• Contemporary HIV tests should detect anti-HIV antibodies and the p24 HIV antigen and reduce the diagnostic
window to ~ 1 month
• Rapid HIV tests can be undertaken with results available within 20 minutes. These tests can be performed using
buccal swabs or finger-prick samples. Most rapid tests only detect anti-HIV antibodies and are more likely to be
negative during the window period. Rapid HIV tests should be offered to women who present in labour with unknown
HIV status
• Women booking at or after 26 weeks should be offered an urgent HIV test with results available within 24h.

This Copy is for Dr. Mohamed ElHodiby


Question 9 The risk of vertical transmission of HIV in untreated non-breastfeeding women in Europe is

Options for Questions 9-9

A 1-2% B 5-10%
C 15-20% D 20-25%
E 30-35%

A(Correct answ er: C)

Explanation
Vertical transmission of HIV
• The risk of vertical transmission: 15-20% in untreated, non-breast-feeding woman in Europe and 25-40% in untreated
breast-feeding woman in Africa.
• The risk of vertical transmission in women treated with Highly Active Anti-retroviral Therapy (HAART) in Europe is
less than 2%. In women treated with HAART and in whom the viral load is less than 50 copies / ml, the risk of vertical
transmission is less than 1% irrespective of mode of delivery.
The risk factors for vertical transmission in treated women include:
1. High plasma viraemia at delivery
2. Short duration of HAART therapy
3. Delivery before 32 weeks
• Less than 2% of vertical transmission occurs in the first / second trimesters. Over 80% of vertical transmission occurs
around the time of labour and delivery
Maternal factors associated with increased risk of vertical transmission include
1. Advanced maternal HIV disease
2. Low antenatal CD4 count
3. High plasma viral load – this being the strongest predictor
Obstetric risk factors associated with increased risk of vertical transmission include
1. Vaginal delivery
2. Duration of membrane rupture
3. Chorioamnionitis
4. Pre-term delivery
5. Breast-feeding (associated with a 2-fold increase in vertical transmission rate).

In HIV positive women treated with highly active anti-retroviral therapy, vaginal birth may be
Question 10
offered if the viral load is
Options for Questions 10-10

A < 5 copies /ml B < 50 copies / ml


C < 100 copies / ml D < 200 copies / ml
E < 300 copies / ml

A(Correct answ er: B)

Explanation
rtical transmission of HIV
• The risk of vertical transmission: 15-20% in untreated, non-breast-feeding woman in Europe and 25-40% in untreated
breast-feeding woman in Africa.
• The risk of vertical transmission in women treated with Highly Active Anti-retroviral Therapy (HAART) in Europe is
less than 2%. In women treated with HAART and in whom the viral load is less than 50 copies / ml, the risk of vertical
transmission is less than 1% irrespective of mode of delivery.
The risk factors for vertical transmission in treated women include:
1. High plasma viraemia at delivery
2. Short duration of HAART therapy

This Copy is for Dr. Mohamed ElHodiby


3. Delivery before 32 weeks
• Less than 2% of vertical transmission occurs in the first / second trimesters. Over 80% of vertical transmission occurs
around the time of labour and delivery
Maternal factors associated with increased risk of vertical transmission include
1. Advanced maternal HIV disease
2. Low antenatal CD4 count
3. High plasma viral load – this being the strongest predictor
Obstetric risk factors associated with increased risk of vertical transmission include
1. Vaginal delivery
2. Duration of membrane rupture
3. Chorioamnionitis
4. Pre-term delivery
5. Breast-feeding (associated with a 2-fold increase in vertical transmission rate).

Question 11 The influenza virus

Options for Questions 11-11

A Is a DNA virus B Is resistant to detergents


Is resistant to standard disinfectants used in
C D Is inactivated by sunlight
hospitals
E Only causes disease in mammals
A(Correct answ er: D)

Explanation
Influenza and ‘Swine Flu’
Influenza virus
• RNA virus
• Family Orthomyxoviridae – different from the para-influenza viruses (Family: Paramyxoviridae) which commonly
cause respiratory tract infections such as croup in children
• Infect birds and mammals
• Transmitted by aerosols and contact with contaminated surfaces and bird droppings
• Inactivated by sun light, disinfectants and detergents (soap)
• New strains of the virus spread to humans from other species, causing pandemics

Question 12 With respect to the influenza virus

Options for Questions 12-12

A Infection does not stimulate antibody production B The viral envelope contains RNA
The viral envelope does not contain
C D The virus only replicates in mature red blood cells
haemagglutinin
E The viral envelope contains neuraminidase

A(Correct answ er: E)

Explanation
Influenza Virus structure
• Viral envelope containing two glycoproteins, wrapped around a central core
• Envelope contains Hemagglutinin (HA) and Neuraminidase (NA)
• Central core contains the viral RNA genome. RNA may be double-stranded
• Hemagglutinin is involved in viral binding to target cells: binds to sialic acid residues on the surface of epithelial cells

This Copy is for Dr. Mohamed ElHodiby


• Neuraminidase is involved in the release of new viruses from infected cells. Drugs that inhibit neuraminidase
(oseltamivir) prevent the release of new infectious viruses and halt viral replication
• Both proteins are antigenic, stimulating antibody production. They are also targets for drug treatment

Question 13 The H1N1 virus

Options for Questions 13-13

A Is an influenza B virus B Is transmitted from pigs to humans


C Undergoes antigenic shift D Does not undergo antigenic drift
E Infection does not result in antibody production

A(Correct answ er: C)

Explanation
Classification of influenza viruses
Three main genera: Influenza A, B & C
Influenza A
• Most pathogenic influenza virus in humans
• Undergoes antigenic shift and antigenic drift, resulting in the production of new variants that evade host immunity with
the potential for pandemics
• Divided into several serotypes based on the antibody binding to the Hemagglutinin and Neuraminidase proteins:
1. H1N1 – Caused 2009 pandemic (Swine flu virus). This virus is not endemic in pigs and is not transmitted from pigs to
humans. It is a recombination of several strains of H1N1 virus found in humans, birds and pigs
2. H2N2 and H3N2 have caused pandemics in Asia
3. H5N1 – Causes highly lethal pneumonia

Question 14 Which drug should be used to treat pregnant women infected wth H1N1 influenza?

Options for Questions 14-14

A Acyclovir B Oseltamivir
C Zanamivir D Amantadine
E Rimantadine

A(Correct answ er: C)

Explanation
Influenza – Treatment
Symptomatic treatment including paracetamol for fever and as analgesia
Anti-viral agents:
1) Neuraminidase inhibitors: oseltamivir (Tamiflu ®) and zanamivir (Relenza ®)
• Interfere with the release of new viruses from infected cells.
• Most effective if started within a few hours of the onset of symptoms.
• Licensed for use in adults within 48h of the onset of symptoms but may be administered within 7 days of onset.
• Reduce the duration of symptoms by 1-1.5 days and can reduce the risk of complications in high risk patients
Post-exposure prophylaxis: oseltamivir should be given within 48h of exposure while zanamivir should be given within
36h
Drug treatment in pregnancy
• Zanamivir is the preferred drug during pregnancy
• Oseltamivir recommended in severe infection or when zanamivir cannot be used
• Oseltamivir is the preferred drug in women who are breastfeeding

This Copy is for Dr. Mohamed ElHodiby


• Oseltamivir: administered orally. Prodrug, hydrolysed by the liver to active metabolite. Extensively metabolised by the
placenta with minimal fetal drug accumulation. Present in low concentration in breast milk
• Use with caution in renal impairment
• Side-effects: Nausea, vomiting, abdominal pain, diarrhoea, headache, conjunctivitis. Less common side-effects
include eczema, hepatitis, GI bleeding, arrhythmia, visual disturbance
• Zanamivir: administered by inhalation of powder (bioavailability is 10-20% after inhaled dose compared to 2% after
oral dose). 90% of absorbed drug excreted unchanged in the urine. Present in breast milk in low concentration.
• Use with caution in patients with asthma and chronic pulmonary disease because of risk of bronchospasm
• Side-effects: bronchospasm, respiratory impairment, angioedema, rticarial, rash
2) M2 protein inhibitors (amantadine, rimantadine): stop the virus infecting cells. High levels of drug resistance have
been observed and not active against influenza B. No longer recommended.

Question 15 Which drug is administered orally to treat H1N1 influenza infection?

Options for Questions 15-15

A Acyclovir B Oseltamivir
C Zanamivir D Amantadine
E Rimantadine

A(Correct answ er: B)

Explanation
Influenza – Treatment
Symptomatic treatment including paracetamol for fever and as analgesia
Anti-viral agents:
1) Neuraminidase inhibitors: oseltamivir (Tamiflu ®) and zanamivir (Relenza ®)
• Interfere with the release of new viruses from infected cells.
• Most effective if started within a few hours of the onset of symptoms.
• Licensed for use in adults within 48h of the onset of symptoms but may be administered within 7 days of onset.
• Reduce the duration of symptoms by 1-1.5 days and can reduce the risk of complications in high risk patients
Post-exposure prophylaxis: oseltamivir should be given within 48h of exposure while zanamivir should be given within
36h
Drug treatment in pregnancy
• Zanamivir is the preferred drug during pregnancy
• Oseltamivir recommended in severe infection or when zanamivir cannot be used
• Oseltamivir is the preferred drug in women who are breastfeeding
• Oseltamivir: administered orally. Prodrug, hydrolysed by the liver to active metabolite. Extensively metabolised by the
placenta with minimal fetal drug accumulation. Present in low concentration in breast milk
• Use with caution in renal impairment
• Side-effects: Nausea, vomiting, abdominal pain, diarrhoea, headache, conjunctivitis. Less common side-effects
include eczema, hepatitis, GI bleeding, arrhythmia, visual disturbance
• Zanamivir: administered by inhalation of powder (bioavailability is 10-20% after inhaled dose compared to 2% after
oral dose). 90% of absorbed drug excreted unchanged in the urine. Present in breast milk in low concentration.
• Use with caution in patients with asthma and chronic pulmonary disease because of risk of bronchospasm
• Side-effects: bronchospasm, respiratory impairment, angioedema, rticarial, rash
2) M2 protein inhibitors (amantadine, rimantadine): stop the virus infecting cells. High levels of drug resistance have
been observed and not active against influenza B. No longer recommended.

Question 16 Which drug is administered by inhalation to treat infection with H1N1 influenza?

Options for Questions 16-16

This Copy is for Dr. Mohamed ElHodiby


A Acyclovir B Oseltamivir
C Zanamivir D Amantadine
E Rimantadine

A(Correct answ er: C)

Explanation
Influenza – Treatment
Symptomatic treatment including paracetamol for fever and as analgesia
Anti-viral agents:
1) Neuraminidase inhibitors: oseltamivir (Tamiflu ®) and zanamivir (Relenza ®)
• Interfere with the release of new viruses from infected cells.
• Most effective if started within a few hours of the onset of symptoms.
• Licensed for use in adults within 48h of the onset of symptoms but may be administered within 7 days of onset.
• Reduce the duration of symptoms by 1-1.5 days and can reduce the risk of complications in high risk patients
Post-exposure prophylaxis: oseltamivir should be given within 48h of exposure while zanamivir should be given within
36h
Drug treatment in pregnancy
• Zanamivir is the preferred drug during pregnancy
• Oseltamivir recommended in severe infection or when zanamivir cannot be used
• Oseltamivir is the preferred drug in women who are breastfeeding
• Oseltamivir: administered orally. Prodrug, hydrolysed by the liver to active metabolite. Extensively metabolised by the
placenta with minimal fetal drug accumulation. Present in low concentration in breast milk
• Use with caution in renal impairment
• Side-effects: Nausea, vomiting, abdominal pain, diarrhoea, headache, conjunctivitis. Less common side-effects
include eczema, hepatitis, GI bleeding, arrhythmia, visual disturbance
• Zanamivir: administered by inhalation of powder (bioavailability is 10-20% after inhaled dose compared to 2% after
oral dose). 90% of absorbed drug excreted unchanged in the urine. Present in breast milk in low concentration.
• Use with caution in patients with asthma and chronic pulmonary disease because of risk of bronchospasm
• Side-effects: bronchospasm, respiratory impairment, angioedema, rticarial, rash
2) M2 protein inhibitors (amantadine, rimantadine): stop the virus infecting cells. High levels of drug resistance have
been observed and not active against influenza B. No longer recommended.

Which drug is effective against H1N1 influenza but should not be used in women with severe
Question 17
asthma?
Options for Questions 17-17

A Acyclovir B Oseltamivir
C Zanamivir D Amantadine
E Rimantadine

A(Correct answ er: C)

Explanation
Influenza – Treatment
Symptomatic treatment including paracetamol for fever and as analgesia
Anti-viral agents:
1) Neuraminidase inhibitors: oseltamivir (Tamiflu ®) and zanamivir (Relenza ®)
• Interfere with the release of new viruses from infected cells.
• Most effective if started within a few hours of the onset of symptoms.
• Licensed for use in adults within 48h of the onset of symptoms but may be administered within 7 days of onset.
• Reduce the duration of symptoms by 1-1.5 days and can reduce the risk of complications in high risk patients
Post-exposure prophylaxis: oseltamivir should be given within 48h of exposure while zanamivir should be given within
36h
Drug treatment in pregnancy

This Copy is for Dr. Mohamed ElHodiby


• Zanamivir is the preferred drug during pregnancy
• Oseltamivir recommended in severe infection or when zanamivir cannot be used
• Oseltamivir is the preferred drug in women who are breastfeeding
• Oseltamivir: administered orally. Prodrug, hydrolysed by the liver to active metabolite. Extensively metabolised by the
placenta with minimal fetal drug accumulation. Present in low concentration in breast milk
• Use with caution in renal impairment
• Side-effects: Nausea, vomiting, abdominal pain, diarrhoea, headache, conjunctivitis. Less common side-effects
include eczema, hepatitis, GI bleeding, arrhythmia, visual disturbance
• Zanamivir: administered by inhalation of powder (bioavailability is 10-20% after inhaled dose compared to 2% after
oral dose). 90% of absorbed drug excreted unchanged in the urine. Present in breast milk in low concentration.
• Use with caution in patients with asthma and chronic pulmonary disease because of risk of bronchospasm
• Side-effects: bronchospasm, respiratory impairment, angioedema, rticarial, rash
2) M2 protein inhibitors (amantadine, rimantadine): stop the virus infecting cells. High levels of drug resistance have
been observed and not active against influenza B. No longer recommended.

At which stage of pregnancy are women at greatest risk of severe disease following H1N1
Question 18
influenza infection?
Options for Questions 18-18

A First trimester B Second trimester


C Third trimester D 24-48h post-partum
E 4-6 weeks post-partum

A(Correct answ er: C)

Explanation
H1N1 Influenza and pregnancy
• Pregnant women are more likely to have severe disease, be admitted to ITU and die from H1N1 influenza compared
to non-pregnant women. Most reported deaths occurred in women who developed pneumonia and acute respiratory
distress syndrome requiring mechanical ventilation
• Risk of morbidity from seasonal flu is also higher during pregnancy
• The risk is highest in the third trimester compared to early pregnancy or post-partum
• Women with asthma and morbid obesity appear to be at higher risk of severe disease
• Most current data from UKOSS indicates that pregnant women admitted to ITU were less likely to have been treated
with antiviral agents within 2 days of the onset of symptoms when compared with other pregnant women who were
admitted to hospital. This indicates the importance of prompt treatment
• H1N1 infection is associated with pre-term delivery, especially in women admitted to ITU
• Vertical transmission of H1N1 influenza has not been reported and the virus is not known to be present in breast
milk. Breastfeeding should continue in women being treated for H1N1 influenza
• The H1N1 virus is not terratogenic but there some evidence that pyrexia in early pregnancy may be associated with
an increased risk of neural tube defects

Question 19 Infection with the H1N1 influenza virus

Options for Questions 19-19

Occurring in the third trimester is associated with Occurring in the first trimester is associated with
A B
a 10-25% risk of vertical transmission an increased risk of congenital anomalies
Is associated with an increased risk of pre-term Is associated with an increased risk of pre-
C D
delivery eclampsia

This Copy is for Dr. Mohamed ElHodiby


E Causes less severe disease in obese women

A(Correct answ er: C)

Explanation
1N1 Influenza and pregnancy
• Pregnant women are more likely to have severe disease, be admitted to ITU and die from H1N1 influenza compared
to non-pregnant women. Most reported deaths occurred in women who developed pneumonia and acute respiratory
distress syndrome requiring mechanical ventilation
• Risk of morbidity from seasonal flu is also higher during pregnancy
• The risk is highest in the third trimester compared to early pregnancy or post-partum
• Women with asthma and morbid obesity appear to be at higher risk of severe disease
• Most current data from UKOSS indicates that pregnant women admitted to ITU were less likely to have been treated
with antiviral agents within 2 days of the onset of symptoms when compared with other pregnant women who were
admitted to hospital. This indicates the importance of prompt treatment
• H1N1 infection is associated with pre-term delivery, especially in women admitted to ITU
• Vertical transmission of H1N1 influenza has not been reported and the virus is not known to be present in breast
milk. Breastfeeding should continue in women being treated for H1N1 influenza
• The H1N1 virus is not terratogenic but there some evidence that pyrexia in early pregnancy may be associated with
an increased risk of neural tube defects

Question 20 With respect to the influenza vaccine

Options for Questions 20-20

Vaccinated women cannot get influenza during


A B Vaccination provides life-long immunity
pregnancy
Seasonal flu vaccine should be administered to all During a pandemic, vaccination should not occur
C D
pregnant women during pregnancy
Flu-like symptoms are a recognised side-effect of
E
vaccination

A(Correct answ er: E)

Explanation
Influenza Vaccine
• Can be a live vaccine or an inactivated vaccine
• Produced from viruses grown in eggs although alternative methods are under investigation
• Due to antigenic shift / drift, the vaccine only confers protection for a few years
• Yearly prediction of the strains of virus likely to pose a threat in the next flu season allows development of appropriate
vaccine against seasonal influenza
• Seasonal influenza vaccination is recommended for vulnerable individuals including children, the elderly, asthmatics,
those with heart disease, diabetes mellitus, chronic renal, liver or neurological disease or immune compromise
• Protection is conferred about 2 weeks after vaccination
• Vaccination does not protect against all strains of the virus so it is still possible to get influenza after vaccination
• Side-effects of the vaccine include influenza-like symptoms (milder and shorter-lasting) and allergic reactions to the
vaccine or egg proteins
• A 2007 Cochrane review on influenza vaccines in healthy adults found that while vaccines were effective against the
influenza strains they are designed to vaccinate against, this ended up translating to only a modest impact on working
days lost due to influenza-like infections
• Vaccine efficacy declines with age, so elderly patients who are most at risk from influenza also benefit least from
vaccination
• Seasonal flu vaccine is administered by injection or nasal spray
H1N1 Vaccine

This Copy is for Dr. Mohamed ElHodiby


• Pandemrix™ (GSK) is a split virion, inactivated, adjuvanted vaccine and is indicated for the prophylaxis of influenza
in an officially declared pandemic situation.
• The adjuvant stimulates a better immune response and contains squalene and vitamin E
• Single dose recommended in adults and should be administered by intramuscular injection preferably into the deltoid
muscle or anterolateral thigh (depending on the muscle mass). If a second dose is administered there should be an
interval of at least three weeks between the first and the second dose.
Contraindications
• Contraindicated in individuals with a history of anaphylactic (i.e. life-threatening) reaction to any of the constituents or
trace residues of this vaccine:
• egg and chicken protein
• ovalbumin
• formaldehyde
• gentamicin sulphate and
• sodium deoxycholate
• If vaccination is considered necessary, facilities for resuscitation should be immediately available in case of need.

This Copy is for Dr. Mohamed ElHodiby

You might also like